Download as pdf or txt
Download as pdf or txt
You are on page 1of 366

Question 2 of 243

A patient undergoes a femoral hernia repair and at operation the surgeon decides to enter the
abdominal cavity to resect small bowel. She makes a transverse incision two thirds of the way
between umbilicus and the symphysis pubis. Which of the structures listed below is least likely to be
divided?

Rectus abdominis

h
External oblique aponeurosis

Peritoneum

la
Fascia transversalis
Sa
Posterior lamina of the rectus sheath

An incision at this level lies below the arcuate line and the posterior wall of the rectus sheath is
deficient at this level.
Please rate this question:
C
Discuss and give feedback
Next question
R

Rectus abdominis muscle

The rectus sheath is formed by the aponeuroses of the lateral abdominal wall muscles. The rectus
sheath has a composition that varies according to anatomical level.
M

1. Above the costal margin the anterior sheath is composed of external oblique aponeurosis, the
costal cartilages are posterior to it.
2. From the costal margin to the arcuate line, the anterior rectus sheath is composed of external
oblique aponeurosis and the anterior part of the internal oblique aponeurosis. The posterior part of
the internal oblique aponeurosis and transversus abdominis form the posterior rectus sheath.
3. Below the arcuate line the aponeuroses of all the abdominal muscles lie in anterior aspect of the
rectus sheath. Posteriorly lies the transversalis fascia and peritoneum.

The arcuate line is the point at which the inferior epigastric vessels enter the rectus sheath.
Question 3 of 243

What is the lymphatic drainage of the ovaries?

Internal iliac nodes

h
Common iliac nodes

la
Para-aortic nodes

Para uterine nodes


Sa
Inguinal nodes

The lymphatic drainage of the ovary follows the gonadal vessels and drainage is therefore to the
C
para-aortic nodes.
Please rate this question:
R

Discuss and give feedback

Next question
M

Lymphatic drainage of the ovaries, uterus and cervix

• The ovaries drain to the para-aortic lymphatics via the gonadal vessels.
• The uterine fundus has a lymphatic drainage that runs with the ovarian vessels and may thus
drain to the para-aortic nodes. Some drainage may also pass along the round ligament to the
inguinal nodes.
• The body of the uterus drains through lymphatics contained within the broad ligament to the
iliac lymph nodes.
• The cervix drains into three potential nodal stations; laterally through the broad ligament to
the external iliac nodes, along the lymphatics of the uterosacral fold to the presacral nodes
and posterolaterally along lymphatics lying alongside the uterine vessels to the internal iliac
nodes.

Next question

h
la
Sa
C
R
M
Question 4 of 243

A 23 year old man undergoes an orchidectomy. The right testicular vein is ligated; into which
structure does it drain?

Right renal vein

h
Inferior vena cava

Common iliac vein

la
Internal iliac vein

External iliac vein


Sa
The testicular venous drainage begins in the septa and these veins together with those of the tunica
vasculosa converge on the posterior border of the testis as the pampiniform plexus. The
pampiniform plexus drains to the testicular vein. The left testicular vein drains into the left renal vein.
The right testicular vein drains into the inferior vena cava.
C
Please rate this question:

Discuss and give feedback


R

Next question

Scrotal and testicular anatomy

Spermatic cord
M

Formed by the vas deferens and is covered by the following structures:


Layer Origin

Internal spermatic fascia Transversalis fascia

Cremasteric fascia From the fascial coverings of internal oblique


Layer Origin

External spermatic fascia External oblique aponeurosis

Contents of the cord


Vas deferens Transmits sperm and accessory gland secretions

h
Testicular artery Branch of abdominal aorta supplies testis and
epididymis

la
Artery of vas deferens Arises from inferior vesical artery

Cremasteric artery Arises from inferior epigastric artery

Pampiniform plexus
Sa Venous plexus, drains into right or left testicular vein

Sympathetic nerve fibres Lie on arteries, the parasympathetic fibres lie on the
vas
C
Genital branch of the genitofemoral Supplies cremaster
nerve
R

Lymphatic vessels Drain to lumbar and para-aortic nodes


M

Scrotum

• Composed of skin and closely attached dartos fascia.


• Arterial supply from the anterior and posterior scrotal arteries
• Lymphatic drainage to the inguinal lymph nodes
• Parietal layer of the tunica vaginalis is the innermost layer

Testes
• The testes are surrounded by the tunica vaginalis (closed peritoneal sac). The parietal layer
of the tunica vaginalis adjacent to the internal spermatic fascia.
• The testicular arteries arise from the aorta immediately inferiorly to the renal arteries.
• The pampiniform plexus drains into the testicular veins, the left drains into the left renal vein
and the right into the inferior vena cava.
• Lymphatic drainage is to the para-aortic nodes.

Next question

h
la
Sa
C
R
M
Question 5 of 243

A 44 year old lady is undergoing an abdominal hysterectomy and the ureter is identified during the
ligation of the uterine artery. At which site does it insert into the bladder?

Posterior

h
Apex

Anterior

la
Base Sa
Superior aspect of the lateral side

The ureters enter the bladder at the upper lateral aspect of the base of the bladder. They are about
5cm apart from each other in the empty bladder. Internally this aspect is contained within the bladder
trigone.
Please rate this question:
C
Discuss and give feedback
Next question
R

Ureter

• 25-35 cm long
• Muscular tube lined by transitional epithelium
M

• Surrounded by thick muscular coat. Becomes 3 muscular layers as it crosses the bony pelvis
• Retroperitoneal structure overlying transverse processes L2-L5
• Lies anterior to bifurcation of iliac vessels
• Blood supply is segmental; renal artery, aortic branches, gonadal branches, common iliac
and internal iliac
• Lies beneath the uterine artery

Next question
Question 6 of 243

A 6 month old child is brought to the surgical clinic because of non descended testes. What is the
main structure that determines the descent path of the testicle?

Processus vaginalis

h
Cremaster

la
Mesorchium

Inguinal canal

Gubernaculum
Sa
The gubernaculum is a ridge of mesenchymal tissue that connects the testis to the inferior aspect of
the scrotum. Early in embryonic development the gubernaculum is long and the testis are located on
C
the posterior abdominal wall. During foetal growth the body grows relative to the gubernaculum, with
resultant descent of the testis.
Please rate this question:
R

Discuss and give feedback


M

Next question

Testicular embryology

Until the end of foetal life the testicles are located within the abdominal cavity. They are initially
located on the posterior abdominal wall on a level with the upper lumbar vertebrae (L2). Attached to
the inferior aspect of the testis is the gubernaculum testis which extends caudally to the inguinal
region, through the canal and down to the superficial skin. Both the testis and the gubernaculum are
extra-peritoneal.
As the foetus grows the gubernaculum becomes progressively shorter. It carries the peritoneum of
the anterior abdominal wall (the processus vaginalis). As the processus vaginalis descends the testis
is guided by the gubernaculum down the posterior abdominal wall and the back of the processus
vaginalis into the scrotum.
By the third month of foetal life the testes are located in the iliac fossae, by the seventh they lie at
the level of the deep inguinal ring.

The processus vaginalis usually closes after birth, but may persist and be the site of indirect hernias.
Part closure may result in development of cysts on the cord.

h
la
Sa
C
R
M
Question 7 of 243

What is the most important structure involved in supporting the uterus?

Round ligament

Broad ligament

h
Uterosacral ligaments

la
Cardinal ligaments
Sa
Central perineal tendon

The central perineal tendon provides the main structural support to the uterus. Damage to this
structure is commonly associated with the development of pelvic organ prolapse, even when other
structures are intact.
Please rate this question:
C

Discuss and give feedback


R

Next question

Uterus
M

The non pregnant uterus resides entirely within the pelvis. The peritoneum invests the uterus and
the structure is contained within the peritoneal cavity. The blood supply to the uterine body is via the
uterine artery (branch of the internal iliac). The uterine artery passes from the inferior aspect of the
uterus (lateral to the cervix) and runs alongside the uterus. It frequently anastomoses with the
ovarian artery superiorly. Inferolaterally the ureter is a close relation and ureteric injuries are a
recognised complication when pathology brings these structures into close proximity.

The supports of the uterus include the central perineal tendon (the most important). The lateral
cervical, round and uterosacral ligaments are condensations of the endopelvic fascia and provide
additional structural support.
Topography of the uterus

h
Image sourced from Wikipedia

la
Sa
C
R
M
Question 8 of 243

What is the lymphatic drainage of the male spongy urethra?

h
External iliac nodes

Internal iliac nodes

la
Para aortic nodes Sa
Deep inguinal nodes

Meso rectal nodes


C
The lymphatic drainage of the spongy urethra and the glans penis is to the deep inguinal nodes. The
prostatic and membranous urethra drains to the internal iliac nodes.
Please rate this question:
R

Discuss and give feedback


M

Next question

Urethral anatomy

Female urethra
The female urethra is shorter and more acutely angulated than the male urethra. It is an extra-
peritoneal structure and embedded in the endopelvic fascia. The neck of the bladder is subjected to
transmitted intra-abdominal pressure and therefore deficiency in this area may result in stress
urinary incontinence. Between the layers of the urogenital diaphragm the female urethra is
surrounded by the external urethral sphincter, this is innervated by the pudendal nerve. It ultimately
lies anterior to the vaginal orifice.
Male urethra
In males the urethra is much longer and is divided into four parts.

Pre-prostatic Extremely short and lies between the bladder and prostate gland.It has a stellate lumen
urethra and is between 1 and 1.5cm long.Innervated by sympathetic noradrenergic fibres, as
this region is composed of striated muscles bundles they may contract and prevent
retrograde ejaculation.

Prostatic This segment is wider than the membranous urethra and contains several openings for

h
urethra the transmission of semen (at the midpoint of the urethral crest).

Membranous Narrowest part of the urethra and surrounded by external sphincter. It traverses the

la
urethra perineal membrane 2.5cm postero-inferior to the symphysis pubis.

Penile urethra Travels through the corpus spongiosum on the underside of the penis. It is the longest
Sa
urethral segment.It is dilated at its origin as the infrabulbar fossa and again in the gland
penis as the navicular fossa. The bulbo-urethral glands open into the spongiose section
of the urethra 2.5cm below the perineal membrane.

The urothelium is transitional in nature near to the bladder and becomes squamous more distally.
C
R
M
Question 4 of 237

A 28 year old man requires a urethral catheter to be inserted prior to undergoing a splenectomy.
Where is the first site of resistance to be encountered on inserting the catheter?

Bulbar urethra

h
Membranous urethra

la
Internal sphincter

Prostatic urethra Sa
Bladder neck

The membranous urethra is the least distensible portion of the urethra. This is due to the fact that it
is surrounded by the external sphincter.
Please rate this question:
C
R

Discuss and give feedback

Next question

Urethral anatomy
M

Female urethra
The female urethra is shorter and more acutely angulated than the male urethra. It is an extra-
peritoneal structure and embedded in the endopelvic fascia. The neck of the bladder is subjected to
transmitted intra-abdominal pressure and therefore deficiency in this area may result in stress
urinary incontinence. Between the layers of the urogenital diaphragm the female urethra is
surrounded by the external urethral sphincter, this is innervated by the pudendal nerve. It ultimately
lies anterior to the vaginal orifice.

Male urethra
In males the urethra is much longer and is divided into four parts.
Pre-prostatic Extremely short and lies between the bladder and prostate gland.It has a stellate lumen
urethra and is between 1 and 1.5cm long.Innervated by sympathetic noradrenergic fibres, as
this region is composed of striated muscles bundles they may contract and prevent
retrograde ejaculation.

Prostatic This segment is wider than the membranous urethra and contains several openings for
urethra the transmission of semen (at the midpoint of the urethral crest).

Membranous Narrowest part of the urethra and surrounded by external sphincter. It traverses the

h
urethra perineal membrane 2.5cm postero-inferior to the symphysis pubis.

la
Penile urethra Travels through the corpus spongiosum on the underside of the penis. It is the longest
urethral segment.It is dilated at its origin as the infrabulbar fossa and again in the gland
penis as the navicular fossa. The bulbo-urethral glands open into the spongiose section
of the urethra 2.5cm below the perineal membrane.
Sa
The urothelium is transitional in nature near to the bladder and becomes squamous more distally.
C
R
M
Question 3 of 235

A 21 year old man has an inguinal hernia and is undergoing a surgical repair. As the surgeons
approach the inguinal canal they expose the superficial inguinal ring. Which of the following forms
the lateral edge of this structure?

Inferior epigastric artery

h
Conjoint tendon

la
Rectus abdominis muscle

External oblique aponeurosis


Sa
Transversalis fascia

The external oblique aponeurosis forms the anterior wall of the inguinal canal and also the lateral
edge of the superficial inguinal ring. The rectus abdominis lies posteromedially and the transversalis
posterior to this.
C
Please rate this question:

Discuss and give feedback


R

Next question

Inguinal canal

Location
M

• Above the inguinal ligament


• The inguinal canal is 4cm long
• The superficial ring is located anterior to the pubic tubercle
• The deep ring is located approximately 1.5-2cm above the half way point between the
anterior superior iliac spine and the pubic tubercle

Boundaries of the inguinal canal


Floor • External oblique aponeurosis
• Inguinal ligament
• Lacunar ligament

Roof • Internal oblique


• Transversus abdominis

Anterior wall External oblique aponeurosis

h
Posterior wall • Transversalis fascia
• Conjoint tendon

la
Laterally • Internal ring
• Transversalis fascia
Sa • Fibres of internal oblique

Medially • External ring


• Conjoint tendon

Contents
Males Spermatic cord and ilioinguinal As it passes through the canal the spermatic cord
C
nerve has 3 coverings:

• External spermatic fascia from external


R

oblique aponeurosis
• Cremasteric fascia
• Internal spermatic fascia
M

Females Round ligament of uterus and


ilioinguinal nerve

Related anatomy of the inguinal region


The boundaries of Hesselbachs triangle are commonly tested and illustrated below:
h
la
Sa
Image sourced from Wikipedia

The image below demonstrates the close relationship of the vessels to the lower limb with the
inguinal canal. A fact to be borne in mind when repairing hernial defects in this region.
C
R
M

Image sourced from Wikipedia

Next question
Question 4 of 235

Which of the structures listed below accompanies the aorta as it traverses the aortic hiatus?

Oesophagus

Thoracic duct

h
Vagal trunks

la
Right phrenic nerve

Left phrenic nerve


Sa
The aorta is accompanied by the thoracic duct as it traverses the aortic hiatus. The vagal trunks
accompany the oesophagus which passes through the muscular part of the diaphragm on the right.
The right phrenic nerve accompanies the IVC as it passes through the caval opening. The left
phrenic nerve passes through the muscular part of the diaphragm anterior to the central tendon on
C
the left.
Please rate this question:
R

Discuss and give feedback


M

Next question

Abdominal aorta

Abdominal aortic topography

Origin T12

Termination L4
Posterior relations L1-L4 Vertebral bodies

Anterior relations Lesser omentum


Liver
Left renal vein
Inferior mesenteric vein
Third part of duodenum
Pancreas
Parietal peritoneum
Peritoneal cavity

h
Right lateral relations Right crus of the diaphragm

la
Cisterna chyli
IVC (becomes posterior distally)

Left lateral relations


Sa
4th part of duodenum
Duodenal-jejunal flexure
Left sympathetic trunk

The abdominal aorta


C
R
M
h
la
Sa
Image sourced from Wikipedia

Next question
C
Display my notes on this topic
R
M

Save my notes
Question 7 of 235

A 45 year old lady is undergoing a Whipples procedure for carcinoma of the pancreatic head. The
bile duct is transected. Which of the following vessels is mainly responsible for the blood supply to
the bile duct remnant?

Cystic artery

h
Hepatic artery

la
Portal vein

Left gastric artery


Sa
None of the above

Do not confuse the blood supply of the bile duct with that of the cystic duct.
C
The bile duct has an axial blood supply which is derived from the hepatic artery and from
retroduodenal branches of the gastroduodenal artery. Unlike the liver there is no contribution by the
portal vein to the blood supply of the bile duct. Damage to the hepatic artery during a difficult
cholecystectomy is a recognised cause of bile duct strictures. In this scenario the distal vessels have
R

been removed as the patient is undergoing a resection.


Please rate this question:

Discuss and give feedback


M

Next question

Gallbladder

• Fibromuscular sac with capacity of 50ml


• Columnar epithelium

Relations of the gallbladder


Anterior Liver

Posterior • Covered by peritoneum


• Transverse colon
• 1st part of the duodenum

Laterally Right lobe of liver

Medially Quadrate lobe of liver

h
Arterial supply
Cystic artery (branch of Right hepatic artery)

la
Venous drainage
Directly to the liver

Nerve supply
Sympathetic- mid thoracic spinal cord, Parasympathetic- anterior vagal trunk

Common bile duct


Sa
Origin Confluence of cystic and common hepatic ducts

Relations at • Medially - Hepatic artery


C
origin • Posteriorly- Portal vein

Relations distally • Duodenum - anteriorly


R

• Pancreas - medially and laterally


• Right renal vein - posteriorly
M

Arterial supply Branches of hepatic artery and retroduodenal branches of gastroduodenal


artery

Hepatobiliary triangle

Medially Common hepatic duct

Inferiorly Cystic duct


Superiorly Inferior edge of liver

Contents Cystic artery

Relations of the gallbladder

h
la
Sa
© Image provided by the University of Sheffield

Next question
C
Display my notes on this topic
R
M

Save my notes
Question 8 of 235

What vessel is the origin of the middle rectal artery?

Aorta

h
Inferior mesenteric artery

la
Superior mesenteric artery
Sa
Internal iliac artery

Internal pudendal artery

The rectum is supplied by 3 main vessels


C
• Superior rectal artery from inferior mesenteric artery
• Middle rectal artery from the internal iliac artery
• Inferior rectal artery from the internal pudendal artery
R

Please rate this question:


M

Discuss and give feedback


Next question

Rectum

The rectum is approximately 12 cm long. It is a capacitance organ. It has both intra and
extraperitoneal components. The transition between the sigmoid colon is marked by the
disappearance of the tenia coli.The extra peritoneal rectum is surrounded by mesorectal fat that also
contains lymph nodes. This mesorectal fatty layer is removed surgically during rectal cancer surgery
(Total Mesorectal Excision). The fascial layers that surround the rectum are important clinical
landmarks, anteriorly lies the fascia of Denonvilliers. Posteriorly lies Waldeyers fascia.
Extra peritoneal rectum

• Posterior upper third


• Posterior and lateral middle third
• Whole lower third

Relations
Anteriorly (Males) Rectovesical pouch
Bladder
Prostate

h
Seminal vesicles

la
Anteriorly (Females) Recto-uterine pouch (Douglas)
Cervix
Vaginal wall

Posteriorly
Sa Sacrum
Coccyx
Middle sacral artery

Laterally Levator ani


Coccygeus
C
Arterial supply
Superior rectal artery
R

Venous drainage
Superior rectal vein

Lymphatic drainage
M

• Mesorectal lymph nodes (superior to dentate line)


• Inguinal nodes (inferior to dentate line)

Next question

Display my notes on this topic


Question 9 of 235

A 7 year old boy presents with right iliac fossa pain and there is a clinical suspicion that appendicitis
is present. From which of the following embryological structures is the appendix derived?

Vitello-intestinal duct

Uranchus

h
Foregut

la
Hindgut

Midgut
Sa
The appendix is derived from the midgut

It is derived from the midgut which is why early appendicitis may present with periumbilical pain.
Please rate this question:
C
Discuss and give feedback
Next question

Appendix
R

• Location: Base of caecum.


• Up to 10cm long.
• Mainly lymphoid tissue (Hence mesenteric adenitis may mimic appendicitis).
• Caecal taenia coli converge at base of appendix and form a longitudinal muscle cover over
M

the appendix. This convergence should facilitate its identification at surgery if it is retrocaecal
and difficult to find (which it can be when people start doing appendicectomies!)
• Arterial supply: Appendicular artery (branch of the ileocolic).
• It is intra peritoneal.

McBurney's point

• 1/3 of the way along a line drawn from the Anterior Superior Iliac Spine to the Umbilicus
6 Positions:

• Retrocaecal 74%
• Pelvic 21%
• Postileal
• Subcaecal
• Paracaecal
• Preileal

Next question

h
la
Display my notes on this topic

Sa
Save my notes
C
R
M
Question 10 of 235

You decide to take an arterial blood gas from the femoral artery. Where should the needle be
inserted to gain the sample?

1-2 cm inferiorly to the mid point of the inguinal ligament

h
1-2cm inferiorly to the mid inguinal point

la
2cm inferomedially to the pubic tubercle

2cm superomedially to the pubic tubercle


Sa
3cm inferolaterally to the deep inguinal ring

The mid inguinal point is midway between the anterior superior iliac spine and the symphysis pubis

The mid inguinal point in the surface marking for the femoral artery.
Please rate this question:
C
Discuss and give feedback
Next question
R

Femoral triangle anatomy

Boundaries
M

Superiorly Inguinal ligament

Laterally Sartorius

Medially Adductor longus

Floor Iliopsoas, adductor longus and pectineus


Roof • Fascia lata and Superficial fascia
• Superficial inguinal lymph nodes (palpable below the inguinal ligament)
• Long saphenous vein

h
la
Sa
C

Image sourced from Wikipedia


R

Contents
M

• Femoral vein (medial to lateral)


• Femoral artery-pulse palpated at the mid inguinal point
• Femoral nerve
• Deep and superficial inguinal lymph nodes
• Lateral cutaneous nerve
• Great saphenous vein
• Femoral branch of the genitofemoral nerve

Next question
Question 11 of 235

A 25 year old man has an inguinal hernia, which of the following structures must be divided (at open
surgery) to gain access to the inguinal canal?

Transversalis fascia

External oblique aponeurosis

h
Conjoint tendon

la
Rectus abdominis

Inferior epigastric artery


Sa
This question is asking what structure forms the anterior wall of the inguinal canal. The anterior wall
is formed by the external oblique aponeurosis. Once this is divided the canal is entered, the cord can
be mobilised and a hernia repair performed. The transversalis fascia and conjoint tendons form the
posterior wall and would not routinely be divided to gain access to the inguinal canal itself.
Please rate this question:
C
Discuss and give feedback
Next question
R

External oblique muscle

External oblique forms the outermost muscle of the three muscles comprising the anterolateral
aspect of the abdominal wall. Its aponeurosis comprises the anterior wall of the inguinal canal.
M

Origin Outer surfaces of the lowest eight ribs

Insertion • Anterior two thirds of the outer lip of the iliac crest.
• The remainder becomes the aponeurosis that fuses with the linea alba in the
midline.
Nerve Ventral rami of the lower six thoracic nerves
supply

Actions Contains the abdominal viscera, may contract to raise intra abdominal pressure.
Moves trunk to one side.

Next question

h
la
Sa
C
R
M
Question 12 of 235

A 67 year old man has an abdominal aortic aneurysm which displaces the left renal vein. Which
branch of the aorta is most likely to be affected at this level?

Inferior mesenteric artery

h
Superior mesenteric artery

la
Coeliac axis

Testicular artery Sa
None of the above

The left renal vein lies behind of the SMA as it branches off the aorta. Whilst juxtarenal AAA may
sometimes require the division of the left renal vein, direct involvement of the SMA may require a
hybrid surgical bypass and subsequent endovascular occlusion.
C
Please rate this question:
R

Discuss and give feedback

Next question
M

Abdominal aortic branches

Branches Level Paired Type

Inferior phrenic T12 (Upper border) Yes Parietal

Coeliac T12 No Visceral


Branches Level Paired Type

Superior mesenteric L1 No Visceral

Middle suprarenal L1 Yes Visceral

Renal L1-L2 Yes Visceral

h
Gonadal L2 Yes Visceral

la
Lumbar L1-L4 Yes Parietal

Inferior mesenteric L3 No Visceral


Sa
Median sacral L4 No Parietal

Common iliac L4 Yes Terminal


C
Next question
R
M
Question 13 of 235

What is the anatomical level of the transpyloric plane?

T11

T12

h
L1

la
L4

T10

Please rate this question:


Sa
Discuss and give feedback
Next question
C
Transpyloric plane

Transpyloric plane
Level of the body of L1
R

• Pylorus stomach
• Left kidney hilum (L1- left one!)
M

• Right hilum of the kidney (1.5cm lower than the left)


• Fundus of the gallbladder
• Neck of pancreas
• Duodenojejunal flexure
• Superior mesenteric artery
• Portal vein
• Left and right colic flexure
• Root of the transverse mesocolon
• 2nd part of the duodenum
• Upper part of conus medullaris
• Spleen
• Question 15 of 235

• The pudendal canal is a fascial canal located on the lateral wall of the ischioanal fossa. In
this location, it lies on the inferior border of which of the following muscles?

Coccygeus

Obturator internus

h
Pubococcygeus

Iliococcygeus

la
Piriformis


Sa
The coccygeus, pubococcygeus and iliococcygeus form part of the pelvic diaphragm and are
not related to it. The piriformis exits the pelvis via the greater sciatic foramen and is not
associated with the canal in the ischiorectal fossa.
• Please rate this question:


C
• Discuss and give feedback

• Next question
R

• Pudendal canal

• The pudendal canal is located along the lateral wall of the ischioanal fossa at the inferior
margin of the obturator internus muscle. It extends from the lesser sciatic foramen to the
M

posterior margin of the urogenital diaphragm. It conveys the internal pudendal vessels and
nerve.
• Next question
Question 17 of 235

What is the lymphatic drainage of the upper ureter?

Common iliac nodes

Para aortic nodes

h
External iliac nodes

la
Internal iliac nodes

Meso colic nodes


Sa
The upper ureter drains to the para-aortic nodes, the lower ureter drains to the common iliac nodes.
Please rate this question:
C
Discuss and give feedback
Next question

Ureter
R

• 25-35 cm long
• Muscular tube lined by transitional epithelium
• Surrounded by thick muscular coat. Becomes 3 muscular layers as it crosses the bony pelvis
• Retroperitoneal structure overlying transverse processes L2-L5
M

• Lies anterior to bifurcation of iliac vessels


• Blood supply is segmental; renal artery, aortic branches, gonadal branches, common iliac
and internal iliac
• Lies beneath the uterine artery

Next question
Question 21 of 235

A 53 year old man is undergoing a radical gastrectomy for carcinoma of the stomach. Which of the
following structures will need to be divided to gain access to the coeliac axis?

Lesser omentum

h
Greater omentum

la
Falciform ligament

Median arcuate ligament


Sa
Gastrosplenic ligament

The lesser omentum will need to be divided. During a radical gastrectomy this forms one of the
nodal stations that will need to be taken.
Please rate this question:
C
Discuss and give feedback
Next question
R

Coeliac axis

The coeliac axis has three main branches.


M

• Left gastric
• Hepatic: branches-Right Gastric, Gastroduodenal, Superior Pancreaticoduodenal, Cystic
(occasionally).
• Splenic: branches- Pancreatic, Short Gastric, Left Gastroepiploic

It occasionally gives off one of the inferior phrenic arteries.


h
Image sourced from Wikipedia

la
Relations
Anteriorly Lesser omentum
Sa
Right Right coeliac ganglion and caudate process of liver

Left Left coeliac ganglion and gastric cardia


C
Inferiorly Upper border of pancreas and renal vein

Next question
R
M
Question 22 of 235

A 17 year old lady presents with right iliac fossa pain and diagnosed as having acute appendicitis.
You take her to theatre to perform a laparoscopic appendicectomy. During the procedure the scrub
nurse distracts you and you inadvertently avulse the appendicular artery. The ensuing haemorrhage
is likely to be supplied directly from which vessel?

Inferior mesenteric artery

h
Superior mesenteric artery

la
Ileo-colic artery
Sa
Internal iliac artery

None of the above

The appendicular artery is a branch of the ileocolic artery.


C
Please rate this question:

Discuss and give feedback


R

Next question

Appendix

• Location: Base of caecum.


M

• Up to 10cm long.
• Mainly lymphoid tissue (Hence mesenteric adenitis may mimic appendicitis).
• Caecal taenia coli converge at base of appendix and form a longitudinal muscle cover over
the appendix. This convergence should facilitate its identification at surgery if it is retrocaecal
and difficult to find (which it can be when people start doing appendicectomies!)
• Arterial supply: Appendicular artery (branch of the ileocolic).
• It is intra peritoneal.

McBurney's point
• 1/3 of the way along a line drawn from the Anterior Superior Iliac Spine to the Umbilicus

6 Positions:

• Retrocaecal 74%
• Pelvic 21%
• Postileal
• Subcaecal
• Paracaecal
• Preileal

h
Next question

la
Sa
C
R
M
Question 23 of 235

Which of the vessels listed below is the most inferiorly sited single aortic branch?

Common iliac artery

h
Inferior mesenteric artery

la
Superior mesenteric artery

Gonadal artery
Sa
Median sacral artery

The median sacral artery leaves the aorta a little above its bifurcation. It descends in the midline
anterior to L4 and L5.
C
Please rate this question:
R

Discuss and give feedback

Next question
M

Abdominal aortic branches

Branches Level Paired Type

Inferior phrenic T12 (Upper border) Yes Parietal

Coeliac T12 No Visceral


Branches Level Paired Type

Superior mesenteric L1 No Visceral

Middle suprarenal L1 Yes Visceral

Renal L1-L2 Yes Visceral

h
Gonadal L2 Yes Visceral

la
Lumbar L1-L4 Yes Parietal

Inferior mesenteric L3 No Visceral


Sa
Median sacral L4 No Parietal

Common iliac L4 Yes Terminal


C
R
M
Question 24 of 235

A 63 year old man who smokes heavily presents with dyspepsia. He is tested and found to be
positive for helicobacter pylori infection. One evening he has an episode of haematemesis and
collapses. What is the most likely vessel to be responsible?

h
Portal vein

Short gastric arteries

la
Superior mesenteric artery
Sa
Gastroduodenal artery

None of the above


C
He is most likely to have a posteriorly sited duodenal ulcer. These can invade the gastroduodenal
artery and present with major bleeding. Although gastric ulcers may invade vessels they do not tend
to produce major bleeding of this nature.
Please rate this question:
R

Discuss and give feedback


M

Next question

Gastroduodenal artery

Supplies
Pylorus, proximal part of the duodenum, and indirectly to the pancreatic head (via the anterior and
posterior superior pancreaticoduodenal arteries)

Path
The gastroduodenal artery most commonly arises from the common hepatic artery of the coeliac
trunk. It terminates by bifurcating into the right gastroepiploic artery and the superior
pancreaticoduodenal artery

h
la
Sa
C
R
M
Question 25 of 235

Which of the following nerves is responsible for the cremasteric reflex?

Lateral femoral cutaneous nerve

Femoral nerve

h
Obturator nerve

la
Genitofemoral nerve

None of the above


Sa
The motor and sensory fibres of the genitofemoral nerve are tested in the cremasteric reflex. A small
contribution is also played by the ilioinguinal nerve and thus the reflex may be lost following an
inguinal hernia repair.
Please rate this question:
C
Discuss and give feedback
Next question

Genitofemoral nerve
R

Supplies
Small area of the upper medial thigh.

Path
M

• Arises from the first and second lumbar nerves.


• Passes obliquely through psoas major, and emerges from its medial border opposite the
fibrocartilage between the third and fourth lumbar vertebrae.
• It then descends on the surface of psoas major, under cover of the peritoneum
• Divides into genital and femoral branches.
• The genital branch passes through the inguinal canal, within the spermatic cord, to supply
the skin and fascia of the scrotum. The femoral branch enters the thigh posterior to the
inguinal ligament, lateral to the femoral artery. It supplies an area of skin and fascia over the
femoral triangle.
• It may be injured during abdominal or pelvic surgery, or during inguinal hernia repairs.
Question 28 of 235

Which of the following is not found within the deep perineal pouch in an adult male?

Pudendal nerve

Dorsal nerve of the penis

Sphinter urethrae

h
Urethral artery

la
Obturator nerve

Please rate this question:


Sa
Discuss and give feedback
Next question

Deep perineal pouch


C
The perineal pouch is surrounded inferiorly by the inferior fascia of the urogenital diaphragm. This
fascial boundary extends laterally to form the medial wall of the ischiorectal fossa. The pouch is
bounded superiorly by the superior fascia of the urogenital diaphragm and this lies beneath the
levator ani muscle.
R

Contents of the deep perineal pouch

• Urethral sphincter
M

• Transversus perinei
• Dorsal nerve of penis, muscular branches of the perineal nerve
• Deep and dorsal arteries of penis, stem of origin of artery to the bulb of penis, urethral artery.
Question 31 of 235

A 73 year old lady is admitted with brisk rectal bleeding. Despite attempts at resuscitation the
bleeding proceeds to cause haemodynamic compromise. An upper GI endoscopy is normal. A
mesenteric angiogram is performed and a contrast blush is seen in the region of the sigmoid colon.

h
The radiologist decides to embolise the vessel supplying this area. At what spinal level does it leave
the aorta?

la
L2

L1 Sa
L4

L3
C
T10

The inferior mesenteric artery leaves the aorta at L3. It supplies the left colon and sigmoid. Its
R

proximal continuation to communicate with the middle colic artery is via the marginal artery.
Please rate this question:

Discuss and give feedback


M

Next question

Levels

Transpyloric plane
Level of the body of L1

• Pylorus stomach
• Left kidney hilum (L1- left one!)
• Fundus of the gallbladder
• Neck of pancreas
• Duodenojejunal flexure
• Superior mesenteric artery
• Portal vein
• Left and right colic flexure
• Root of the transverse mesocolon
• 2nd part of the duodenum
• Upper part of conus medullaris
• Spleen

Can be identified by asking the supine patient to sit up without using their arms. The plane is located
where the lateral border of the rectus muscle crosses the costal margin.

h
Anatomical planes
Subcostal plane Lowest margin of 10th costal cartilage

la
Intercristal plane Level of body L4 (highest point of iliac crest)

Intertubercular plane
Sa Level of body L5

Common level landmarks


Inferior mesenteric artery L3
C
Bifurcation of aorta into common iliac arteries L4
R

Formation of IVC L5 (union of common iliac veins)

Diaphragm apertures • Vena cava T8



M

Oesophagus T10
• Aortic hiatus T12
Question 35 of 235

A 43 year old man is undergoing a right hemicolectomy and the ileo-colic artery is ligated. From
which of the following vessels is it derived?

Inferior mesenteric artery

h
Superior mesenteric artery

la
Coeliac axis

Aorta Sa
None of the above

The ileocolic artery is a branch of the SMA and supplies the right colon and terminal ileum. The
transverse colon is supplied by the middle colic artery. As veins accompany arteries in the
mesentery and are lined by lymphatics, high ligation is the norm in cancer resections. The ileo-colic
C
artery branches off the SMA near the duodenum.
Please rate this question:
R

Discuss and give feedback

Next question
M

Colon anatomy

The colon commences with the caecum. This represents the most dilated segment of the human
colon and its base (which is intraperitoneal) is marked by the convergence of teniae coli. At this point
is located the vermiform appendix. The colon continues as the ascending colon, the posterior aspect
of which is retroperitoneal. The line of demarcation between the intra and retro peritoneal right colon
is visible as a white line, in the living, and forms the line of incision for colonic resections.

The ascending colon becomes the transverse colon after passing the hepatic flexure. At this location
the colon becomes wholly intra peritoneal once again. The superior aspect of the transverse colon is
the point of attachment of the transverse colon to the greater omentum. This is an important
anatomical site since division of these attachments permits entry into the lesser sac. Separation of
the greater omentum from the transverse colon is a routine operative step in both gastric and colonic
resections.

At the left side of the abdomen the transverse colon passes to the left upper quadrant and makes an
oblique inferior turn at the splenic flexure. Following this, the posterior aspect becomes
retroperitoneal once again.

At the level of approximately L4 the descending colon becomes wholly intraperitoneal and becomes
the sigmoid colon. Whilst the sigmoid is wholly intraperitoneal there are usually attachments laterally
between the sigmoid and the lateral pelvic sidewall. These small congenital adhesions are not formal
anatomical attachments but frequently require division during surgical resections.

h
At its distal end the sigmoid passes to the midline and at the region around the sacral promontary it
becomes the upper rectum. This transition is visible macroscopically as the point where the teniae
fuse. More distally the rectum passes through the peritoneum at the region of the peritoneal

la
reflection and becomes extraperitoneal.

Arterial supply
Superior mesenteric artery and inferior mesenteric artery: linked by the marginal artery.
Sa
Ascending colon: ileocolic and right colic arteries
Transverse colon: middle colic artery
Descending and sigmoid colon: inferior mesenteric artery

Venous drainage
From regional veins (that accompany arteries) to superior and inferior mesenteric vein

Lymphatic drainage
C
Initially along nodal chains that accompany supplying arteries, then para-aortic nodes.

Embryology
Midgut- Second part of duodenum to 2/3 transverse colon
Hindgut- Distal 1/3 transverse colon to anus
R

Peritoneal location
The right and left colon are part intraperitoneal and part extraperitoneal. The sigmoid and transverse
colon are generally wholly intraperitoneal. This has implications for the sequelae of perforations,
M

which will tend to result in generalised peritonitis in the wholly intra peritoneal segments.

Colonic relations

Region of colon Relation

Caecum/ right colon Right ureter, gonadal vessels


Region of colon Relation

Hepatic flexure Gallbladder (medially)

Splenic flexure Spleen and tail of pancreas

Distal sigmoid/ upper rectum Left ureter

h
Rectum Ureters, autonomic nerves, seminal vesicles, prostate, urethra (distally)

la
Sa
C
R
M
Question 36 of 235

A 53 year old man is undergoing a distal pancreatectomy for trauma. Which of the following vessels
is responsible for the arterial supply to the tail of the pancreas?

Splenic artery

Pancreaticoduodenal artery

h
Gastric artery

la
Hepatic artery

Superior mesenteric artery


Sa
Pancreatic head is supplied by the pancreaticoduodenal artery
Pancreatic tail is supplied by branches of the splenic artery

There is an arterial watershed in the supply between the head and tail of the pancreas. The head is
supplied by the pancreaticoduodenal artery and the tail is supplied by branches of the splenic artery.
Please rate this question:
C
Discuss and give feedback
Next question

Pancreas
R

The pancreas is a retroperitoneal organ and lies posterior to the stomach. It may be accessed
surgically by dividing the peritoneal reflection that connects the greater omentum to the transverse
colon. The pancreatic head sits in the curvature of the duodenum. Its tail lies close to the hilum of
M

the spleen, a site of potential injury during splenectomy.

Relations
Posterior to the pancreas
Pancreatic head Inferior vena cava
Common bile duct
Right and left renal veins
Superior mesenteric vein and artery
Pancreatic neck Superior mesenteric vein, portal vein

Pancreatic body- Left renal vein


Crus of diaphragm
Psoas muscle
Adrenal gland
Kidney
Aorta

h
Pancreatic tail Left kidney

la
Anterior to the pancreas
Pancreatic head 1st part of the duodenum
Pylorus
Gastroduodenal artery
Sa SMA and SMV(uncinate process)

Pancreatic body Stomach


Duodenojejunal flexure
C
Pancreatic tail Splenic hilum

Superior to the pancreas


R

Coeliac trunk and its branches common hepatic artery and splenic artery

Grooves of the head of the pancreas


2nd and 3rd part of the duodenum
M

Arterial supply

• Head: pancreaticoduodenal artery


• Rest: splenic artery

Venous drainage

• Head: superior mesenteric vein


• Body and tail: splenic vein
Ampulla of Vater

• Merge of pancreatic duct and common bile duct


• Is an important landmark, halfway along the second part of the duodenum, that marks the
anatomical transition from foregut to midgut (also the site of transition between regions
supplied by coeliac trunk and SMA).

h
la
Sa
C
Image sourced from Wikipedia
R
M
Question 37 of 235

Surgical occlusion of which of these structures, will result in the greatest reduction in hepatic blood
flow?

Portal vein

h
Common hepatic artery

la
Right hepatic artery

Coeliac axis Sa
Left hepatic artery

The portal vein transports 70% of the blood supply to the liver, while the hepatic artery provides
30%. The portal vein contains the products of digestion. The arterial and venous blood is dispersed
by sinusoids to the central veins of the liver lobules; these drain into the hepatic veins and then into
C
the IVC. The caudate lobe drains directly into the IVC rather than into other hepatic veins.
Please rate this question:

Discuss and give feedback


R

Next question

Liver
M

Structure of the liver


Right lobe • Supplied by right hepatic artery
• Contains Couinaud segments V to VIII (-/+Sg I)

Left lobe • Supplied by the left hepatic artery


• Contains Couinaud segments II to IV (+/- Sg1)

Quadrate lobe • Part of the right lobe anatomically, functionally is part of the left
• Couinaud segment IV
• Porta hepatis lies behind
• On the right lies the gallbladder fossa
• On the left lies the fossa for the umbilical vein

Caudate lobe • Supplied by both right and left hepatic arteries


• Couinaud segment I
• Lies behind the plane of the porta hepatis
• Anterior and lateral to the inferior vena cava
• Bile from the caudate lobe drains into both right and left hepatic ducts

h
Detailed knowledge of Couinaud segments is not required for MRCS

la
• Between the liver lobules are portal canals which contain the portal triad: Hepatic Artery,
Portal Vein, tributary of Bile Duct.

Relations of the liver


Anterior
Sa Postero inferiorly

Diaphragm Oesophagus
C
Xiphoid process Stomach
R

Duodenum

Hepatic flexure of colon


M

Right kidney

Gallbladder

Inferior vena cava

Porta hepatis
Location Postero inferior surface, it joins nearly at right angles with the left sagittal fossa, and
separates the caudate lobe behind from the quadrate lobe in front

Transmits • Common hepatic duct


• Hepatic artery
• Portal vein
• Sympathetic and parasympathetic nerve fibres
• Lymphatic drainage of the liver (and nodes)

h
Ligaments
Falciform ligament • 2 layer fold peritoneum from the umbilicus to anterior liver surface

la
Contains ligamentum teres (remnant umbilical vein)
• On superior liver surface it splits into the coronary and left
triangular ligaments

Ligamentum teres

Ligamentum
Sa
Joins the left branch of the portal vein in the porta hepatis

Remnant of ductus venosus


venosum

Arterial supply
C
• Hepatic artery

Venous
R

• Hepatic veins
• Portal vein
M

Nervous supply

• Sympathetic and parasympathetic trunks of coeliac plexus


Question 38 of 235

A 23 year old man presents with appendicitis. A decision is made to perform an appendicectomy.
The operation commences with a 5cm incision centered on McBurneys point. Which of the following
structures will be encountered first during the dissection?

h
External oblique aponeurosis

Internal oblique muscle

la
Transversalis fascia
Sa
Rectus sheath

Peritoneum
C
The external oblique will be encountered first in this location. The rectus sheath lies more medially.
The external oblique muscle is the most superficial of the abdominal wall muscles. It originates from
the 5th to 12th ribs and passes inferomedially to insert into the linea alba, pubic tubercle and anterior
half of the iliac crest. It is innervated by the thoracoabdominal nerves (T7-T11) and sub costal
nerves.
R

Please rate this question:

Discuss and give feedback


M

Next question

Abdominal wall

The 2 main muscles of the abdominal wall are the rectus abdominis (anterior) and the quadratus
lumborum (posterior).
The remaining abdominal wall consists of 3 muscular layers. Each muscle passes from the lateral
aspect of the quadratus lumborum posteriorly to the lateral margin of the rectus sheath anteriorly.
Each layer is muscular posterolaterally and aponeurotic anteriorly.
Image sourced from Wikipedia

Muscles of abdominal wall


External • Lies most superficially

h
oblique • Originates from 5th to 12th ribs
• Inserts into the anterior half of the outer aspect of the iliac crest, linea
alba and pubic tubercle
• More medially and superiorly to the arcuate line, the aponeurotic layer

la
overlaps the rectus abdominis muscle
• The lower border forms the inguinal ligament
• The triangular expansion of the medial end of the inguinal ligament is
the lacunar ligament.

Internal •
Sa
Arises from the thoracolumbar fascia, the anterior 2/3 of the iliac crest
oblique and the lateral 2/3 of the inguinal ligament
• The muscle sweeps upwards to insert into the cartilages of the lower 3
ribs
• The lower fibres form an aponeurosis that runs from the tenth costal
cartilage to the body of the pubis
C
• At its lowermost aspect it joins the fibres of the aponeurosis of
transversus abdominis to form the conjoint tendon.
R

Transversus • Innermost muscle


abdominis • Arises from the inner aspect of the costal cartilages of the lower 6 ribs ,
from the anterior 2/3 of the iliac crest and lateral 1/3 of the inguinal
ligament
M

• Its fibres run horizontally around the abdominal wall ending in an


aponeurosis. The upper part runs posterior to the rectus abdominis.
Lower down the fibres run anteriorly only.
• The rectus abdominis lies medially; running from the pubic crest and
symphysis to insert into the xiphoid process and 5th, 6th and 7th costal
cartilages. The muscles lies in a aponeurosis as described above.
• Nerve supply: anterior primary rami of T7-12

Surgical notes
During abdominal surgery it is usually necessary to divide either the muscles or their aponeuroses.
During a midline laparotomy it is desirable to divide the aponeurosis. This will leave the rectus
sheath intact above the arcuate line and the muscles intact below it. Straying off the midline will
often lead to damage to the rectus muscles, particularly below the arcuate line where they may often
be in close proximity to each other.

h
la
Sa
C
R
M
Question 39 of 235

A 23 year old man is undergoing an inguinal hernia repair. The surgeons mobilise the spermatic cord
and place it in a hernia ring. A small slender nerve is identified superior to the cord. Which nerve is it
most likely to be?

Iliohypogastric nerve

h
Pudendal nerve

Femoral branch of the genitofemoral nerve

la
Ilioinguinal nerve

Obturator nerve
Sa
The ilioinguinal nerve passes through the inguinal canal and is the nerve most commonly identified
during hernia surgery. The genitofemoral nerve splits into two branches, the genital branch passes
through the inguinal canal within the cord structures. The femoral branch of the genitofemoral nerve
enters the thigh posterior to the inguinal ligament, lateral to the femoral artery. The iliohypogastric
C
nerve pierces the external oblique aponeurosis above the superficial inguinal ring.
Please rate this question:

Discuss and give feedback


R

Next question

Ilioinguinal nerve
M

Arises from the first lumbar ventral ramus with the iliohypogastric nerve. It passes inferolaterally
through the substance of psoas major and over the anterior surface of quadratus lumborum. It
pierces the internal oblique muscle and passes deep to the aponeurosis of the external oblique
muscle. It enters the inguinal canal and then passes through the superficial inguinal ring to reach the
skin.

Branches

• To supply those muscles of the abdominal wall through which it passes.


• Skin and fascia over the pubic symphysis, superomedial part of the femoral triangle, surface
of the scrotum, root and dorsum of penis or labum majus in females.
Question 41 of 235

A 45 year old man is undergoing a low anterior resection for a carcinoma of the rectum. Which of the
following fascial structures will need to be divided to mobilise the mesorectum from the sacrum and
coccyx?

Denonvilliers fascia

Colles fascia

h
Sibsons fascia

la
Waldeyers fascia

None of the above


Sa
Fascial layers surrounding the rectum:

• Anteriorly lies the fascia of Denonvilliers


• Posteriorly lies Waldeyers fascia
C
Waldeyers fascia separates the mesorectum from the sacrum and will need to be divided.
Please rate this question:
R

Discuss and give feedback


Next question

Rectum
M

The rectum is approximately 12 cm long. It is a capacitance organ. It has both intra and
extraperitoneal components. The transition between the sigmoid colon is marked by the
disappearance of the tenia coli.The extra peritoneal rectum is surrounded by mesorectal fat that also
contains lymph nodes. This mesorectal fatty layer is removed surgically during rectal cancer surgery
(Total Mesorectal Excision). The fascial layers that surround the rectum are important clinical
landmarks, anteriorly lies the fascia of Denonvilliers. Posteriorly lies Waldeyers fascia.

Extra peritoneal rectum

• Posterior upper third


• Posterior and lateral middle third
• Whole lower third

Relations
Anteriorly (Males) Rectovesical pouch
Bladder
Prostate
Seminal vesicles

Anteriorly (Females) Recto-uterine pouch (Douglas)

h
Cervix
Vaginal wall

la
Posteriorly Sacrum
Coccyx
Sa Middle sacral artery

Laterally Levator ani


Coccygeus

Arterial supply
Superior rectal artery
C
Venous drainage
Superior rectal vein

Lymphatic drainage
R

• Mesorectal lymph nodes (superior to dentate line)


• Inguinal nodes (inferior to dentate line)
M
Question 43 of 235

A 73 year old lady is admitted with acute mesenteric ischaemia. A CT angiogram is performed and a
stenotic lesion is noted at the origin of the superior mesenteric artery. At which of the following levels
does this branch from the aorta?

L1

h
L2

la
L3

L4

L5
Sa
The SMA leaves the aorta at L1. It passes under the neck of the pancreas prior to giving its first
branch the inferior pancreatico-duodenal artery.
Please rate this question:
C
Discuss and give feedback
Next question
R

Superior mesenteric artery

• Branches off aorta at L1


• Supplies small bowel from duodenum (distal to ampulla of vater) through to mid transverse
M

colon
• Takes more oblique angle from aorta and thus more likely to recieve emboli than coeliac axis

Relations of superior mesenteric artery


Superiorly Neck of pancreas

Third part of duodenum


Postero-inferiorly
Uncinate process

Posteriorly Left renal vein


Right Superior mesenteric vein

Branches of the superior mesenteric artery

• Inferior pancreatico-duodenal artery


• Jejunal and ileal arcades
• Ileo-colic artery
• Right colic artery
• Middle colic artery

h
Overview of SMA and branches

la
Sa
C
R
M
Question 44 of 235

A 72 year old man is undergoing an open abdominal aortic aneurysm repair. The aneurysm is
located in a juxtarenal location and surgical access to the neck of aneurysm is difficult. Which of the
following structures may be divided to improve access?

Cisterna chyli

h
Transverse colon

la
Left renal vein

Superior mesenteric artery


Sa
Coeliac axis

The left renal vein will be stretched over the neck of the anuerysm in this location and is not
infrequently divided. This adds to the nephrotoxic insult of juxtarenal aortic surgery as a supra renal
C
clamp is also often applied. Deliberate division of the Cisterna Chyli will not improve access and will
result in a chyle leak. Division of the transverse colon will not help at all and would result in a high
risk of graft infection. Division of the SMA is pointless for a juxtarenal procedure.
Please rate this question:
R
M

Discuss and give feedback

Next question

Abdominal aorta

Abdominal aortic topography

Origin T12
Termination L4

Posterior relations L1-L4 Vertebral bodies

Anterior relations Lesser omentum


Liver
Left renal vein
Inferior mesenteric vein
Third part of duodenum

h
Pancreas
Parietal peritoneum
Peritoneal cavity

la
Right lateral relations Right crus of the diaphragm
Cisterna chyli
Sa
IVC (becomes posterior distally)

Left lateral relations 4th part of duodenum


Duodenal-jejunal flexure
Left sympathetic trunk
C
The abdominal aorta
R
M
h
la
Sa
Image sourced from Wikipedia
C
R
M
• End and review

Question 45 of 235

What is the longest part of the male urethra?

Membranous urethra

h
Spongy urethra

la
Prostatic urethra

Urethra within the internal urethra orifice


Sa
Urethra within the urethral crest

The spongy urethra is around 15cm long and is the longest part of the male urethra.
C
Please rate this question:

Discuss and give feedback


R

Next question

Urethral anatomy

Female urethra
M

The female urethra is shorter and more acutely angulated than the male urethra. It is an extra-
peritoneal structure and embedded in the endopelvic fascia. The neck of the bladder is subjected to
transmitted intra-abdominal pressure and therefore deficiency in this area may result in stress
urinary incontinence. Between the layers of the urogenital diaphragm the female urethra is
surrounded by the external urethral sphincter, this is innervated by the pudendal nerve. It ultimately
lies anterior to the vaginal orifice.

Male urethra
In males the urethra is much longer and is divided into four parts.
Pre-prostatic Extremely short and lies between the bladder and prostate gland.It has a stellate
urethra lumen and is between 1 and 1.5cm long.Innervated by sympathetic
noradrenergic fibres, as this region is composed of striated muscles bundles they
may contract and prevent retrograde ejaculation.

Prostatic This segment is wider than the membranous urethra and contains several
urethra openings for the transmission of semen (at the midpoint of the urethral crest).

Membranous Narrowest part of the urethra and surrounded by external sphincter. It traverses

h
urethra the perineal membrane 2.5cm postero-inferior to the symphysis pubis.

la
Penile urethra Travels through the corpus spongiosum on the underside of the penis. It is the
longest urethral segment.It is dilated at its origin as the infrabulbar fossa and
again in the gland penis as the navicular fossa. The bulbo-urethral glands open
into the spongiose section of the urethra 2.5cm below the perineal membrane.
Sa
The urothelium is transitional in nature near to the bladder and becomes squamous more distally.
C
R
M
• End and review

Question 49 of 235

Your consultant decides to perform an open inguinal hernia repair under local anaesthesia. Which of
the following dermatomal levels will require blockade?

T10

h
T12

la
T11

S1
Sa
S2

Please rate this question:


C
Discuss and give feedback
Next question
R

Dermatomes

The common dermatomal levels and cutaneous nerves responsible for them is illustrated below.
M
h
la
Sa
C
R

Image sourced from Wikipedia


M
Question 51 of 235

A patient is found to have an ischaemic left colon. Which artery arising from the aorta at around the
level of L3 is most likely to account for this situation?

Superior mesenteric artery

h
Inferior mesenteric artery

la
Superior rectal artery

Ileocolic artery Sa
Middle colic artery

Only the IMA is likely to affect the left side of the colon and originate at L3.
Please rate this question:
C
Discuss and give feedback
R

Next question

Inferior mesenteric artery


M

The IMA is the main arterial supply of the embryonic hindgut and originates approximately 3-4 cm
superior to the aortic bifurcation. From its aortic origin it passes immediately inferiorly across the
anterior aspect of the aorta to eventually lie on its left hand side. At the level of the left common iliac
artery it becomes the superior rectal artery.

Branches
The left colic artery arises from the IMA near its origin. More distally up to three sigmoid arteries will
exit the IMA to supply the sigmoid colon.
Next question
Question 52 of 235

At which level does the aorta traverse the diaphragm?

T10

T9

h
T8

la
T11

T12

Memory aid:
T8 (8 letters) = vena cava
Sa
T10 (10 letters) = oesophagus
T12 (12 letters) = aortic hiatus

Another memory aid (depending upon your learning style):


C
I ate 10 Eggs At 12' I (IVC) ate (T8) 10 (T10) eggs (eosophagus) At (aorta) 12 (T12)
Please rate this question:
R

Discuss and give feedback

Next question
M

Diaphragm apertures

Diaphragm aperture levels

Vena cava T8
Oesophagus T10
Aortic hiatus T12
Next question
Question 53 of 235

At which level is the hilum of the left kidney located?

L1

L2

h
T12

la
T11

L3
Sa
Remember L1 ('left one') is the level of the hilum of the left kidney

Please rate this question:


C
Discuss and give feedback

Next question
R

Renal anatomy

Each kidney is about 11cm long, 5cm wide and 3cm thick. They are located in a deep gutter
alongside the projecting vertebral bodies, on the anterior surface of psoas major. In most cases the
M

left kidney lies approximately 1.5cm higher than the right. The upper pole of both kidneys
approximates with the 11th rib (beware pneumothorax during nephrectomy). On the left hand side
the hilum is located at the L1 vertebral level and the right kidney at level L1-2. The lower border of
the kidneys is usually alongside L3.

The table below shows the anatomical relations of the kidneys:

Relations
Relations Right Kidney Left Kidney

Posterior Quadratus lumborum, diaphragm, psoas Quadratus lumborum, diaphragm, psoas


major, transversus abdominis major, transversus abdominis

Anterior Hepatic flexure of colon Stomach, Pancreatic tail

Superior Liver, adrenal gland Spleen, adrenal gland

h
la
Fascial covering
Each kidney and suprarenal gland is enclosed within a common layer of investing fascia, derived
from the transversalis fascia. It is divided into anterior and posterior layers (Gerotas fascia).

Renal structure
Sa
Kidneys are surrounded by an outer cortex and an inner medulla which usually contains between 6
and 10 pyramidal structures. The papilla marks the innermost apex of these. They terminate at the
renal pelvis, into the ureter.
Lying in a hollow within the kidney is the renal sinus. This contains:
1. Branches of the renal artery
2. Tributaries of the renal vein
3. Major and minor calyces's
4. Fat
C
Structures at the renal hilum
The renal vein lies most anteriorly, then renal artery (it is an end artery) and the ureter lies most
posterior.
R
M
Question 54 of 235

A 73 year old lady presents with symptoms of faecal incontinence. On examination she has weak
anal sphincter muscles. What are the main nerve root values of the nerves supplying the external
anal sphincter?

S2,3

L5, S1

h
S4,5

la
S5

S2,3,4
Sa
S2, 3, 4 Keeps the poo off the floor

The external anal sphincter is innervated by the inferior rectal branch of the pudendal nerve, this has
root values of S2, 3 and the perineal branch of S4.
Please rate this question:
C
Discuss and give feedback
Next question

Anal sphincter
R

• Internal anal sphincter composed of smooth muscle continuous with the circular muscle of
the rectum. It surrounds the upper two- thirds of the anal canal and is supplied by
sympathetic nerves.
M

• External anal sphincter is composed of striated muscle which surrounds the internal
sphincter but extends more distally.
• The nerve supply of the external anal sphincter is from the inferior rectal branch of the
pudendal nerve (S2 and S3) and the perineal branch of the S4 nerve roots.

Image showing relationship of internal and external anal sphincters


h
la
Image sourced from Wikipedia

Sa
C
R
M
Question 55 of 235

You are assisting in an open right adrenalectomy for a large adrenal adenoma. The consultant is
distracted and you helpfully pull the adrenal into the wound to improve the view. Unfortunately this is
followed by brisk bleeding. The vessel responsible for this is most likely to be:

Portal vein

h
Phrenic vein

la
Right renal vein

Superior mesenteric vein

Inferior vena cava


Sa
It drains directly via a very short vessel. If the sutures are not carefully tied then it may be avulsed off
the IVC. An injury best managed using a Satinsky clamp and a 6/0 prolene suture.
C
Please rate this question:
R

Discuss and give feedback

Next question
M

Adrenal gland anatomy

Anatomy

Location Superomedially to the upper pole of each kidney


Relationships of the right Diaphragm-Posteriorly, Kidney-Inferiorly, Vena Cava-Medially, Hepato-renal
adrenal pouch and bare area of the liver-Anteriorly

Relationships of the left Crus of the diaphragm-Postero- medially, Pancreas and splenic vessels-
adrenal Inferiorly, Lesser sac and stomach-Anteriorly

Superior adrenal arteries- from inferior phrenic artery, Middle adrenal


Arterial supply arteries - from aorta, Inferior adrenal arteries -from renal arteries

h
Venous drainage of the Via one central vein directly into the IVC
right adrenal

la
Venous drainage of the Via one central vein into the left renal vein
left adrenal
Sa
C
R
M
Question 56 of 235

A 28 year old lady requires an episiotomy for a ventouse vaginal delivery. Which of the nerves listed
below will usually be anaesthetised to allow the episiotomy?

Femoral

h
Ilioinguinal

la
Pudendal

Genitofemoral Sa
Sacral plexus

The pudendal nerve innervates the posterior vulval area and is routinely blocked in procedures such
as episiotomy.
Please rate this question:
C
R

Discuss and give feedback

Next question

Pudendal nerve
M

The pudendal nerve arises from nerve roots S2, S3 and S4 and exits the pelvis through the greater
sciatic foramen. It re-enters the perineum through the lesser sciatic foramen. It travels inferior to give
innervation to the anal sphincters and external urethral sphincter. It also provides cutaneous
innervation to the region of perineum surrounding the anus and posterior vulva.

Traction and compression of the pudendal nerve by the foetus in late pregnancy may result in late
onset pudendal neuropathy which may be part of the process involved in the development of faecal
incontinence.
Question 57 of 235

An enthusiastic surgical registrar undertakes his first solo splenectomy. The operation is far more
difficult than anticipated and the registrar leaves a tube drain to the splenic bed at the end of the
procedure. Over the following 24 hours approximately 500ml of clear fluid has entered the drain.
Biochemical testing of the fluid is most likely to reveal:

Elevated creatinine

h
Elevated triglycerides

la
Elevated glucagon

Elevated amylase Sa
None of the above

During splenectomy the tail of the pancreas may be damaged. The pancreatic duct will then drain
into the splenic bed, amylase is the most likely biochemical finding. Glucagon is not secreted into the
pancreatic duct.
C
Please rate this question:

Discuss and give feedback


R

Next question

Splenic anatomy

The spleen is the largest lymphoid organ in the body. It is an intraperitoneal organ, the peritoneal
M

attachments condense at the hilum where the vessels enter the spleen. Its blood supply is from the
splenic artery (derived from the coeliac axis) and the splenic vein (which is joined by the IMV and
unites with the SMV).

• Embryology: derived from mesenchymal tissue


• Shape: clenched fist
• Position: below 9th-12th ribs
• Weight: 75-150g
Relations

• Superiorly- diaphragm
• Anteriorly- gastric impression
• Posteriorly- kidney
• Inferiorly- colon
• Hilum: tail of pancreas and splenic vessels
• Forms apex of lesser sac (containing short gastric vessels)

h
la
Sa
C
R
M
Question 58 of 235

A 56 year old lady is referred to the colorectal clinic with symptoms of pruritus ani. On examination a
polypoidal mass is identified inferior to the dentate line. A biopsy confirms squamous cell carcinoma.
To which of the following lymph node groups will the lesion potentially metastasise?

Internal iliac

h
External iliac

Mesorectal

la
Inguinal

None of the above


Sa
Lesions distal to the dentate line drain to the inguinal nodes. Occasionally this will result in the need
for a block dissection of the groin.
Please rate this question:
C
Discuss and give feedback
Next question

Rectum
R

The rectum is approximately 12 cm long. It is a capacitance organ. It has both intra and
extraperitoneal components. The transition between the sigmoid colon is marked by the
disappearance of the tenia coli.The extra peritoneal rectum is surrounded by mesorectal fat that also
M

contains lymph nodes. This mesorectal fatty layer is removed surgically during rectal cancer surgery
(Total Mesorectal Excision). The fascial layers that surround the rectum are important clinical
landmarks, anteriorly lies the fascia of Denonvilliers. Posteriorly lies Waldeyers fascia.

Extra peritoneal rectum

• Posterior upper third


• Posterior and lateral middle third
• Whole lower third
Relations
Anteriorly (Males) Rectovesical pouch
Bladder
Prostate
Seminal vesicles

Anteriorly (Females) Recto-uterine pouch (Douglas)


Cervix
Vaginal wall

h
Posteriorly Sacrum

la
Coccyx
Middle sacral artery

Laterally Levator ani


Sa Coccygeus

Arterial supply
Superior rectal artery

Venous drainage
C
Superior rectal vein

Lymphatic drainage
R

• Mesorectal lymph nodes (superior to dentate line)


• Inguinal nodes (inferior to dentate line)
M
Question 60 of 235

A 72 year old man develops a hydrocele which is being surgically managed. As part of the
procedure the surgeons divide the tunica vaginalis. From which of the following is this structure
derived?

Peritoneum

h
External oblique aponeurosis

la
Internal oblique aponeurosis

Transversalis fascia

Rectus sheath
Sa
The tunica vaginalis is derived from peritoneum, it secretes the fluid that fills the hydrocele cavity.
Please rate this question:
C
Discuss and give feedback
Next question
R

Scrotal and testicular anatomy

Spermatic cord
Formed by the vas deferens and is covered by the following structures:
M

Layer Origin

Internal spermatic fascia Transversalis fascia

Cremasteric fascia From the fascial coverings of internal oblique


Layer Origin

External spermatic fascia External oblique aponeurosis

Contents of the cord


Vas deferens Transmits sperm and accessory gland secretions

h
Testicular artery Branch of abdominal aorta supplies testis and
epididymis

la
Artery of vas deferens Arises from inferior vesical artery

Cremasteric artery Arises from inferior epigastric artery

Pampiniform plexus
Sa Venous plexus, drains into right or left testicular vein

Sympathetic nerve fibres Lie on arteries, the parasympathetic fibres lie on the
vas
C
Genital branch of the genitofemoral Supplies cremaster
nerve
R

Lymphatic vessels Drain to lumbar and para-aortic nodes


M

Scrotum

• Composed of skin and closely attached dartos fascia.


• Arterial supply from the anterior and posterior scrotal arteries
• Lymphatic drainage to the inguinal lymph nodes
• Parietal layer of the tunica vaginalis is the innermost layer

Testes
• The testes are surrounded by the tunica vaginalis (closed peritoneal sac). The parietal layer
of the tunica vaginalis adjacent to the internal spermatic fascia.
• The testicular arteries arise from the aorta immediately inferiorly to the renal arteries.
• The pampiniform plexus drains into the testicular veins, the left drains into the left renal vein
and the right into the inferior vena cava.
• Lymphatic drainage is to the para-aortic nodes.

h
la
Sa
C
R
M
Question 61 of 235

A 43 year old lady is donating her left kidney to her sister and the surgeons are harvesting the left
kidney. Which of the following structures will lie most anteriorly at the hilum of the left kidney?

Left renal artery

Left renal vein

h
Left ureter

la
Left ovarian vein

Left ovarian artery


Sa
The renal veins lie most anteriorly, then artery and ureter lies posteriorly.
Please rate this question:

Discuss and give feedback


C
Next question

Renal arteries

• The right renal artery is longer than the left renal artery
R

• The renal vein/artery/pelvis enter the kidney at the hilum

Relations
M

Right Anterior- IVC, right renal vein, the head of the pancreas, and the descending part of the
duodenum

Left Anterior- left renal vein, the tail of the pancreas

Branches
• The renal arteries are direct branches off the aorta (upper border of L2- right side and L1 -
left side)
• In 30% there may be accessory arteries (mainly left side). Instead of entering the kidney at
the hilum, they usually pierce the upper or lower part of the organ.
• Before reaching the hilum of the kidney, each artery divides into four or five segmental
branches (renal vein anterior and ureter posterior); which then divide within the sinus into
lobar arteries supplying each pyramid and cortex.
• Each vessel gives off some small inferior suprarenal branches to the suprarenal gland, the
ureter, and the surrounding cellular tissue and muscles.

Next question

h
la
Sa
C
R
M
Question 63 of 235

A 56 year old lady is due to undergo a left hemicolectomy for carcinoma of the splenic flexure. The
surgeons decide to perform a high ligation of the inferior mesenteric vein. Into which of the following
does this structure usually drain?

Portal vein

h
Inferior vena cava

la
Left renal vein

Left iliac vein


Sa
Splenic vein

Beware of ureteric injury in colonic surgery.


C
The inferior mesenteric vein drains into the splenic vein, this point of union lies close to the
duodenum and this surgical maneouvre is a recognised cause of ileus.
Please rate this question:
R

Discuss and give feedback


Next question

Left colon
M

Position

• As the left colon passes inferiorly its posterior aspect becomes extraperitoneal, and the
ureter and gonadal vessels are close posterior relations that may become involved in
disease processes
• At a level of L3-4 (variable) the left colon becomes the sigmoid colon and wholly
intraperitoneal once again
• The sigmoid colon is a highly mobile structure and may even lie on the right side of the
abdomen
• It passes towards the midline, the taenia blend and this marks the transition between sigmoid
colon and upper rectum

Blood supply

• Inferior mesenteric artery


• However, the marginal artery (from the right colon) contributes, this contribution becomes
clinically significant when the IMA is divided surgically (e.g. During AAA repair)

Next question

h
la
Sa
C
R
M
Question 64 of 235

A man undergoes a high anterior resection for carcinoma of the upper rectum. Which of the following
vessels will require ligation?

Superior mesenteric artery

h
Inferior mesenteric artery

la
Coeliac axis

Perineal artery
Sa
Middle colic artery

The IMA is usually divided during anterior resection. Not only is this borne out of oncological
C
necessity but it also permits sufficient colonic mobilisation for anastomosis.
Please rate this question:
R

Discuss and give feedback

Next question
M

Colon anatomy

The colon commences with the caecum. This represents the most dilated segment of the human
colon and its base (which is intraperitoneal) is marked by the convergence of teniae coli. At this point
is located the vermiform appendix. The colon continues as the ascending colon, the posterior aspect
of which is retroperitoneal. The line of demarcation between the intra and retro peritoneal right colon
is visible as a white line, in the living, and forms the line of incision for colonic resections.

The ascending colon becomes the transverse colon after passing the hepatic flexure. At this location
the colon becomes wholly intra peritoneal once again. The superior aspect of the transverse colon is
the point of attachment of the transverse colon to the greater omentum. This is an important
anatomical site since division of these attachments permits entry into the lesser sac. Separation of
the greater omentum from the transverse colon is a routine operative step in both gastric and colonic
resections.

At the left side of the abdomen the transverse colon passes to the left upper quadrant and makes an
oblique inferior turn at the splenic flexure. Following this, the posterior aspect becomes
retroperitoneal once again.

At the level of approximately L4 the descending colon becomes wholly intraperitoneal and becomes
the sigmoid colon. Whilst the sigmoid is wholly intraperitoneal there are usually attachments laterally
between the sigmoid and the lateral pelvic sidewall. These small congenital adhesions are not formal
anatomical attachments but frequently require division during surgical resections.

h
At its distal end the sigmoid passes to the midline and at the region around the sacral promontary it
becomes the upper rectum. This transition is visible macroscopically as the point where the teniae
fuse. More distally the rectum passes through the peritoneum at the region of the peritoneal

la
reflection and becomes extraperitoneal.

Arterial supply
Superior mesenteric artery and inferior mesenteric artery: linked by the marginal artery.
Sa
Ascending colon: ileocolic and right colic arteries
Transverse colon: middle colic artery
Descending and sigmoid colon: inferior mesenteric artery

Venous drainage
From regional veins (that accompany arteries) to superior and inferior mesenteric vein

Lymphatic drainage
C
Initially along nodal chains that accompany supplying arteries, then para-aortic nodes.

Embryology
Midgut- Second part of duodenum to 2/3 transverse colon
Hindgut- Distal 1/3 transverse colon to anus
R

Peritoneal location
The right and left colon are part intraperitoneal and part extraperitoneal. The sigmoid and transverse
colon are generally wholly intraperitoneal. This has implications for the sequelae of perforations,
M

which will tend to result in generalised peritonitis in the wholly intra peritoneal segments.

Colonic relations

Region of colon Relation

Caecum/ right colon Right ureter, gonadal vessels


Region of colon Relation

Hepatic flexure Gallbladder (medially)

Splenic flexure Spleen and tail of pancreas

Distal sigmoid/ upper rectum Left ureter

h
Rectum Ureters, autonomic nerves, seminal vesicles, prostate, urethra (distally)

la
Next question

Sa
C
R
M
Question 65 of 235

A 43 year old lady is due to undergo an axillary node clearance as part of treatment for carcinoma of
the breast. Which of the following fascial layers will be divided during the surgical approach to the
axilla?

h
Sibsons fascia

la
Pre tracheal fascia

Waldayers fascia Sa
Clavipectoral fascia

None of the above


C
The clavipectoral fascia is situated under the clavicular portion of pectoralis major. It protects both
the axillary vessels and nodes. During an axillary node clearance for breast cancer the clavipectoral
fascia is incised and this allows access to the nodal stations. The nodal stations are; level 1 nodes
inferior to pectoralis minor, level 2 lie behind it and level 3 above it. During a Patey Mastectomy
R

surgeons divide pectoralis minor to gain access to level 3 nodes. The use of sentinel node biopsy
(and stronger assistants!) have made this procedure far less common.
Please rate this question:
M

Discuss and give feedback

Next question

Axilla

Boundaries of the axilla


Medially Chest wall and Serratus anterior

Laterally Humeral head

Floor Subscapularis

Anterior aspect Lateral border of Pectoralis major

h
Fascia Clavipectoral fascia

la
Content:
Long thoracic nerve (of Derived from C5-C7 and passes behind the brachial plexus to enter the axilla.
Bell) It lies on the medial chest wall and supplies serratus anterior. Its location
Sa
puts it at risk during axillary surgery and damage will lead to winging of the
scapula.

Thoracodorsal nerve and Innervate and vascularise latissimus dorsi.


thoracodorsal trunk
C
Axillary vein Lies at the apex of the axilla, it is the continuation of the basilic vein.
Becomes the subclavian vein at the outer border of the first rib.
R

Intercostobrachial nerves Traverse the axillary lymph nodes and are often divided during axillary
surgery. They provide cutaneous sensation to the axillary skin.
M

Lymph nodes The axilla is the main site of lymphatic drainage for the breast.

Next question
Question 67 of 235

h
Where is a gomphoses type of fibrous joint typically found?

la
Teeth

Skull Sa
Manubriosternum

Ribs
C
Femur

Please rate this question:


R
M

Discuss and give feedback

Next question

Types of joint

There are three main types of joint, fibrous, cartilaginous and synovial
Type of joint Features Example

Fibrous Comprise sutures, gomphoses and syndesmoses Sutures- skull


Tough connective tissues unite the joints Gomphoses-Peg to socket e.g.
teeth
Syndesmosis-tibiofibular joint

Cartilaginous Joints united by layer of hyaline cartilage Epiphyseal growth plates


Symphysis pubis

h
Synovial Bone ends permitted free movement Hip/ knee joint

la
joints Bone ends covered by cartilage and surrounded by
fluid

Next question
Sa
C
R
M
Question 70 of 235

Which of the following statements relating to quadratus lumborum is false?

Causes flexion of the thoracic spine

Causes the rib cage to be pulled down

h
Innervated by anterior primary rami of T12 and L1-3

la
Attached to the iliac crest
Sa
Inserts into the 12th rib

Quadratus lumborum
Origin: Medial aspect of iliac crest and iliolumbar ligament
Insertion: 12th rib
Action: Pulls the rib cage inferiorly. Lateral flexion.
Nerve supply: Anterior primary rami of T12 and L1-3
C
The rectus abdominis causes flexion of the thoracic spine and therefore the statement suggesting
that quaratus lumborum does so is incorrect.
Please rate this question:
R

Discuss and give feedback


Next question
M

Abdominal wall

The 2 main muscles of the abdominal wall are the rectus abdominis (anterior) and the quadratus
lumborum (posterior).
The remaining abdominal wall consists of 3 muscular layers. Each muscle passes from the lateral
aspect of the quadratus lumborum posteriorly to the lateral margin of the rectus sheath anteriorly.
Each layer is muscular posterolaterally and aponeurotic anteriorly.
Image sourced from Wikipedia

Muscles of abdominal wall


External • Lies most superficially

h
oblique • Originates from 5th to 12th ribs
• Inserts into the anterior half of the outer aspect of the iliac crest, linea
alba and pubic tubercle
• More medially and superiorly to the arcuate line, the aponeurotic layer

la
overlaps the rectus abdominis muscle
• The lower border forms the inguinal ligament
• The triangular expansion of the medial end of the inguinal ligament is
the lacunar ligament.

Internal •
Sa
Arises from the thoracolumbar fascia, the anterior 2/3 of the iliac crest
oblique and the lateral 2/3 of the inguinal ligament
• The muscle sweeps upwards to insert into the cartilages of the lower 3
ribs
• The lower fibres form an aponeurosis that runs from the tenth costal
cartilage to the body of the pubis
C
• At its lowermost aspect it joins the fibres of the aponeurosis of
transversus abdominis to form the conjoint tendon.
R

Transversus • Innermost muscle


abdominis • Arises from the inner aspect of the costal cartilages of the lower 6 ribs ,
from the anterior 2/3 of the iliac crest and lateral 1/3 of the inguinal
ligament
M

• Its fibres run horizontally around the abdominal wall ending in an


aponeurosis. The upper part runs posterior to the rectus abdominis.
Lower down the fibres run anteriorly only.
• The rectus abdominis lies medially; running from the pubic crest and
symphysis to insert into the xiphoid process and 5th, 6th and 7th costal
cartilages. The muscles lies in a aponeurosis as described above.
• Nerve supply: anterior primary rami of T7-12

Surgical notes
During abdominal surgery it is usually necessary to divide either the muscles or their aponeuroses.
During a midline laparotomy it is desirable to divide the aponeurosis. This will leave the rectus
sheath intact above the arcuate line and the muscles intact below it. Straying off the midline will
often lead to damage to the rectus muscles, particularly below the arcuate line where they may often
be in close proximity to each other.
Next question

h
la
Sa
C
R
M
Question 71 of 235

A 67 year old man is undergoing an angiogram for gastro intestinal bleeding. The radiologist
advances the catheter into the coeliac axis. At what spinal level does this vessel typically arise from
the aorta?

h
T10

la
L3

L4 Sa
T12

None of the above


C
The coeliac axis lies at T12, it takes an almost horizontal angle off the aorta. It has three major
branches.
Please rate this question:
R

Discuss and give feedback


M

Next question

Abdominal aortic branches

Branches Level Paired Type

Inferior phrenic T12 (Upper border) Yes Parietal


Branches Level Paired Type

Coeliac T12 No Visceral

Superior mesenteric L1 No Visceral

Middle suprarenal L1 Yes Visceral

h
Renal L1-L2 Yes Visceral

la
Gonadal L2 Yes Visceral

Lumbar L1-L4 Yes Parietal


Sa
Inferior mesenteric L3 No Visceral

Median sacral L4 No Parietal


C
Common iliac L4 Yes Terminal
R

Next question
M
Question 72 of 235

During a radical gastrectomy for carcinoma of the stomach the surgeons remove the omentum.
What is the main source of its blood supply?

Ileocolic artery

h
Superior mesenteric artery

la
Gastroepiploic artery

Middle colic artery


Sa
Inferior mesenteric artery

The vessels supplying the omentum are the omental branches of the right and left gastro-epiploic
arteries. The colonic vessels are not responsible for the arterial supply to the omentum. The left
C
gastro-epiploic artery is a branch of the splenic artery and the right gastro-epiploic artery is a
terminal branch of the gastroduodenal artery.
Please rate this question:
R

Discuss and give feedback


Next question

Omentum
M

• The omentum is divided into two parts which invest the stomach. Giving rise to the greater
and lesser omentum. The greater omentum is attached to the inferolateral border of the
stomach and houses the gastro-epiploic arteries.
• It is of variable size but is less well developed in children. This is important as the omentum
confers protection against visceral perforation (e.g. Appendicitis).
• Inferiorly between the omentum and transverse colon is one potential entry point into the
lesser sac.
• Several malignant processes may involve the omentum of which ovarian cancer is the most
notable.

Next question
Question 73 of 235

A 45 year old man has a long femoral line inserted to provide CVP measurements. The catheter
passes from the common iliac vein into the inferior vena cava. At which of the following vertebral
levels will this occur?

L5

L4

h
S1

la
L3

L2
Sa
The common iliac veins fuse with the IVC at L5.
Please rate this question:
C
Discuss and give feedback
Next question

Inferior vena cava


R

Origin

• L5
M

Path

• Left and right common iliac veins merge to form the IVC.
• Passes right of midline
• Paired segmental lumbar veins drain into the IVC throughout its length
• The right gonadal vein empties directly into the cava and the left gonadal vein generally
empties into the left renal vein.
• The next major veins are the renal veins and the hepatic veins
• Pierces the central tendon of diaphragm at T8
• Right atrium
h
la
Image sourced from Wikipedia

Relations
Anteriorly
Sa
Small bowel, first and third part of duodenum, head of pancreas, liver and bile duct,
right common iliac artery, right gonadal artery

Posteriorly Right renal artery, right psoas, right sympathetic chain, coeliac ganglion
C
Levels
Level Vein
R

T8 Hepatic vein, inferior phrenic vein, pierces diaphragm


M

L1 Suprarenal veins, renal vein

L2 Gonadal vein

L1-5 Lumbar veins


L5 Common iliac vein, formation of IVC

Next question

h
la
Sa
C
R
M
Question 74 of 235

At which of the following levels does the inferior vena cava exit the abdominal cavity?

T6

T7

h
T10

la
T8

T12
Sa
Please rate this question:

Discuss and give feedback


C
Next question

Levels

Transpyloric plane
R

Level of the body of L1

• Pylorus stomach
M

• Left kidney hilum (L1- left one!)


• Fundus of the gallbladder
• Neck of pancreas
• Duodenojejunal flexure
• Superior mesenteric artery
• Portal vein
• Left and right colic flexure
• Root of the transverse mesocolon
• 2nd part of the duodenum
• Upper part of conus medullaris
• Spleen
Can be identified by asking the supine patient to sit up without using their arms. The plane is located
where the lateral border of the rectus muscle crosses the costal margin.

Anatomical planes
Subcostal plane Lowest margin of 10th costal cartilage

Intercristal plane Level of body L4 (highest point of iliac crest)

h
Intertubercular plane Level of body L5

la
Common level landmarks
Inferior mesenteric artery L3

Bifurcation of aorta into common iliac arteries L4


Sa
Formation of IVC L5 (union of common iliac veins)

Diaphragm apertures • Vena cava T8


• Oesophagus T10
C
• Aortic hiatus T12

Next question
R
M
Question 75 of 235

A 23 year old man complains of severe groin pain several weeks after a difficult inguinal hernia
repair. Which nerve is most likely to have been involved?

Genitofemoral

h
Ilioinguinal

Femoral

la
Iliohypogastric Sa
Pudendal

The ilioinguinal nerve may have been entrapped in the mesh causing a neuroma.
Please rate this question:
C
Discuss and give feedback
Next question

Ilioinguinal nerve
R

Arises from the first lumbar ventral ramus with the iliohypogastric nerve. It passes inferolaterally
through the substance of psoas major and over the anterior surface of quadratus lumborum. It
pierces the internal oblique muscle and passes deep to the aponeurosis of the external oblique
muscle. It enters the inguinal canal and then passes through the superficial inguinal ring to reach the
M

skin.

Branches

• To supply those muscles of the abdominal wall through which it passes.


• Skin and fascia over the pubic symphysis, superomedial part of the femoral triangle, surface
of the scrotum, root and dorsum of penis or labum majus in females.

Next question
Question 76 of 235

Which of the positions listed below best describes the location of the coeliac autonomic plexus?

Anterolateral to the aorta

Posterolateral to the aorta

h
Anterolateral to the sympathetic chain

la
Anteromedial to the sympathetic chain

Posterior to L1
Sa
Please rate this question:
C
Discuss and give feedback

Next question
R

Coeliac plexus

The coeliac plexus is the largest of the autonomic plexuses. It is located on a level of the last
M

thoracic and first lumbar vertebrae. It surrounds the coeliac axis and the SMA. It lies posterior to the
stomach and the lesser sac. It lies anterior to the crura of the diaphragm and the aorta. The plexus
and ganglia are joined by the greater and lesser splanchnic nerves on both sides and branches from
both the vagus and phrenic nerves.
h
la
Image sourced from Wikipedia

Sa Next question
C
R
M
Question 77 of 235

What is the nerve root value of the external urethral sphincter?

S4

h
S1, S2, S3

la
S2, S3, S4

L3, L4, L5

L5, S1, S2
Sa
The external urethral sphincter is innervated by branches of the pudendal nerve, therefore the root
values are S2, S3, S4.
C
Please rate this question:
R

Discuss and give feedback

Next question
M

Urethral anatomy

Female urethra
The female urethra is shorter and more acutely angulated than the male urethra. It is an extra-
peritoneal structure and embedded in the endopelvic fascia. The neck of the bladder is subjected to
transmitted intra-abdominal pressure and therefore deficiency in this area may result in stress
urinary incontinence. Between the layers of the urogenital diaphragm the female urethra is
surrounded by the external urethral sphincter, this is innervated by the pudendal nerve. It ultimately
lies anterior to the vaginal orifice.

Male urethra
In males the urethra is much longer and is divided into four parts.

Pre-prostatic Extremely short and lies between the bladder and prostate gland.It has a stellate lumen
urethra and is between 1 and 1.5cm long.Innervated by sympathetic noradrenergic fibres, as
this region is composed of striated muscles bundles they may contract and prevent
retrograde ejaculation.

Prostatic This segment is wider than the membranous urethra and contains several openings for
urethra the transmission of semen (at the midpoint of the urethral crest).

h
Membranous Narrowest part of the urethra and surrounded by external sphincter. It traverses the

la
urethra perineal membrane 2.5cm postero-inferior to the symphysis pubis.

Penile urethra Travels through the corpus spongiosum on the underside of the penis. It is the longest
urethral segment.It is dilated at its origin as the infrabulbar fossa and again in the gland
Sa
penis as the navicular fossa. The bulbo-urethral glands open into the spongiose section
of the urethra 2.5cm below the perineal membrane.

The urothelium is transitional in nature near to the bladder and becomes squamous more distally.
Next question
C
R
M
Question 78 of 235

A 45 year old man is stabbed in the abdomen and the inferior vena cava is injured. How many
functional valves does this vessel usually have?

h
1

la
2 Sa
4

Mnemonic for the Inferior vena cava tributaries: I Like To Rise So High:

Iliacs
Lumbar
Testicular
C
Renal
Suprarenal
Hepatic vein
R

The lack of valves in the IVC is important clinically when it is cannulated during cardiopulmonary
bypass, using separate SVC and IVC catheters, such as when the right atrium is to be opened. Note
that there is a non functional valve between the right atrium and inferior vena cava.
Please rate this question:
M

Discuss and give feedback


Next question

Inferior vena cava

Origin

• L5
Path

• Left and right common iliac veins merge to form the IVC.
• Passes right of midline
• Paired segmental lumbar veins drain into the IVC throughout its length
• The right gonadal vein empties directly into the cava and the left gonadal vein generally
empties into the left renal vein.
• The next major veins are the renal veins and the hepatic veins
• Pierces the central tendon of diaphragm at T8
• Right atrium

h
la
Sa
C
Image sourced from Wikipedia
R

Relations
Anteriorly Small bowel, first and third part of duodenum, head of pancreas, liver and bile duct,
right common iliac artery, right gonadal artery
M

Posteriorly Right renal artery, right psoas, right sympathetic chain, coeliac ganglion

Levels
Level Vein
T8 Hepatic vein, inferior phrenic vein, pierces diaphragm

L1 Suprarenal veins, renal vein

L2 Gonadal vein

L1-5 Lumbar veins

h
L5 Common iliac vein, formation of IVC

la
Next question

Sa
C
R
M
Question 79 of 235

Which of the nerves listed below is responsible for providing voluntary control of the urethral
sphincter?

Inferior hypogastric plexus

h
Superior hypogastric plexus

la
Obturator nerve

Femoral nerve
Sa
Pudendal nerve

The hypogastric plexuses provide autonomic control of the bladder. However, voluntary control of
C
the urethral sphincter is provided by the pudendal nerve.
Please rate this question:
R

Discuss and give feedback

Next question
M

Pudendal nerve

The pudendal nerve arises from nerve roots S2, S3 and S4 and exits the pelvis through the greater
sciatic foramen. It re-enters the perineum through the lesser sciatic foramen. It travels inferior to give
innervation to the anal sphincters and external urethral sphincter. It also provides cutaneous
innervation to the region of perineum surrounding the anus and posterior vulva.

Traction and compression of the pudendal nerve by the foetus in late pregnancy may result in late
onset pudendal neuropathy which may be part of the process involved in the development of faecal
incontinence.
Question 81 of 235

A 53 year old man is to undergo a thyroidectomy. Which nerve is at greatest risk?

Hypoglossal

Recurrent laryngeal

h
Ansa cervicalis

la
Accessory

Marginal mandibularSa
Recurrent laryngeal nerve injury may complicate thyroid surgery in up to 1- 2% of cases.
Please rate this question:

Discuss and give feedback


Next question
C
Recurrent laryngeal nerve

• Branch of the vagus nerve


R

Path

Right
M

• Arises anterior to the subclavian artery and ascends obliquely next to the trachea, behind the
common carotid artery
• It is either anterior or posterior to the inferior thyroid artery

Left

• Arises left to the arch of the aorta


• Winds below the aorta
• Ascends along the side of the trachea
Then both

• Pass in a groove between the trachea and oesophagus


• Enters the larynx behind the articulation between the thyroid cartilage and cricoid
• Distributed to larynx muscles

Branches to

h
• Cardiac plexus
• Mucous membrane and muscular coat of the oesophagus and trachea

la
Innervates


Sa
Intrinsic larynx muscles (excluding cricothyroid)
C
R
M

Image sourced from Wikipedia

Next question
Question 83 of 235

Which of the following is the first vessel to branch from the external carotid artery?

Superior thyroid artery

Inferior thyroid artery

h
Lingual artery

la
Facial artery

Occipital artery
Sa
Mnemonic
(Order in which they branch off)Some (sup thyroid)Attendings (Ascending Pharyngeal)Like
(Lingual)Freaking (Facial)Out (Occipital)Potential (Post auricular)Medical (Maxillary)Students (Sup
temporal)
C
The first branch of the external carotid artery is the superior thyroid artery. The inferior thyroid artery
is derived from the thyrocervical trunk. The other branches are illustrated below.
Please rate this question:
R

Discuss and give feedback


M

Next question

External carotid artery

The external carotid commences immediately lateral to the pharyngeal side wall. It ascends and lies
anterior to the internal carotid and posterior to the posterior belly of digastric and stylohyoid. More
inferiorly it is covered by sternocleidomastoid, passed by hypoglossal nerves, lingual and facial
veins.
It then pierces the fascia of the parotid gland finally dividing into its terminal branches within the
gland itself.
Surface marking of the carotid
This is an imaginary line drawn from the bifurcation of the common carotid passing behind the angle
of the jaw to a point immediately anterior to the tragus of the ear.

Branches of the external carotid artery


It has six main branches, three in front, two behind and one deep.

Three in front Superior thyroid


Lingual
Facial

h
Two behind Occipital
Posterior auricular

la
Deep Ascending pharyngeal
Sa
It terminates by dividing into the superficial temporal and maxillary arteries in the parotid gland.
C
R
M

Image sourced from Wikipedia

Next question
Question 84 of 235

A 32 year old man is undergoing a splenectomy. Division of which of the following will be necessary
during the procedure?

Left crus of diaphragm

Short gastric vessels

h
Gerotas fascia

la
Splenic flexure of colon

Marginal artery
Sa
During a splenectomy the short gastric vessels which lie within the gastrosplenic ligament will need
to be divided. The splenic flexure of the colon may need to be mobilised. However, it will almost
never need to be divided, as this is watershed area that would necessitate a formal colonic resection
in the event of division.
Please rate this question:
C
Discuss and give feedback
Next question
R

Splenic anatomy

The spleen is the largest lymphoid organ in the body. It is an intraperitoneal organ, the peritoneal
attachments condense at the hilum where the vessels enter the spleen. Its blood supply is from the
splenic artery (derived from the coeliac axis) and the splenic vein (which is joined by the IMV and
M

unites with the SMV).

• Embryology: derived from mesenchymal tissue


• Shape: clenched fist
• Position: below 9th-12th ribs
• Weight: 75-150g

Relations
• Superiorly- diaphragm
• Anteriorly- gastric impression
• Posteriorly- kidney
• Inferiorly- colon
• Hilum: tail of pancreas and splenic vessels
• Forms apex of lesser sac (containing short gastric vessels)

Next question

Display my notes on this topic

h
la
Save my notes
Sa
C
R
M
Question 86 of 235

Two teenagers are playing with an airgun when one accidentally shoots his friend in the abdomen.
He is brought to the emergency department. On examination there is a bullet entry point immediately
to the right of the rectus sheath at the level of the 1st lumbar vertebra. Which of the following
structures is most likely to be injured by the bullet?

h
Head of pancreas

la
Right ureter

Right adrenal glandSa


Fundus of the gallbladder

Gastric antrum
C
The fundus of the gallbladder lies at this level and is the most superficially located structure.
Please rate this question:
R

Discuss and give feedback


Next question

Levels
M

Transpyloric plane
Level of the body of L1

• Pylorus stomach
• Left kidney hilum (L1- left one!)
• Fundus of the gallbladder
• Neck of pancreas
• Duodenojejunal flexure
• Superior mesenteric artery
• Portal vein
• Left and right colic flexure
• Root of the transverse mesocolon
• 2nd part of the duodenum
• Upper part of conus medullaris
• Spleen

Can be identified by asking the supine patient to sit up without using their arms. The plane is located
where the lateral border of the rectus muscle crosses the costal margin.

Anatomical planes
Subcostal plane Lowest margin of 10th costal cartilage

h
Intercristal plane Level of body L4 (highest point of iliac crest)

la
Intertubercular plane Level of body L5

Common level landmarks


Inferior mesenteric artery
Sa L3

Bifurcation of aorta into common iliac arteries L4


C
Formation of IVC L5 (union of common iliac veins)

Diaphragm apertures • Vena cava T8


R

• Oesophagus T10
• Aortic hiatus T12

Next question
M

Display my notes on this topic


Question 87 of 235

Which of the following anatomical planes separates the prostate from the rectum?

Sibsons fascia

Denonvilliers fascia

h
Levator ani muscle

la
Waldeyers fascia

None of the above Sa


The Denonvilliers fascia separates the rectum from the prostate. Waldeyers fascia separates the
rectum from the sacrum
Please rate this question:

Discuss and give feedback


C
Next question

Prostate gland

The prostate gland is approximately the shape and size of a walnut and is located inferior to the
R

bladder. It is separated from the rectum by Denonvilliers fascia and its blood supply is derived from
the internal iliac vessels (via inferior vesical artery). The internal sphincter lies at the apex of the
gland and may be damaged during prostatic surgery, affected individuals may complain of
retrograde ejaculation.
M

Summary of prostate gland


Arterial supply Inferior vesical artery (from internal iliac)

Venous drainage Prostatic venous plexus (to paravertebral veins)

Lymphatic Internal iliac nodes


drainage
Innervation Inferior hypogastric plexus

Dimensions • Transverse diameter (4cm)


• AP diameter (2cm)
• Height (3cm)

Lobes • Posterior lobe: posterior to urethra


• Median lobe: posterior to urethra, in between ejaculatory ducts
• Lateral lobes x 2

h
• Isthmus

la
Zones • Peripheral zone: subcapsular portion of posterior prostate. Most
prostate cancers are here
• Central zone
• Transition zone
• Stroma
Sa
Relations
Pubic symphysis
Anterior Prostatic venous plexus
C
Posterior Denonvilliers fascia
Rectum
Ejaculatory ducts
R

Lateral Venous plexus (lies on prostate)


Levator ani (immediately below the puboprostatic ligaments)
M
h
la
Image sourced from Wikipedia

Next question
Sa
Display my notes on this topic
C
Save my notes
R
M
Question 88 of 235

A 56 year old lady is undergoing an adrenalectomy for Conns syndrome. During the operation the
surgeon damages the middle adrenal artery and haemorrhage ensues. From which of the following
structures does this vessel originate?

h
Aorta

la
Renal artery

Splenic artery Sa
Coeliac axis

Superior mesenteric artery


C
The middle adrenal artery is usually a branch of the aorta, the lower adrenal artery typically arises
from the renal vessels.
Please rate this question:
R

Discuss and give feedback


M

Next question

Adrenal gland anatomy

Anatomy

Location Superomedially to the upper pole of each kidney


Relationships of the right Diaphragm-Posteriorly, Kidney-Inferiorly, Vena Cava-Medially, Hepato-renal
adrenal pouch and bare area of the liver-Anteriorly

Relationships of the left Crus of the diaphragm-Postero- medially, Pancreas and splenic vessels-
adrenal Inferiorly, Lesser sac and stomach-Anteriorly

Superior adrenal arteries- from inferior phrenic artery, Middle adrenal


Arterial supply arteries - from aorta, Inferior adrenal arteries -from renal arteries

h
Venous drainage of the Via one central vein directly into the IVC
right adrenal

la
Venous drainage of the Via one central vein into the left renal vein
left adrenal
Sa Next question

Display my notes on this topic


C
R

Save my notes
M
Question 90 of 235

A 24 year old man falls and lands astride a manhole cover. He suffers from an injury to the anterior
bulbar urethra. Where will the extravasated urine tend to collect?

h
Lesser pelvis

la
Connective tissue of the scrotum

Deep perineal space


Sa
Ischiorectal fossa

Posterior abdominal wall


C
This portion of the urethra is contained between the perineal membrane and the membranous layer
of the superficial fascia. As these are densely adherent to the ischiopubic rami, extravasated urine
cannot pass posteriorly because the 2 layers are continuous around the superficial transverse
R

perineal muscles.
Please rate this question:

Discuss and give feedback


M

Next question

Lower genitourinary tract trauma

• Most bladder injuries occur due to blunt trauma


• 85% associated with pelvic fractures
• Easily overlooked during assessment in trauma
• Up to 10% of male pelvic fractures are associated with urethral or bladder injuries

Types of injury
Urethral injury • Mainly in males
• Blood at the meatus (50% cases)
• There are 2 types:

i.Bulbar rupture
- most common
- straddle type injury e.g. bicycles
- triad signs: urinary retention, perineal haematoma, blood at the
meatus
ii. Membranous rupture
- can be extra or intraperitoneal

h
- commonly due to pelvic fracture
- Penile or perineal oedema/ hematoma
- PR: prostate displaced upwards (beware co-existing

la
retroperitoneal haematomas as they may make examination
difficult)

- Investigation: ascending urethrogram


- Management: suprapubic catheter (surgical placement, not
Sa
External genitalia injuries
percutaneously)

• Secondary to injuries caused by penetration, blunt trauma,


(i.e., the penis and the continence- or sexual pleasure-enhancing devices, and
scrotum) mutilation
C
Bladder injury • rupture is intra or extraperitoneal
• presents with haematuria or suprapubic pain
• history of pelvic fracture and inability to void: always
suspect bladder or urethral injury
• inability to retrieve all fluid used to irrigate the bladder
R

through a Foley catheter indicates bladder injury


• investigation- IVU or cystogram
• management: laparotomy if intraperitoneal, conservative
if extraperitoneal
M

Next question

Display my notes on this topic


Question 91 of 235

A 29 year old woman has had a Pfannenstiel incision. She has pain over the inguinal ligament which
radiates to the lower abdomen. There is tenderness when the inguinal canal is compressed. Which
of the following is most likely to have been affected?

Genitofemoral nerve

Ilioinguinal nerve

h
Lateral cutaneous nerve of the thigh

la
Iliohypogastric nerve

Saphenous nerve
Sa
Please rate this question:

Discuss and give feedback


C
Next question

Nerve lesions during surgery

A variety of different procedures carry the risk of iatrogenic nerve injury. These are important not
R

only from the patients perspective but also from a medicolegal standpoint.

The following operations and their associated nerve lesions are listed here:
M

• Posterior triangle lymph node biopsy and accessory nerve lesion.


• Lloyd Davies stirrups and common peroneal nerve.
• Thyroidectomy and laryngeal nerve.
• Anterior resection of rectum and hypogastric autonomic nerves.
• Axillary node clearance; long thoracic nerve, thoracodorsal nerve and intercostobrachial
nerve.
• Inguinal hernia surgery and ilioinguinal nerve.
• Varicose vein surgery- sural and saphenous nerves.
• Posterior approach to the hip and sciatic nerve.
• Carotid endarterectomy and hypoglossal nerve.
There are many more, with sound anatomical understanding of the commonly performed procedures
the incidence of nerve lesions can be minimised. They commonly occur when surgeons operate in
an unfamiliar tissue plane or by blind placement of haemostats (not recommended).
Next question

Display my notes on this topic

h
la
Save my notes

Sa
C
R
M
Question 92 of 235

A 73 year old man presents with symptoms of mesenteric ischaemia. As part of his diagnostic work
up a diagnostic angiogram is performed .The radiologist is attempting to cannulate the coeliac axis
from the aorta. At which of the following vertebral levels does this is usually originate?

T10

h
L2

la
L3

T8
Sa
T12

Coeliac trunk branches:


C
Left Hand Side (LHS)

Left gastric
Hepatic
Splenic
R

The coeliac axis branches off the aorta at T12.


Please rate this question:
M

Discuss and give feedback


Next question

Coeliac axis

The coeliac axis has three main branches.

• Left gastric
• Hepatic: branches-Right Gastric, Gastroduodenal, Superior Pancreaticoduodenal, Cystic
(occasionally).
• Splenic: branches- Pancreatic, Short Gastric, Left Gastroepiploic

It occasionally gives off one of the inferior phrenic arteries.

h
la
Relations
Sa
Image sourced from Wikipedia

Anteriorly Lesser omentum

Right Right coeliac ganglion and caudate process of liver


C
Left Left coeliac ganglion and gastric cardia
R

Inferiorly Upper border of pancreas and renal vein


M
Question 93 of 235

A 43 year old man is diagnosed as having a malignancy of the right adrenal gland. The decision is
made to resect this via an open anterior approach. Which of the following will be most useful during
the surgery?

h
Division of the coronary ligaments of the liver

la
Mobilisation of the colonic hepatic flexure

Division of the right renal vein


Sa
Division of the ligament of Trietz

Division of the right colic artery


C
Mobilisation of the hepatic flexure and right colon are standard steps in open adrenal surgery from
an anterior approach. Mobilisation of the liver is seldom required.
Please rate this question:
R

Discuss and give feedback


M

Next question

Adrenal gland anatomy

Anatomy

Location Superomedially to the upper pole of each kidney


Relationships of the right Diaphragm-Posteriorly, Kidney-Inferiorly, Vena Cava-Medially, Hepato-renal
adrenal pouch and bare area of the liver-Anteriorly

Relationships of the left Crus of the diaphragm-Postero- medially, Pancreas and splenic vessels-
adrenal Inferiorly, Lesser sac and stomach-Anteriorly

Superior adrenal arteries- from inferior phrenic artery, Middle adrenal


Arterial supply arteries - from aorta, Inferior adrenal arteries -from renal arteries

h
Venous drainage of the Via one central vein directly into the IVC
right adrenal

la
Venous drainage of the Via one central vein into the left renal vein
left adrenal
Sa
C
R
M
Question 94 of 235

During a liver resection a surgeon performs a pringles manoeuvre to control bleeding. Which of the
following structures will lie posterior to the epiploic foramen at this level?

Hepatic artery

Cystic duct

h
Greater omentum

la
Superior mesenteric artery

Inferior vena cava


Sa
Bleeding from liver trauma or a difficult cholecystectomy can be controlled with a vascular clamp
applied at the epiploic foramen.

The epiploic foramen has the following boundaries:


Anteriorly (in the free edge of the lesser omentum): Bile duct to the right, portal vein behind and
hepatic artery to the left.
C
PosteriorlyInferior vena cava
Inferiorly1st part of the duodenum
SuperiorlyCaudate process of the liver
Please rate this question:
R

Discuss and give feedback


Next question
M

Liver

Structure of the liver


Right lobe • Supplied by right hepatic artery
• Contains Couinaud segments V to VIII (-/+Sg I)

Left lobe • Supplied by the left hepatic artery


• Contains Couinaud segments II to IV (+/- Sg1)
Quadrate lobe • Part of the right lobe anatomically, functionally is part of the left
• Couinaud segment IV
• Porta hepatis lies behind
• On the right lies the gallbladder fossa
• On the left lies the fossa for the umbilical vein

Caudate lobe • Supplied by both right and left hepatic arteries


• Couinaud segment I
• Lies behind the plane of the porta hepatis
• Anterior and lateral to the inferior vena cava

h
• Bile from the caudate lobe drains into both right and left hepatic ducts

la
Detailed knowledge of Couinaud segments is not required for MRCS

• Between the liver lobules are portal canals which contain the portal triad: Hepatic Artery,
Portal Vein, tributary of Bile Duct.
Sa
Relations of the liver
Anterior Postero inferiorly
C
Diaphragm Oesophagus

Xiphoid process Stomach


R

Duodenum
M

Hepatic flexure of colon

Right kidney

Gallbladder
Inferior vena cava

Porta hepatis
Location Postero inferior surface, it joins nearly at right angles with the left sagittal fossa, and
separates the caudate lobe behind from the quadrate lobe in front

Transmits • Common hepatic duct


• Hepatic artery

h
• Portal vein
• Sympathetic and parasympathetic nerve fibres
• Lymphatic drainage of the liver (and nodes)

la
Ligaments
Falciform ligament • 2 layer fold peritoneum from the umbilicus to anterior liver surface
Sa•

Contains ligamentum teres (remnant umbilical vein)
On superior liver surface it splits into the coronary and left
triangular ligaments

Ligamentum teres Joins the left branch of the portal vein in the porta hepatis

Ligamentum Remnant of ductus venosus


C
venosum

Arterial supply
R

• Hepatic artery
M

Venous

• Hepatic veins
• Portal vein

Nervous supply

• Sympathetic and parasympathetic trunks of coeliac plexus


Question 96 of 235

In which of the following cranial bones does the foramen spinosum lie?

Sphenoid bone

Frontal bone

h
Temporal bone

la
Occipital bone

Parietal bone
Sa
The foramen spinosum (which transmits the middle meningeal artery and vein) lies in the sphenoid
bone.
Please rate this question:
C

Discuss and give feedback


R

Next question

Foramina of the base of the skull


M

Foramen Location Contents

Foramen ovale Sphenoid Otic ganglion


bone V3 (Mandibular nerve:3rd branch of
trigeminal)
Accessory meningeal artery
Foramen Location Contents

Lesser petrosal nerve


Emissary veins

Foramen spinosum Sphenoid Middle meningeal artery


bone Meningeal branch of the Mandibular nerve

h
Foramen rotundum Sphenoid Maxillary nerve (V2)
bone

la
Foramen lacerum/ Sphenoid Base of the medial pterygoid plate.
carotid canal bone Internal carotid artery*
Nerve and artery of the pterygoid canal

Jugular foramen
Sa
Temporal Anterior: inferior petrosal sinus
bone Intermediate: glossopharyngeal, vagus, and accessory nerves.
Posterior: sigmoid sinus (becoming the internal jugular vein) and
some meningeal branches from the occipital and ascending
pharyngeal arteries.
C
Foramen magnum Occipital Anterior and posterior spinal arteries
bone Vertebral arteries
Medulla oblongata
R

Stylomastoid Temporal Stylomastoid artery


foramen bone Facial nerve
M

Superior orbital Sphenoid Oculomotor nerve (III)


fissure bone Recurrent meningeal artery
Trochlear nerve (IV)
Lacrimal, frontal and nasociliary branches of ophthalmic nerve (V1)
Abducent nerve (VI)
Superior ophthalmic vein
*= In life the foramen lacerum is occluded by a cartilagenous plug. The ICA initially passes into the
carotid canal which ascends superomedially to enter the cranial cavity through the foramen lacerum.

Base of skull anatomical overview

h
la
Sa
Image sourced from Wikipedia
C
R
M
Question 98 of 235

An 18 year old lady with troublesome hyperhidrosis of the hands and arms is due to undergo a
sympathectomy to treat the condition. Which of the following should the surgeons divide to most
effectively treat her condition?

Sympathetic ganglia at T1, T2 and T3

h
Sympathetic ganglia at T2 and T3

Sympathetic ganglia at T1 and T2

la
Stellate ganglion
Sa
Superior cervical ganglion

To treat hyperhidrosis the sympathetic ganglia at T2 and T3 should be divided. Dividing the other
structures listed would either carry a risk of Horners syndrome or be ineffective.
Please rate this question:
C
Discuss and give feedback
Next question

Sympathetic nervous system- anatomy


R

The cell bodies of the pre-ganglionic efferent neurones lie in the lateral horn of the grey matter of the
spinal cord in the thoraco-lumbar regions.
The pre-ganglionic efferents leave the spinal cord at levels T1-L2. These pass to the sympathetic
M

chain.
Lateral branches of the sympathetic chain connect it to every spinal nerve. These post ganglionic
nerves will pass to structures that receive sympathetic innervation at the periphery.

Sympathetic chains
These lie on the vertebral column and run from the base of the skull to the coccyx.
Cervical Lie anterior to the transverse processes of the cervical vertebrae and posterior to
region the carotid sheath.
Thoracic Lie anterior to the neck of the upper ribs and and lateral sides of the lower thoracic
region vertebrae.They are covered by the parietal pleura

Lumbar Enter by passing posterior to the medial arcuate ligament. Lie anteriorly to the
region vertebrae and medial to psoas major.

Sympathetic ganglia

h
• Superior cervical ganglion lies anterior to C2 and C3.
• Middle cervical ganglion (if present) C6
• Stellate ganglion- anterior to transverse process of C7, lies posterior to the subclavian artery,

la
vertebral artery and cervical pleura.
• Thoracic ganglia are segmentally arranged.
• There are usually 4 lumbar ganglia.

Clinical importance
Sa
• Interruption of the head and neck supply of the sympathetic nerves will result in an ipsilateral
Horners syndrome.
• For treatment of hyperhidrosis the sympathetic denervation can be achieved by removing the
second and third thoracic ganglia with their rami. Removal of T1 will cause a Horners
syndrome and is therefore not performed.
• In patients with vascular disease of the lower limbs a lumbar sympathetomy may be
C
performed, either radiologically or (more rarely now) surgically. The ganglia of L2 and below
are disrupted. If L1 is removed then ejaculation may be compromised (and little additional
benefit conferred as the preganglionic fibres do not arise below L2.
R
M
Question 99 of 235

During a right hemicolectomy the caecum is mobilised. As the bowel is retracted medially a vessel is
injured, posterior to the colon. Which of the following is the most likely vessel?

Right colic artery

h
Inferior vena cava

Aorta

la
External iliac artery
Sa
Gonadal vessels

The key in this question is that its during the caecal mobilization. The gonadal vessels and ureter are
important posterior relations that are at risk during a right hemicolectomy. During latter stages of the
procedure, the ileocolic artery and vein are traced along the anterior aspect of the duodenum. At this
point it is possible to injure these, the superior mesenteric vein or the middle colic vein, injury to any
C
of these can result in torrential bleeding that is very difficult to control.
Please rate this question:
R

Discuss and give feedback


Next question

Caecum
M

Location • Proximal right colon below the ileocaecal valve


• Intraperitoneal

Posterior relations • Psoas


• Iliacus
• Femoral nerve
• Genitofemoral nerve
• Gonadal vessels
Anterior relations Greater omentum

Arterial supply Ileocolic artery

Lymphatic drainage Mesenteric nodes accompany the venous drainage

• The caecum is the most distensible part of the colon and in complete large bowel obstruction
with a competent ileocaecal valve the most likely site of eventual perforation.

h
Next question

la
Sa
C
R
M
Question 101 of 235

A 66 year old man is undergoing a left nephro-ureterectomy. The surgeons remove the ureter, which
of the following is responsible for the blood supply to the proximal ureter?

Branches of the renal artery

External iliac artery

h
Internal iliac artery

la
Direct branches from the aorta

Common iliac artery


Sa
The proximal ureter is supplied by branches from the renal artery. For the other feeding vessels -
see below.
Please rate this question:
C
Discuss and give feedback
Next question

Ureter
R

• 25-35 cm long
• Muscular tube lined by transitional epithelium
• Surrounded by thick muscular coat. Becomes 3 muscular layers as it crosses the bony pelvis
• Retroperitoneal structure overlying transverse processes L2-L5
M

• Lies anterior to bifurcation of iliac vessels


• Blood supply is segmental; renal artery, aortic branches, gonadal branches, common iliac
and internal iliac
• Lies beneath the uterine artery
Question 103 of 235

Which of the following is not a content of the porta hepatis?

Portal vein

Hepatic artery

h
Cystic duct

la
Lymph nodes

None of the above


Sa
The cystic duct lies outside the porta hepatis and is an important landmark in laparoscopic
cholecystectomy. The structures in the porta hepatis are:
C
• Portal vein
• Hepatic artery
• Common hepatic duct

These structures divide immediately after or within the porta hepatis to supply the functional left and
R

right lobes of the liver.


The porta hepatis is also surrounded by lymph nodes, that may enlarge to produce obstructive
jaundice and parasympathetic nervous fibres that travel along vessels to enter the liver.
Please rate this question:
M

Discuss and give feedback


Next question

Liver

Structure of the liver


Right lobe • Supplied by right hepatic artery
• Contains Couinaud segments V to VIII (-/+Sg I)
Left lobe • Supplied by the left hepatic artery
• Contains Couinaud segments II to IV (+/- Sg1)

Quadrate lobe • Part of the right lobe anatomically, functionally is part of the left
• Couinaud segment IV
• Porta hepatis lies behind
• On the right lies the gallbladder fossa
• On the left lies the fossa for the umbilical vein

h
Caudate lobe • Supplied by both right and left hepatic arteries
• Couinaud segment I
• Lies behind the plane of the porta hepatis

la
• Anterior and lateral to the inferior vena cava
• Bile from the caudate lobe drains into both right and left hepatic ducts

Detailed knowledge of Couinaud segments is not required for MRCS


Sa
Between the liver lobules are portal canals which contain the portal triad: Hepatic Artery,
Portal Vein, tributary of Bile Duct.

Relations of the liver


C
Anterior Postero inferiorly

Diaphragm Oesophagus
R

Xiphoid process Stomach


M

Duodenum

Hepatic flexure of colon

Right kidney
Gallbladder

Inferior vena cava

Porta hepatis
Location Postero inferior surface, it joins nearly at right angles with the left sagittal fossa, and
separates the caudate lobe behind from the quadrate lobe in front

h
Transmits • Common hepatic duct
• Hepatic artery

la
• Portal vein
• Sympathetic and parasympathetic nerve fibres
• Lymphatic drainage of the liver (and nodes)

Ligaments
Falciform ligament
Sa• 2 layer fold peritoneum from the umbilicus to anterior liver surface
• Contains ligamentum teres (remnant umbilical vein)
• On superior liver surface it splits into the coronary and left
triangular ligaments
C
Ligamentum teres Joins the left branch of the portal vein in the porta hepatis

Ligamentum Remnant of ductus venosus


venosum
R

Arterial supply
M

• Hepatic artery

Venous

• Hepatic veins
• Portal vein

Nervous supply
• Sympathetic and parasympathetic trunks of coeliac plexus

h
la
Sa
C
R
M
Question 104 of 235

A surgeon is due to perform a laparotomy for perforated duodenal ulcer. An upper midline incision is
to be performed. Which of the following structures is the incision most likely to divide?

Rectus abdominis muscle

External oblique muscle

h
Linea alba

la
Internal oblique muscle

None of the above Sa


Upper midline abdominal incisions will involve the division of the linea alba. Division of muscles will
not usually improve access in this approach and they would not be routinely encountered during this
incision.
Please rate this question:
C
Discuss and give feedback
Next question

Abdominal incisions
R

Midline incision • Commonest approach to the abdomen


• Structures divided: linea alba, transversalis fascia, extraperitoneal fat,
peritoneum (avoid falciform ligament above the umbilicus)
• Bladder can be accessed via an extraperitoneal approach through the
M

space of Retzius

Paramedian • Parallel to the midline (about 3-4cm)


incision • Structures divided/retracted: anterior rectus sheath, rectus (retracted),
posterior rectus sheath, transversalis fascia, extraperitoneal fat,
peritoneum
• Incision is closed in layers
Battle • Similar location to paramedian but rectus displaced medially (and thus
denervated)
• Now seldom used

Kocher's Incision under right subcostal margin e.g. Cholecystectomy (open)

Lanz Incision in right iliac fossa e.g. Appendicectomy

Gridiron Oblique incision centered over McBurneys point- usually appendicectomy


(less cosmetically acceptable than Lanz

h
Gable Rooftop incision

Pfannenstiel's Transverse supra pubic, primarily used to access pelvic organs

la
McEvedy's Groin incision e.g. Emergency repair strangulated femoral hernia

Rutherford Extraperitoneal approach to left or right lower quadrants. Gives excellent


Morrison
Sa
access to iliac vessels and is the approach of choice for first time renal
transplantation.
C
R
M

Image sourced from Wikipedia


Question 105 of 235

A 59 year old man is undergoing an extended right hemicolectomy for a carcinoma of the splenic
flexure of the colon. The surgeons divide the middle colic vein close to its origin. Into which of the
following structures does this vessel primarily drain?

h
Superior mesenteric vein

Portal vein

la
Inferior mesenteric vein
Sa
Inferior vena cava

Ileocolic vein
C
The middle colonic vein drains into the SMV, if avulsed during mobilisation then dramatic
haemorrhage can occur and be difficult to control.
Please rate this question:
R

Discuss and give feedback


Next question

Transverse colon
M

• The right colon undergoes a sharp turn at the level of the hepatic flexure to become the
transverse colon.
• At this point it also becomes intraperitoneal.
• It is connected to the inferior border of the pancreas by the transverse mesocolon.
• The greater omentum is attached to the superior aspect of the transverse colon from which it
can easily be separated. The mesentery contains the middle colic artery and vein. The
greater omentum remains attached to the transverse colon up to the splenic flexure. At this
point the colon undergoes another sharp turn.

Relations
Liver and gall-bladder, the greater curvature of the stomach, and the lower end of the
Superior
spleen

Inferior Small intestine

Anterior Greater omentum

From right to left with the descending portion of the duodenum, the head of the
Posterior
pancreas, convolutions of the jejunum and ileum, spleen

h
la
Sa
C
R
M
Question 106 of 235

What is the lymphatic drainage of the female urethra?

Superficial inguinal nodes

Deep inguinal nodes

h
Internal iliac nodes

la
External iliac nodes

Para-aortic nodes
Sa
The entire female urethra drains to the internal iliac nodes.
Please rate this question:
C
Discuss and give feedback

Next question
R

Urethral anatomy

Female urethra
The female urethra is shorter and more acutely angulated than the male urethra. It is an extra-
M

peritoneal structure and embedded in the endopelvic fascia. The neck of the bladder is subjected to
transmitted intra-abdominal pressure and therefore deficiency in this area may result in stress
urinary incontinence. Between the layers of the urogenital diaphragm the female urethra is
surrounded by the external urethral sphincter, this is innervated by the pudendal nerve. It ultimately
lies anterior to the vaginal orifice.

Male urethra
In males the urethra is much longer and is divided into four parts.
Pre-prostatic Extremely short and lies between the bladder and prostate gland.It has a stellate lumen
urethra and is between 1 and 1.5cm long.Innervated by sympathetic noradrenergic fibres, as
this region is composed of striated muscles bundles they may contract and prevent
retrograde ejaculation.

Prostatic This segment is wider than the membranous urethra and contains several openings for
urethra the transmission of semen (at the midpoint of the urethral crest).

Membranous Narrowest part of the urethra and surrounded by external sphincter. It traverses the

h
urethra perineal membrane 2.5cm postero-inferior to the symphysis pubis.

la
Penile urethra Travels through the corpus spongiosum on the underside of the penis. It is the longest
urethral segment.It is dilated at its origin as the infrabulbar fossa and again in the gland
penis as the navicular fossa. The bulbo-urethral glands open into the spongiose section
of the urethra 2.5cm below the perineal membrane.
Sa
The urothelium is transitional in nature near to the bladder and becomes squamous more distally.
C
R
M
Question 107 of 235

A 23 year old man is stabbed in the chest approximately 10cm below the right nipple. In the
emergency department a abdominal ultrasound scan shows a large amount of intraperitoneal blood.
Which of the following statements relating to the likely site of injury is untrue?

Part of its posterior surface is devoid of peritoneum.

h
The quadrate lobe is contained within the functional right lobe.

la
Its nerve supply is from the coeliac plexus.

The hepatic flexure of the colon lies posterio-inferiorly.


Sa
The right kidney is closely related posteriorly.

The right lobe of the liver is the most likely site of injury. Therefore the answer is B as the quadrate
lobe is functionally part of the left lobe of the liver. The liver is largely covered in peritoneum.
C
Posteriorly there is an area devoid of peritoneum (the bare area of the liver). The right lobe of the
liver has the largest bare area (and is larger than the left lobe).
Please rate this question:
R

Discuss and give feedback


Next question

Liver
M

Structure of the liver


Right lobe • Supplied by right hepatic artery
• Contains Couinaud segments V to VIII (-/+Sg I)

Left lobe • Supplied by the left hepatic artery


• Contains Couinaud segments II to IV (+/- Sg1)

Quadrate lobe • Part of the right lobe anatomically, functionally is part of the left
• Couinaud segment IV
• Porta hepatis lies behind
• On the right lies the gallbladder fossa
• On the left lies the fossa for the umbilical vein

Caudate lobe • Supplied by both right and left hepatic arteries


• Couinaud segment I
• Lies behind the plane of the porta hepatis
• Anterior and lateral to the inferior vena cava
• Bile from the caudate lobe drains into both right and left hepatic ducts

h
Detailed knowledge of Couinaud segments is not required for MRCS

la
• Between the liver lobules are portal canals which contain the portal triad: Hepatic Artery,
Portal Vein, tributary of Bile Duct.
Sa
Relations of the liver
Anterior Postero inferiorly

Diaphragm Oesophagus
C
Xiphoid process Stomach
R

Duodenum

Hepatic flexure of colon


M

Right kidney

Gallbladder

Inferior vena cava


Porta hepatis
Location Postero inferior surface, it joins nearly at right angles with the left sagittal fossa, and
separates the caudate lobe behind from the quadrate lobe in front

Transmits • Common hepatic duct


• Hepatic artery
• Portal vein
• Sympathetic and parasympathetic nerve fibres
• Lymphatic drainage of the liver (and nodes)

h
Ligaments

la
Falciform ligament • 2 layer fold peritoneum from the umbilicus to anterior liver surface
• Contains ligamentum teres (remnant umbilical vein)
• On superior liver surface it splits into the coronary and left
triangular ligaments

Ligamentum teres
Sa
Joins the left branch of the portal vein in the porta hepatis

Ligamentum Remnant of ductus venosus


venosum
C
Arterial supply

• Hepatic artery
R

Venous

• Hepatic veins
M

• Portal vein

Nervous supply

• Sympathetic and parasympathetic trunks of coeliac plexus


Question 109 of 235

Which of the following nerves passes through the greater sciatic foramen and innervates the
perineum?

h
Pudendal

la
Sciatic

Superior gluteal

Inferior gluteal
Sa
Posterior cutaneous nerve of the thigh

3 divisions of the pudendal nerve:


C
• Rectal nerve
• Perineal nerve
R

• Dorsal nerve of penis/ clitoris

All these pass through the greater sciatic foramen.


M

The pudendal nerve innervates the perineum. It passes between piriformis and coccygeus medial to
the sciatic nerve.
Please rate this question:

Discuss and give feedback


Next question

Gluteal region

Gluteal muscles
• Gluteus maximus: inserts to gluteal tuberosity of the femur and iliotibial tract
• Gluteus medius: attach to lateral greater trochanter
• Gluteus minimis: attach to anterior greater trochanter
• All extend and abduct the hip

Deep lateral hip rotators

• Piriformis
• Gemelli
• Obturator internus
• Quadratus femoris

h
Nerves

la
Superior gluteal nerve (L5, S1) • Gluteus medius
• Gluteus minimis
Sa • Tensor fascia lata

Inferior gluteal nerve Gluteus maximus


Damage to the superior gluteal nerve will result in the patient developing a Trendelenberg gait.
Affected patients are unable to abduct the thigh at the hip joint. During the stance phase, the
weakened abductor muscles allow the pelvis to tilt down on the opposite side. To compensate, the
trunk lurches to the weakened side to attempt to maintain a level pelvis throughout the gait cycle.
The pelvis sags on the opposite side of the lesioned superior gluteal nerve.
C
R
M
Question 112 of 235

A 53 year old man is undergoing a left hemicolectomy for carcinoma of the descending colon. From
which embryological structure is this region of the gastrointestinal tract derived?

Vitellino-intestinal duct

Hind gut

h
Mid gut

la
Fore gut

Woolffian duct
Sa
The left colon is embryologically part of the hind gut. Which accounts for its separate blood supply
via the IMA.
Please rate this question:
C
Discuss and give feedback
R

Next question

Colon anatomy
M

The colon commences with the caecum. This represents the most dilated segment of the human
colon and its base (which is intraperitoneal) is marked by the convergence of teniae coli. At this point
is located the vermiform appendix. The colon continues as the ascending colon, the posterior aspect
of which is retroperitoneal. The line of demarcation between the intra and retro peritoneal right colon
is visible as a white line, in the living, and forms the line of incision for colonic resections.

The ascending colon becomes the transverse colon after passing the hepatic flexure. At this location
the colon becomes wholly intra peritoneal once again. The superior aspect of the transverse colon is
the point of attachment of the transverse colon to the greater omentum. This is an important
anatomical site since division of these attachments permits entry into the lesser sac. Separation of
the greater omentum from the transverse colon is a routine operative step in both gastric and colonic
resections.
At the left side of the abdomen the transverse colon passes to the left upper quadrant and makes an
oblique inferior turn at the splenic flexure. Following this, the posterior aspect becomes
retroperitoneal once again.

At the level of approximately L4 the descending colon becomes wholly intraperitoneal and becomes
the sigmoid colon. Whilst the sigmoid is wholly intraperitoneal there are usually attachments laterally
between the sigmoid and the lateral pelvic sidewall. These small congenital adhesions are not formal
anatomical attachments but frequently require division during surgical resections.

At its distal end the sigmoid passes to the midline and at the region around the sacral promontary it
becomes the upper rectum. This transition is visible macroscopically as the point where the teniae

h
fuse. More distally the rectum passes through the peritoneum at the region of the peritoneal
reflection and becomes extraperitoneal.

Arterial supply

la
Superior mesenteric artery and inferior mesenteric artery: linked by the marginal artery.
Ascending colon: ileocolic and right colic arteries
Transverse colon: middle colic artery
Descending and sigmoid colon: inferior mesenteric artery

Venous drainage
Sa
From regional veins (that accompany arteries) to superior and inferior mesenteric vein

Lymphatic drainage
Initially along nodal chains that accompany supplying arteries, then para-aortic nodes.

Embryology
C
Midgut- Second part of duodenum to 2/3 transverse colon
Hindgut- Distal 1/3 transverse colon to anus

Peritoneal location
The right and left colon are part intraperitoneal and part extraperitoneal. The sigmoid and transverse
R

colon are generally wholly intraperitoneal. This has implications for the sequelae of perforations,
which will tend to result in generalised peritonitis in the wholly intra peritoneal segments.

Colonic relations
M

Region of colon Relation

Caecum/ right colon Right ureter, gonadal vessels

Hepatic flexure Gallbladder (medially)


Region of colon Relation

Splenic flexure Spleen and tail of pancreas

Distal sigmoid/ upper rectum Left ureter

Rectum Ureters, autonomic nerves, seminal vesicles, prostate, urethra (distally)

h
la
Sa
C
R
M
Question 113 of 235

You excitedly embark on your first laparoscopic cholecystectomy and during the operation the
anatomy of Calots triangle is more hostile than anticipated. Whilst trying to apply a haemostatic clip
you avulse the cystic artery. This is followed by brisk haemorrhage. From which source is this most
likely to originate ?

Right hepatic artery

h
Portal vein

la
Gastroduodenal artery

Liver bed Sa
Common hepatic artery

The cystic artery is a branch of the right hepatic artery. There are recognised variations in the
anatomy of the blood supply to the gallbladder. However, the commonest situation is for the cystic
artery to branch from the right hepatic artery.
C
Please rate this question:

Discuss and give feedback


R

Next question

Gallbladder

• Fibromuscular sac with capacity of 50ml


M

• Columnar epithelium

Relations of the gallbladder


Anterior Liver

Posterior • Covered by peritoneum


• Transverse colon
• 1st part of the duodenum
Laterally Right lobe of liver

Medially Quadrate lobe of liver

Arterial supply
Cystic artery (branch of Right hepatic artery)

Venous drainage
Directly to the liver

Nerve supply
Sympathetic- mid thoracic spinal cord, Parasympathetic- anterior vagal trunk

h
Common bile duct

la
Origin Confluence of cystic and common hepatic ducts

Relations at • Medially - Hepatic artery



origin
Sa Posteriorly- Portal vein

Relations distally • Duodenum - anteriorly


• Pancreas - medially and laterally
• Right renal vein - posteriorly
C
Arterial supply Branches of hepatic artery and retroduodenal branches of gastroduodenal
artery

Hepatobiliary triangle
R

Medially Common hepatic duct


M

Inferiorly Cystic duct

Superiorly Inferior edge of liver

Contents Cystic artery

Relations of the gallbladder


h
la
© Image provided by the University of Sheffield

Sa
C
R
M
Question 114 of 235

A 43 year old man suffers a pelvic fracture which is complicated by an injury to the junction of the
membranous urethra to the bulbar urethra. In which of the following directions is the extravasated
urine most likely to pass?

Posteriorly into extra peritoneal tissues

h
Laterally into the buttocks

la
Into the abdomen

Anteriorly into the connective tissues surrounding the scrotum

None of the above


Sa
The superficial perineal pouch is a compartment bounded superficially by the superficial perineal
fascia, deep by the perineal membrane (inferior fascia of the urogenital diaphragm), and laterally by
C
the ischiopubic ramus. It contains the crura of the penis or clitoris, muscles, viscera, blood vessels,
nerves, the proximal part of the spongy urethra in males, and the greater vestibular glands in
females.
When urethral rupture occurs as in this case the urine will tend to pass anteriorly because the fascial
condensations will prevent lateral and posterior passage of the urine.
R

Please rate this question:

Discuss and give feedback


M

Next question

Urogenital triangle

The urogenital triangle is formed by the:

• Ischiopubic inferior rami


• Ischial tuberosities

A fascial sheet is attached to the sides, forming the inferior fascia of the urogenital diaphragm.

It transmits the urethra in males and both the urethra and vagina in females. The membranous
urethra lies deep to this structure and is surrounded by the external urethral sphincter.

Superficial to the urogenital diaphragm lies the superficial perineal pouch. In males this contains:

• Bulb of penis
• Crura of the penis
• Superficial transverse perineal muscle
• Posterior scrotal arteries
• Posterior scrotal nerves

In females the internal pudendal artery branches to become the posterior labial arteries in the

h
superficial perineal pouch.
Next question

la
Sa
C
R
M
Question 115 of 235

During the course of a radical gastrectomy the surgeons detach the omentum and ligate the right
gastro-epiploic artery. From which vessel does it originate?

Superior mesenteric artery

h
Inferior mesenteric artery

la
Coeliac axis

Common hepatic artery


Sa
Gastroduodenal artery

The gastroduodenal artery arises at the superior part of the duodenum and descends behind it to
terminate at its lower border. It terminates by dividing into the right gastro-epiploic artery and the
superior pancreaticoduodenal artery. The right gastro-opiploic artery passes to the left and passes
C
between the layers of the greater omentum to anastomose with the left gastro-epiploic artery.
Please rate this question:
R

Discuss and give feedback

Next question
M

Gastroduodenal artery

Supplies
Pylorus, proximal part of the duodenum, and indirectly to the pancreatic head (via the anterior and
posterior superior pancreaticoduodenal arteries)

Path
The gastroduodenal artery most commonly arises from the common hepatic artery of the coeliac
trunk. It terminates by bifurcating into the right gastroepiploic artery and the superior
pancreaticoduodenal artery
Question 117 of 235

Through which of the following foramina does the genital branch of the genitofemoral nerve exit the
abdominal cavity?

Superficial inguinal ring

Sciatic notch

h
Obturator foramen

la
Femoral canal

Deep inguinal ring Sa


The genitofemoral nerve divides into two branches as it approaches the inguinal ligament. The
genital branch passes anterior to the external iliac artery through the deep inguinal ring into the
inguinal canal. It communicates with the ilioinguinal nerve in the inguinal canal (though this is seldom
of clinical significance).
Please rate this question:
C
Discuss and give feedback
Next question

Genitofemoral nerve
R

Supplies
Small area of the upper medial thigh.

Path
M

• Arises from the first and second lumbar nerves.


• Passes obliquely through psoas major, and emerges from its medial border opposite the
fibrocartilage between the third and fourth lumbar vertebrae.
• It then descends on the surface of psoas major, under cover of the peritoneum
• Divides into genital and femoral branches.
• The genital branch passes through the inguinal canal, within the spermatic cord, to supply
the skin and fascia of the scrotum. The femoral branch enters the thigh posterior to the
inguinal ligament, lateral to the femoral artery. It supplies an area of skin and fascia over the
femoral triangle.
• It may be injured during abdominal or pelvic surgery, or during inguinal hernia repairs.
Question 119 of 235

A 63 year old lady is diagnosed as having an endometrial carcinoma arising from the uterine body.
To which nodal region will the tumour initially metastasise?

Para aortic nodes

Iliac lymph nodes

h
Inguinal nodes

la
Pre sacral nodes

Mesorectal lymph nodes


Sa
Tumours of the uterine body will tend to spread to the iliac nodes initially. Tumour expansion
crossing different nodal margins this is of considerable clinical significance, if nodal clearance is
performed during a Wertheims type hysterectomy.
Please rate this question:
C
Discuss and give feedback
Next question

Lymphatic drainage of the ovaries, uterus and cervix


R

• The ovaries drain to the para-aortic lymphatics via the gonadal vessels.
• The uterine fundus has a lymphatic drainage that runs with the ovarian vessels and may thus
drain to the para-aortic nodes. Some drainage may also pass along the round ligament to the
inguinal nodes.
• The body of the uterus drains through lymphatics contained within the broad ligament to the
M

iliac lymph nodes.


• The cervix drains into three potential nodal stations; laterally through the broad ligament to
the external iliac nodes, along the lymphatics of the uterosacral fold to the presacral nodes
and posterolaterally along lymphatics lying alongside the uterine vessels to the internal iliac
nodes.

Next question
Question 120 of 235

Which of the following structures is not located in the superficial perineal space in females?

Posterior labial arteries

h
Pudendal nerve

la
Superficial transverse perineal muscle
Sa
Greater vestibular glands

None of the above

The pudendal nerve is located in the deep perineal space and then branches to innervate more
C
superficial structures.
Please rate this question:

Discuss and give feedback


R

Next question

Urogenital triangle
M

The urogenital triangle is formed by the:

• Ischiopubic inferior rami


• Ischial tuberosities

A fascial sheet is attached to the sides, forming the inferior fascia of the urogenital diaphragm.

It transmits the urethra in males and both the urethra and vagina in females. The membranous
urethra lies deep to this structure and is surrounded by the external urethral sphincter.

Superficial to the urogenital diaphragm lies the superficial perineal pouch. In males this contains:
• Bulb of penis
• Crura of the penis
• Superficial transverse perineal muscle
• Posterior scrotal arteries
• Posterior scrotal nerves

In females the internal pudendal artery branches to become the posterior labial arteries in the
superficial perineal pouch.
Next question

h
la
Sa
C
R
M
Question 121 of 235

Which of the following is not a branch of the hepatic artery?

Pancreatic artery

Cystic artery

Right gastric artery

h
Right hepatic artery

la
Gastroduodenal artery
Sa
The pancreatic artery is a branch of the splenic artery.
Please rate this question:

Discuss and give feedback


Next question
C
Coeliac axis

The coeliac axis has three main branches.


R

• Left gastric
• Hepatic: branches-Right Gastric, Gastroduodenal, Superior Pancreaticoduodenal, Cystic
(occasionally).
• Splenic: branches- Pancreatic, Short Gastric, Left Gastroepiploic
M

It occasionally gives off one of the inferior phrenic arteries.


h
Image sourced from Wikipedia

la
Relations
Anteriorly Lesser omentum
Sa
Right Right coeliac ganglion and caudate process of liver

Left Left coeliac ganglion and gastric cardia


C
Inferiorly Upper border of pancreas and renal vein

Next question
R
M
Question 122 of 235

Which of the following structures does not pass close to the piriformis muscle in the greater sciatic
foramen?

Sciatic nerve

Posterior cutaneous nerve of the thigh

h
Inferior gluteal artery

la
Obturator nerve

None of the above


Sa
The obturator nerve does not pass through the greater sciatic foramen.
Please rate this question:

Discuss and give feedback


C
Next question

Greater sciatic foramen


R

Contents
Nerves • Sciatic Nerve
• Superior and Inferior Gluteal Nerves
• Pudendal Nerve
M

• Posterior Femoral Cutaneous Nerve


• Nerve to Quadratus Femoris
• Nerve to Obturator internus

Vessels • Superior Gluteal Artery and vein


• Inferior Gluteal Artery and vein
• Internal Pudendal Artery and vein

Piriformis
The piriformis is a landmark for identifying structures passing out of the sciatic notch

• Above piriformis: Superior gluteal vessels


• Below piriformis: Inferior gluteal vessels, sciatic nerve (10% pass through it, <1% above it),
posterior cutaneous nerve of the thigh

Greater sciatic foramen boundaries


Anterolaterally Greater sciatic notch of the ilium

h
Posteromedially Sacrotuberous ligament

la
Inferior Sacrospinous ligament and the ischial spine

Superior Sa Anterior sacroiliac ligament

The greater sciatic foramen


C
R
M
h
la
Sa
Image sourced from Wikipedia
C
Structures passing between both foramina (Medial to lateral)
R

• Pudendal nerve
• Internal pudendal artery
• Nerve to obturator internus
M

Contents of the lesser sciatic foramen

• Tendon of the obturator internus


• Pudendal nerve
• Internal pudendal artery and vein
• Nerve to the obturator internus

Next question
Question 123 of 235

A 56 year old man is undergoing a right nephrectomy. The surgeons divide the renal artery. At what
level does this usually branch off the abdominal aorta?

T9

h
L2

la
L3

T10

L4
Sa
The renal arteries usually branch off the aorta on a level with L2.
Please rate this question:
C
Discuss and give feedback
Next question
R

Renal arteries

• The right renal artery is longer than the left renal artery
• The renal vein/artery/pelvis enter the kidney at the hilum
M

Relations
Right Anterior- IVC, right renal vein, the head of the pancreas, and the descending part of the
duodenum

Left Anterior- left renal vein, the tail of the pancreas


Branches

• The renal arteries are direct branches off the aorta (upper border of L2- right side and L1 -
left side)
• In 30% there may be accessory arteries (mainly left side). Instead of entering the kidney at
the hilum, they usually pierce the upper or lower part of the organ.
• Before reaching the hilum of the kidney, each artery divides into four or five segmental
branches (renal vein anterior and ureter posterior); which then divide within the sinus into
lobar arteries supplying each pyramid and cortex.
• Each vessel gives off some small inferior suprarenal branches to the suprarenal gland, the
ureter, and the surrounding cellular tissue and muscles.

h
Next question

la
Sa
C
R
M
Question 125 of 235

A 22 year old man presents with appendicitis. At operation the appendix is retrocaecal and difficult to
access. Division of which of the following anatomical structures should be undertaken?

Ileocolic artery

Mesentery of the caecum

h
Gonadal vessels

la
Lateral peritoneal attachments of the caecum

Right colic artery


Sa
The commonest appendiceal location is retrocaecal. Those struggling to find it at operation should
trace the tenia to the caecal pole where the appendix is located. If it cannot be mobilised easily then
division of the lateral caecal peritoneal attachments (as for a right hemicolectomy) will allow caecal
mobilisation and facilitate the procedure.
Please rate this question:
C
Discuss and give feedback
Next question
R

Appendix

• Location: Base of caecum.


• Up to 10cm long.
M

• Mainly lymphoid tissue (Hence mesenteric adenitis may mimic appendicitis).


• Caecal taenia coli converge at base of appendix and form a longitudinal muscle cover over
the appendix. This convergence should facilitate its identification at surgery if it is retrocaecal
and difficult to find (which it can be when people start doing appendicectomies!)
• Arterial supply: Appendicular artery (branch of the ileocolic).
• It is intra peritoneal.

McBurney's point

• 1/3 of the way along a line drawn from the Anterior Superior Iliac Spine to the Umbilicus
6 Positions:

• Retrocaecal 74%
• Pelvic 21%
• Postileal
• Subcaecal
• Paracaecal
• Preileal

h
la
Sa
C
R
M
Question 126 of 235

A 56 year old man is left impotent following an abdomino-perineal excision of the colon and rectum.
What is the most likely explanation?

Psychosexual issues related to an end colostomy

Damage to the sacral venous plexus during total mesorectal excision

h
Damage to the left ureter during sigmoid mobilisation

la
Damage to the hypogastric plexus during mobilisation of the inferior mesenteric artery

Damage to the internal iliac artery during total mesorectal excision


Sa
Autonomic nerve injury is the most common cause.
Please rate this question:

Discuss and give feedback


Next question
C
Nerve lesions during surgery

A variety of different procedures carry the risk of iatrogenic nerve injury. These are important not
only from the patients perspective but also from a medicolegal standpoint.
R

The following operations and their associated nerve lesions are listed here:

• Posterior triangle lymph node biopsy and accessory nerve lesion.


M

• Lloyd Davies stirrups and common peroneal nerve.


• Thyroidectomy and laryngeal nerve.
• Anterior resection of rectum and hypogastric autonomic nerves.
• Axillary node clearance; long thoracic nerve, thoracodorsal nerve and intercostobrachial
nerve.
• Inguinal hernia surgery and ilioinguinal nerve.
• Varicose vein surgery- sural and saphenous nerves.
• Posterior approach to the hip and sciatic nerve.
• Carotid endarterectomy and hypoglossal nerve.

There are many more, with sound anatomical understanding of the commonly performed procedures
the incidence of nerve lesions can be minimised. They commonly occur when surgeons operate in
an unfamiliar tissue plane or by blind placement of haemostats (not recommended).

h
la
Sa
C
R
M
Question 127 of 235

A 73 year old man is due to undergo a radical prostatectomy for carcinoma of the prostate gland. To
which of the following lymph nodes will the tumour drain primarily?

Para aortic

h
Internal iliac

Superficial inguinal

la
Meso rectal

None of the above


Sa
The prostate lymphatic drainage is primarily to the internal iliac nodes and also the sacral nodes.
Although internal iliac is the first site.
Please rate this question:
C
Discuss and give feedback
Next question
R

Prostate gland

The prostate gland is approximately the shape and size of a walnut and is located inferior to the
bladder. It is separated from the rectum by Denonvilliers fascia and its blood supply is derived from
the internal iliac vessels (via inferior vesical artery). The internal sphincter lies at the apex of the
M

gland and may be damaged during prostatic surgery, affected individuals may complain of
retrograde ejaculation.

Summary of prostate gland


Arterial supply Inferior vesical artery (from internal iliac)

Venous drainage Prostatic venous plexus (to paravertebral veins)


Lymphatic Internal iliac nodes
drainage

Innervation Inferior hypogastric plexus

Dimensions • Transverse diameter (4cm)


• AP diameter (2cm)
• Height (3cm)

h
Lobes • Posterior lobe: posterior to urethra
• Median lobe: posterior to urethra, in between ejaculatory ducts

la
• Lateral lobes x 2
• Isthmus

Zones • Peripheral zone: subcapsular portion of posterior prostate. Most


Sa


prostate cancers are here
Central zone
Transition zone
• Stroma

Relations
C
Pubic symphysis
Anterior Prostatic venous plexus
R

Posterior Denonvilliers fascia


Rectum
Ejaculatory ducts
M

Lateral Venous plexus (lies on prostate)


Levator ani (immediately below the puboprostatic ligaments)
h
la
Image sourced from Wikipedia

Sa
C
R
M
Question 129 of 235

A 73 year old man undergoes a sub total oesophagectomy with anastomosis of the stomach to the
cervical oesophagus. Which vessel will be primarily responsible for the arterial supply to the
oesophageal portion of the anastomosis?

Superior thyroid artery

h
Internal carotid artery

la
Direct branches from the thoracic aorta

Inferior thyroid artery


Sa
Subclavian artery

The cervical oesophagus is supplied by the inferior thyroid artery. The thoracic oesophagus
(removed in this case) is supplied by direct branches from the thoracic aorta.
Please rate this question:
C
Discuss and give feedback
Next question
R

Oesophagus

• 25cm long
• Starts at C6 vertebra, pierces diaphragm at T10 and ends at T11
M

• Squamous epithelium

Constrictions of the oesophagus


Structure Distance from incisors

Cricoid cartilage 15cm


Arch of the Aorta 22.5cm

Left principal bronchus 27cm

Diaphragmatic hiatus 40cm

Relations

h
Anteriorly • Trachea to T4
• Recurrent laryngeal nerve
• Left bronchus, Left atrium

la
• Diaphragm

Posteriorly • Thoracic duct to left at T5


• Hemiazygos to the left T8
Sa •

Descending aorta
First 2 intercostal branches of aorta

Left • Thoracic duct


• Left subclavian artery
C
Right • Azygos vein
R

Arterial, venous and lymphatic drainage of the oesophagus


Artery Vein Lymphatics Muscularis externa
M

Upper third Inferior thyroid Inferior thyroid Deep cervical Striated muscle

Mid third Aortic branches Azygos branches Mediastinal Smooth & striated muscle

Lower third Left gastric Left gastric Gastric Smooth muscle

Nerve supply
• Upper half is supplied by recurrent laryngeal nerve
• Lower half by oesophageal plexus (vagus)

Histology

• Mucosa :Non-keratinized stratified squamous epithelium


• Submucosa: glandular tissue
• Muscularis externa (muscularis): composition varies. See table
• Adventitia

h
la
Sa
C
R
M
Question 131 of 235

A 28 year old man has sustained a non salvageable testicular injury to his left testicle. The surgeon
decides to perform an orchidectomy and divides the left testicular artery. From which of the following
does this vessel originate?

Abdominal aorta

Internal iliac artery

h
Inferior epigastric artery

la
Inferior vesical artery
Sa
External iliac artery

The testicular artery is a branch of the abdominal aorta.


Please rate this question:
C
Discuss and give feedback
Next question

Scrotal and testicular anatomy


R

Spermatic cord
Formed by the vas deferens and is covered by the following structures:
Layer Origin
M

Internal spermatic fascia Transversalis fascia

Cremasteric fascia From the fascial coverings of internal oblique

External spermatic fascia External oblique aponeurosis

Contents of the cord


Vas deferens Transmits sperm and accessory gland secretions

Testicular artery Branch of abdominal aorta supplies testis and


epididymis

Artery of vas deferens Arises from inferior vesical artery

Cremasteric artery Arises from inferior epigastric artery

h
Pampiniform plexus Venous plexus, drains into right or left testicular vein

la
Sympathetic nerve fibres Lie on arteries, the parasympathetic fibres lie on the
Sa vas

Genital branch of the genitofemoral Supplies cremaster


nerve

Lymphatic vessels Drain to lumbar and para-aortic nodes


C
Scrotum
R

• Composed of skin and closely attached dartos fascia.


• Arterial supply from the anterior and posterior scrotal arteries
• Lymphatic drainage to the inguinal lymph nodes
• Parietal layer of the tunica vaginalis is the innermost layer
M

Testes

• The testes are surrounded by the tunica vaginalis (closed peritoneal sac). The parietal layer
of the tunica vaginalis adjacent to the internal spermatic fascia.
• The testicular arteries arise from the aorta immediately inferiorly to the renal arteries.
• The pampiniform plexus drains into the testicular veins, the left drains into the left renal vein
and the right into the inferior vena cava.
• Lymphatic drainage is to the para-aortic nodes.
Question 132 of 235

A 44 year old man is stabbed in the back and the left kidney is injured. A haematoma forms, which of
the following fascial structures will contain the haematoma?

h
Waldeyers fascia

la
Sibsons fascia

Bucks fascia Sa
Gerotas fascia

Denonvilliers fascia
C
Waldeyers fascia- Posterior ano-rectum
Sibsons fascia- Lung apex
Bucks fascia- Base of penis
Gerotas fascia- Surrounding kidney
R

Denonvilliers fascia- Between rectum and prostate


Please rate this question:
M

Discuss and give feedback

Next question

Renal anatomy

Each kidney is about 11cm long, 5cm wide and 3cm thick. They are located in a deep gutter
alongside the projecting vertebral bodies, on the anterior surface of psoas major. In most cases the
left kidney lies approximately 1.5cm higher than the right. The upper pole of both kidneys
approximates with the 11th rib (beware pneumothorax during nephrectomy). On the left hand side
the hilum is located at the L1 vertebral level and the right kidney at level L1-2. The lower border of
the kidneys is usually alongside L3.

The table below shows the anatomical relations of the kidneys:

Relations

Relations Right Kidney Left Kidney

Posterior Quadratus lumborum, diaphragm, psoas Quadratus lumborum, diaphragm, psoas


major, transversus abdominis major, transversus abdominis

h
Anterior Hepatic flexure of colon Stomach, Pancreatic tail

la
Superior Liver, adrenal gland Spleen, adrenal gland

Fascial covering
Sa
Each kidney and suprarenal gland is enclosed within a common layer of investing fascia, derived
from the transversalis fascia. It is divided into anterior and posterior layers (Gerotas fascia).

Renal structure
Kidneys are surrounded by an outer cortex and an inner medulla which usually contains between 6
and 10 pyramidal structures. The papilla marks the innermost apex of these. They terminate at the
C
renal pelvis, into the ureter.
Lying in a hollow within the kidney is the renal sinus. This contains:
1. Branches of the renal artery
2. Tributaries of the renal vein
R

3. Major and minor calyces's


4. Fat

Structures at the renal hilum


The renal vein lies most anteriorly, then renal artery (it is an end artery) and the ureter lies most
M

posterior.
Question 135 of 235

Which of the following structures is not directly related to the right adrenal gland?

h
Diaphragm posteriorly

la
Kidney inferiorly

Right renal vein


Sa
Inferior vena cava

Hepato-renal pouch
C
The right renal vein is very short and lies more inferiorly.
Please rate this question:
R
M

Discuss and give feedback

Next question

Adrenal gland anatomy

Anatomy

Location Superomedially to the upper pole of each kidney


Relationships of the right Diaphragm-Posteriorly, Kidney-Inferiorly, Vena Cava-Medially, Hepato-renal
adrenal pouch and bare area of the liver-Anteriorly

Relationships of the left Crus of the diaphragm-Postero- medially, Pancreas and splenic vessels-
adrenal Inferiorly, Lesser sac and stomach-Anteriorly

Superior adrenal arteries- from inferior phrenic artery, Middle adrenal


Arterial supply arteries - from aorta, Inferior adrenal arteries -from renal arteries

h
Venous drainage of the Via one central vein directly into the IVC
right adrenal

la
Venous drainage of the Via one central vein into the left renal vein
left adrenal
Sa
C
R
M
Question 136 of 235

Which of the following aortic branches leaves the aorta approximately 1cm below the coeliac axis?

Renal artery

h
Inferior mesenteric artery

la
Superior mesenteric artery

Lumbar artery

Gonadal artery
Sa
The SMA leaves the aorta approximately 1cm below the coeliac axis. This is usually a level of L1. It
is crossed anteriorly by the splenic vein and the body of the pancreas. It runs downwards and
C
forwards anterior to the uncinate process.
Please rate this question:
R

Discuss and give feedback

Next question
M

Abdominal aortic branches

Branches Level Paired Type

Inferior phrenic T12 (Upper border) Yes Parietal


Branches Level Paired Type

Coeliac T12 No Visceral

Superior mesenteric L1 No Visceral

Middle suprarenal L1 Yes Visceral

h
Renal L1-L2 Yes Visceral

la
Gonadal L2 Yes Visceral

Lumbar L1-L4 Yes Parietal


Sa
Inferior mesenteric L3 No Visceral

Median sacral L4 No Parietal


C
Common iliac L4 Yes Terminal
R
M
• End and review

Question 137 of 235

Mobilisation of the left lobe of the liver will facilitate surgical access to which of the following?

h
Abdominal oesophagus

Duodenum

la
Right colic flexure Sa
Right kidney

Pylorus of stomach

The fundus of the stomach is a posterior relation. The pylorus lies more inferolaterally. During a total
C
gastrectomy division of the ligaments holding the left lobe of the liver will facilitate access to the
proximal stomach and abdominal oesophagus. This manoeuvre is seldom beneficial during a distal
gastrectomy.
Please rate this question:
R

Discuss and give feedback


Next question
M

Liver

Structure of the liver


Right lobe • Supplied by right hepatic artery
• Contains Couinaud segments V to VIII (-/+Sg I)

Left lobe • Supplied by the left hepatic artery


• Contains Couinaud segments II to IV (+/- Sg1)
Quadrate lobe • Part of the right lobe anatomically, functionally is part of the left
• Couinaud segment IV
• Porta hepatis lies behind
• On the right lies the gallbladder fossa
• On the left lies the fossa for the umbilical vein

Caudate lobe • Supplied by both right and left hepatic arteries


• Couinaud segment I
• Lies behind the plane of the porta hepatis
• Anterior and lateral to the inferior vena cava

h
• Bile from the caudate lobe drains into both right and left hepatic ducts

la
Detailed knowledge of Couinaud segments is not required for MRCS

• Between the liver lobules are portal canals which contain the portal triad: Hepatic Artery, Portal
Vein, tributary of Bile Duct.
Sa
Relations of the liver
Anterior Postero inferiorly
C
Diaphragm Oesophagus

Xiphoid process Stomach


R

Duodenum
M

Hepatic flexure of colon

Right kidney

Gallbladder
Inferior vena cava

Porta hepatis
Location Postero inferior surface, it joins nearly at right angles with the left sagittal fossa, and
separates the caudate lobe behind from the quadrate lobe in front

Transmits • Common hepatic duct


• Hepatic artery

h
• Portal vein
• Sympathetic and parasympathetic nerve fibres
• Lymphatic drainage of the liver (and nodes)

la
Ligaments
Falciform ligament • 2 layer fold peritoneum from the umbilicus to anterior liver surface
Sa•

Contains ligamentum teres (remnant umbilical vein)
On superior liver surface it splits into the coronary and left
triangular ligaments

Ligamentum teres Joins the left branch of the portal vein in the porta hepatis

Ligamentum Remnant of ductus venosus


C
venosum

Arterial supply
R

• Hepatic artery
M

Venous

• Hepatic veins
• Portal vein

Nervous supply

• Sympathetic and parasympathetic trunks of coeliac plexus

Next question
Question 140 of 235

At what level does the aorta bifurcate into the left and right common iliac arteries?

L1

L2

L3

h
L4

la
L5

exam.
Please rate this question:
Sa
The aorta typically bifurcates at L4. This level is usually fairly constant and is often tested in the

Discuss and give feedback


Next question
C
Levels

Transpyloric plane
Level of the body of L1
R

• Pylorus stomach
• Left kidney hilum (L1- left one!)
M

• Fundus of the gallbladder


• Neck of pancreas
• Duodenojejunal flexure
• Superior mesenteric artery
• Portal vein
• Left and right colic flexure
• Root of the transverse mesocolon
• 2nd part of the duodenum
• Upper part of conus medullaris
• Spleen
Can be identified by asking the supine patient to sit up without using their arms. The plane is located
where the lateral border of the rectus muscle crosses the costal margin.

Anatomical planes
Subcostal plane Lowest margin of 10th costal cartilage

Intercristal plane Level of body L4 (highest point of iliac crest)

h
Intertubercular plane Level of body L5

la
Common level landmarks
Inferior mesenteric artery L3

Bifurcation of aorta into common iliac arteries L4


Sa
Formation of IVC L5 (union of common iliac veins)

Diaphragm apertures • Vena cava T8


• Oesophagus T10
C
• Aortic hiatus T12

Next question
R
M
Question 143 of 235

A 32 year old man presents with an inguinal hernia and undergoes an open surgical repair. The
surgeons decide to place a mesh on the posterior wall of the inguinal canal to complete the repair,
which of the following structures will lie posterior to the mesh?

Transversalis fascia

External oblique

h
Rectus abdominis

la
Obturator nerve

None of the above


Sa
Inguinal canal walls: 'MALT: 2M, 2A, 2L, 2T':

Starting from superior, moving around in order to posterior:


Superior wall (roof): 2 Muscles:Internal oblique, transversus abdominis
Anterior wall: 2 Aponeuroses: Aponeurosis of external oblique, Aponeurosis of internal oblique
C
Lower wall (floor): 2 Ligaments: Inguinal Ligament, Lacunar Ligament Posterior wall: 2Ts:
Transversalis fascia, Conjoint Tendon

This is actually quite a straightforward question. It is simply asking for the structure that forms the
posterior wall of the inguinal canal. This is composed of the transversalis fascia, the conjoint tendon
R

and more laterally the deep inguinal ring.


Please rate this question:
M

Discuss and give feedback


Next question

Inguinal canal

Location

• Above the inguinal ligament


• The inguinal canal is 4cm long
• The superficial ring is located anterior to the pubic tubercle
• The deep ring is located approximately 1.5-2cm above the half way point between the
anterior superior iliac spine and the pubic tubercle

Boundaries of the inguinal canal


Floor • External oblique aponeurosis
• Inguinal ligament
• Lacunar ligament

Roof • Internal oblique


h
Transversus abdominis

Anterior wall External oblique aponeurosis

la
Posterior wall • Transversalis fascia
• Conjoint tendon

Laterally
Sa •

Internal ring
Transversalis fascia
• Fibres of internal oblique

Medially • External ring


• Conjoint tendon
C
Contents
Males Spermatic cord and ilioinguinal As it passes through the canal the spermatic cord
R

nerve has 3 coverings:

• External spermatic fascia from external


M

oblique aponeurosis
• Cremasteric fascia
• Internal spermatic fascia

Females Round ligament of uterus and


ilioinguinal nerve

Related anatomy of the inguinal region


The boundaries of Hesselbachs triangle are commonly tested and illustrated below:
h
la
Sa
Image sourced from Wikipedia

The image below demonstrates the close relationship of the vessels to the lower limb with the
inguinal canal. A fact to be borne in mind when repairing hernial defects in this region.
C
R
M

Image sourced from Wikipedia

Next question
Question 146 of 235

At which of the following spinal levels does the oesophagus pass through the diaphragm into the
abdominal cavity?

L2

L1

h
T10

la
T5

T12
Sa
The oesophagus passes into the abdomen at T10.
Please rate this question:

Discuss and give feedback


C
Next question

Oesophagus

• 25cm long
R

• Starts at C6 vertebra, pierces diaphragm at T10 and ends at T11


• Squamous epithelium

Constrictions of the oesophagus


M

Structure Distance from incisors

Cricoid cartilage 15cm

Arch of the Aorta 22.5cm


Left principal bronchus 27cm

Diaphragmatic hiatus 40cm

Relations
Anteriorly • Trachea to T4
• Recurrent laryngeal nerve
• Left bronchus, Left atrium

h
• Diaphragm

la
Posteriorly Thoracic duct to left at T5
• Hemiazygos to the left T8
• Descending aorta
Sa • First 2 intercostal branches of aorta

Left • Thoracic duct


• Left subclavian artery

Right • Azygos vein


C
Arterial, venous and lymphatic drainage of the oesophagus
Artery Vein Lymphatics Muscularis externa
R

Upper third Inferior thyroid Inferior thyroid Deep cervical Striated muscle
M

Mid third Aortic branches Azygos branches Mediastinal Smooth & striated muscle

Lower third Left gastric Left gastric Gastric Smooth muscle

Nerve supply

• Upper half is supplied by recurrent laryngeal nerve


• Lower half by oesophageal plexus (vagus)
Histology

• Mucosa :Non-keratinized stratified squamous epithelium


• Submucosa: glandular tissue
• Muscularis externa (muscularis): composition varies. See table
• Adventitia

Next question

h
la
Sa
C
R
M
Question 147 of 235

A 22 year old man is involved in a fight outside a nightclub. He is stabbed in the back, on the left
side, approximately 3cm below the 12th rib in the mid scapular line. The structure most likely to be
injured first as a result is the:

Spleen

h
Left kidney

la
Left adrenal gland

Left ureter Sa
None of the above

The left kidney lies in this location and is the most likely structure to be injured. The Spleen lies more
superiorly, and the left adrenal and ureter are unlikely to be injured in isolation.
Please rate this question:
C
Discuss and give feedback
Next question
R

Levels

Transpyloric plane
Level of the body of L1
M

• Pylorus stomach
• Left kidney hilum (L1- left one!)
• Fundus of the gallbladder
• Neck of pancreas
• Duodenojejunal flexure
• Superior mesenteric artery
• Portal vein
• Left and right colic flexure
• Root of the transverse mesocolon
• 2nd part of the duodenum
• Upper part of conus medullaris
• Spleen

Can be identified by asking the supine patient to sit up without using their arms. The plane is located
where the lateral border of the rectus muscle crosses the costal margin.

Anatomical planes
Subcostal plane Lowest margin of 10th costal cartilage

Intercristal plane Level of body L4 (highest point of iliac crest)

h
Intertubercular plane Level of body L5

la
Common level landmarks
Inferior mesenteric artery L3
Sa
Bifurcation of aorta into common iliac arteries L4

Formation of IVC L5 (union of common iliac veins)


C
Diaphragm apertures • Vena cava T8
• Oesophagus T10
• Aortic hiatus T12
R

Next question
M
Question 148 of 235

A 23 year old man is undergoing a hernia repair and the mesh is to be sutured to the inguinal
ligament. From which of the following does the inguinal ligament arise?

Transversus abdominis fascia

h
Internal oblique

la
Rectus sheath

Rectus abdominis muscle


Sa
External oblique aponeurosis

The inguinal ligament is formed by the external oblique aponeurosis. It runs from the pubic tubercle
to the anterior superior iliac spine.
C
Please rate this question:

Discuss and give feedback


R

Next question

Abdominal wall

The 2 main muscles of the abdominal wall are the rectus abdominis (anterior) and the quadratus
M

lumborum (posterior).
The remaining abdominal wall consists of 3 muscular layers. Each muscle passes from the lateral
aspect of the quadratus lumborum posteriorly to the lateral margin of the rectus sheath anteriorly.
Each layer is muscular posterolaterally and aponeurotic anteriorly.
Image sourced from Wikipedia

Muscles of abdominal wall


External • Lies most superficially

h
oblique • Originates from 5th to 12th ribs
• Inserts into the anterior half of the outer aspect of the iliac crest, linea
alba and pubic tubercle
• More medially and superiorly to the arcuate line, the aponeurotic layer

la
overlaps the rectus abdominis muscle
• The lower border forms the inguinal ligament
• The triangular expansion of the medial end of the inguinal ligament is
the lacunar ligament.

Internal •
Sa
Arises from the thoracolumbar fascia, the anterior 2/3 of the iliac crest
oblique and the lateral 2/3 of the inguinal ligament
• The muscle sweeps upwards to insert into the cartilages of the lower 3
ribs
• The lower fibres form an aponeurosis that runs from the tenth costal
cartilage to the body of the pubis
C
• At its lowermost aspect it joins the fibres of the aponeurosis of
transversus abdominis to form the conjoint tendon.
R

Transversus • Innermost muscle


abdominis • Arises from the inner aspect of the costal cartilages of the lower 6 ribs ,
from the anterior 2/3 of the iliac crest and lateral 1/3 of the inguinal
ligament
M

• Its fibres run horizontally around the abdominal wall ending in an


aponeurosis. The upper part runs posterior to the rectus abdominis.
Lower down the fibres run anteriorly only.
• The rectus abdominis lies medially; running from the pubic crest and
symphysis to insert into the xiphoid process and 5th, 6th and 7th costal
cartilages. The muscles lies in a aponeurosis as described above.
• Nerve supply: anterior primary rami of T7-12

Surgical notes
During abdominal surgery it is usually necessary to divide either the muscles or their aponeuroses.
During a midline laparotomy it is desirable to divide the aponeurosis. This will leave the rectus
sheath intact above the arcuate line and the muscles intact below it. Straying off the midline will
often lead to damage to the rectus muscles, particularly below the arcuate line where they may often
be in close proximity to each other.
Next question

h
la
Sa
C
R
M
Question 149 of 235

A 56 year old man is undergoing a high anterior resection. Which of the following structures is at
greatest risk of injury in this procedure?

Superior mesenteric artery

h
Left ureter

la
External iliac vein

External iliac artery


Sa
Inferior vena cava

A careless surgeon may damage all of these structures. However, the structure at greatest risk and
C
most frequently encountered is the left ureter.
Please rate this question:
R

Discuss and give feedback

Next question
M

Colon anatomy

The colon commences with the caecum. This represents the most dilated segment of the human
colon and its base (which is intraperitoneal) is marked by the convergence of teniae coli. At this point
is located the vermiform appendix. The colon continues as the ascending colon, the posterior aspect
of which is retroperitoneal. The line of demarcation between the intra and retro peritoneal right colon
is visible as a white line, in the living, and forms the line of incision for colonic resections.

The ascending colon becomes the transverse colon after passing the hepatic flexure. At this location
the colon becomes wholly intra peritoneal once again. The superior aspect of the transverse colon is
the point of attachment of the transverse colon to the greater omentum. This is an important
anatomical site since division of these attachments permits entry into the lesser sac. Separation of
the greater omentum from the transverse colon is a routine operative step in both gastric and colonic
resections.

At the left side of the abdomen the transverse colon passes to the left upper quadrant and makes an
oblique inferior turn at the splenic flexure. Following this, the posterior aspect becomes
retroperitoneal once again.

At the level of approximately L4 the descending colon becomes wholly intraperitoneal and becomes
the sigmoid colon. Whilst the sigmoid is wholly intraperitoneal there are usually attachments laterally
between the sigmoid and the lateral pelvic sidewall. These small congenital adhesions are not formal
anatomical attachments but frequently require division during surgical resections.

h
At its distal end the sigmoid passes to the midline and at the region around the sacral promontary it
becomes the upper rectum. This transition is visible macroscopically as the point where the teniae
fuse. More distally the rectum passes through the peritoneum at the region of the peritoneal

la
reflection and becomes extraperitoneal.

Arterial supply
Superior mesenteric artery and inferior mesenteric artery: linked by the marginal artery.
Sa
Ascending colon: ileocolic and right colic arteries
Transverse colon: middle colic artery
Descending and sigmoid colon: inferior mesenteric artery

Venous drainage
From regional veins (that accompany arteries) to superior and inferior mesenteric vein

Lymphatic drainage
C
Initially along nodal chains that accompany supplying arteries, then para-aortic nodes.

Embryology
Midgut- Second part of duodenum to 2/3 transverse colon
Hindgut- Distal 1/3 transverse colon to anus
R

Peritoneal location
The right and left colon are part intraperitoneal and part extraperitoneal. The sigmoid and transverse
colon are generally wholly intraperitoneal. This has implications for the sequelae of perforations,
M

which will tend to result in generalised peritonitis in the wholly intra peritoneal segments.

Colonic relations

Region of colon Relation

Caecum/ right colon Right ureter, gonadal vessels


Region of colon Relation

Hepatic flexure Gallbladder (medially)

Splenic flexure Spleen and tail of pancreas

Distal sigmoid/ upper rectum Left ureter

h
Rectum Ureters, autonomic nerves, seminal vesicles, prostate, urethra (distally)

la
Next question

Sa
C
R
M
Question 150 of 235

A 42 year old lady undergoes a difficult cholecystectomy and significant bleeding is occurring. The
surgeons place a vascular clamp transversely across the anterior border of the epiploic foramen.
Which of the following structures will be occluded in this manoeuvre?

Cystic artery

h
Cystic duct

la
Left gastric artery

Portal vein Sa
None of the above

The portal vein, hepatic artery and common bile duct are occluded.
Please rate this question:
C
Discuss and give feedback
Next question

Epiploic Foramen
R

The epiploic foramen has the following boundaries:


Anteriorly (in the free edge of the lesser Bile duct to the right, portal vein behind and hepatic
M

omentum) artery to the left.

Posteriorly Inferior vena cava

Inferiorly 1st part of the duodenum

Superiorly Caudate process of the liver


During liver surgery bleeding may be controlled using a Pringles manoeuvre, this involves placing a
vascular clamp across the anterior aspect of the epiploic foramen. Thereby occluding:

• Common bile duct


• Hepatic artery
• Portal vein

h
la
Sa
C
R
M
Question 152 of 235

On inspecting the caecum, which of the following structures is most likely to be identified at the point
at which all the tenia coli converge?

Gonadal vessels

h
Appendix base

Appendix tip

la
Ileocaecal valve

Ileocolic artery
Sa
The tenia coli converge at the base of the appendix.
Please rate this question:
C
Discuss and give feedback
Next question

Caecum
R

Location • Proximal right colon below the ileocaecal valve


• Intraperitoneal
M

Posterior relations • Psoas


• Iliacus
• Femoral nerve
• Genitofemoral nerve
• Gonadal vessels

Anterior relations Greater omentum

Arterial supply Ileocolic artery


Lymphatic drainage Mesenteric nodes accompany the venous drainage

• The caecum is the most distensible part of the colon and in complete large bowel obstruction
with a competent ileocaecal valve the most likely site of eventual perforation.

Next question

h
Display my notes on this topic

la
Save my notes
Sa
C
R
M
Question 154 of 235

What is the most inferior anterior aortic branch?

Median sacral artery

Inferior mesenteric artery

h
Lumbar artery

la
Superior mesenteric artery

Gonadal artery
Sa
The IMA leaves the front of the aorta usually about 3 to 4cm superior to its bifurcation. The median
sacral is not an anterior branch.
Please rate this question:
C
Discuss and give feedback
R

Next question

Abdominal aortic branches


M

Branches Level Paired Type

Inferior phrenic T12 (Upper border) Yes Parietal

Coeliac T12 No Visceral


Branches Level Paired Type

Superior mesenteric L1 No Visceral

Middle suprarenal L1 Yes Visceral

Renal L1-L2 Yes Visceral

h
Gonadal L2 Yes Visceral

la
Lumbar L1-L4 Yes Parietal

Inferior mesenteric L3 No Visceral


Sa
Median sacral L4 No Parietal

Common iliac L4 Yes Terminal


C
R
M
Question 155 of 235

Which of the following structures lies most posteriorly at the porta hepatis?

Cystic artery

h
Common hepatic artery

la
Left hepatic artery

Portal vein Sa
Common bile duct

The portal vein is the most posterior structure at the porta hepatis.The common bile duct is a
continuation of the common hepatic duct and is formed by the union of the common hepatic duct and
the cystic duct.
Please rate this question:
C
Discuss and give feedback
Next question
R

Liver

Structure of the liver


M

Right lobe • Supplied by right hepatic artery


• Contains Couinaud segments V to VIII (-/+Sg I)

Left lobe • Supplied by the left hepatic artery


• Contains Couinaud segments II to IV (+/- Sg1)

Quadrate lobe • Part of the right lobe anatomically, functionally is part of the left
• Couinaud segment IV
• Porta hepatis lies behind
• On the right lies the gallbladder fossa
• On the left lies the fossa for the umbilical vein

Caudate lobe • Supplied by both right and left hepatic arteries


• Couinaud segment I
• Lies behind the plane of the porta hepatis
• Anterior and lateral to the inferior vena cava
• Bile from the caudate lobe drains into both right and left hepatic ducts

h
Detailed knowledge of Couinaud segments is not required for MRCS

la
• Between the liver lobules are portal canals which contain the portal triad: Hepatic Artery,
Portal Vein, tributary of Bile Duct.

Relations of the liver


Anterior
Sa Postero inferiorly

Diaphragm Oesophagus
C
Xiphoid process Stomach

Duodenum
R

Hepatic flexure of colon


M

Right kidney

Gallbladder

Inferior vena cava

Porta hepatis
Location Postero inferior surface, it joins nearly at right angles with the left sagittal fossa, and
separates the caudate lobe behind from the quadrate lobe in front

Transmits • Common hepatic duct


• Hepatic artery
• Portal vein
• Sympathetic and parasympathetic nerve fibres
• Lymphatic drainage of the liver (and nodes)

h
Ligaments
Falciform ligament • 2 layer fold peritoneum from the umbilicus to anterior liver surface

la
Contains ligamentum teres (remnant umbilical vein)
• On superior liver surface it splits into the coronary and left
triangular ligaments

Ligamentum teres

Ligamentum
Sa
Joins the left branch of the portal vein in the porta hepatis

Remnant of ductus venosus


venosum

Arterial supply
C
• Hepatic artery

Venous
R

• Hepatic veins
• Portal vein
M

Nervous supply

• Sympathetic and parasympathetic trunks of coeliac plexus

Next question
Question 156 of 235

A 76 year old man is undergoing an abdominal aortic aneurysm repair. The surgeons occlude the
aorta with two clamps, the inferior clamp being placed at the point of aortic bifurcation. Which of the
following vertebral bodies will lie posterior to the clamp at this level?

h
L1

la
T10

L4
Sa
L5

L2
C
The aorta bifurcates at L4. An important landmark that is tested frequently.
Please rate this question:
R

Discuss and give feedback


M

Next question

Abdominal aorta

Abdominal aortic topography

Origin T12
Termination L4

Posterior relations L1-L4 Vertebral bodies

Anterior relations Lesser omentum


Liver
Left renal vein
Inferior mesenteric vein
Third part of duodenum

h
Pancreas
Parietal peritoneum
Peritoneal cavity

la
Right lateral relations Right crus of the diaphragm
Cisterna chyli
Sa
IVC (becomes posterior distally)

Left lateral relations 4th part of duodenum


Duodenal-jejunal flexure
Left sympathetic trunk
C
The abdominal aorta
R
M
h
la
Sa
Image sourced from Wikipedia

Next question
C
R
M
Question 157 of 235

Which of the following statements relating to the greater omentum is false?

It is less well developed in children under 5.

It has no relationship to the lesser sac.

h
It contains the gastroepiploic arteries.

la
Has an attachment to the transverse colon.

It may be a site of metastatic disease in ovarian cancer.


Sa
It is connected with the lesser sac and the transverse colon. This plane is entered when performing
a colonic resection. It is a common site of metastasis in many visceral malignancies.
Please rate this question:
C
Discuss and give feedback
Next question

Omentum
R

• The omentum is divided into two parts which invest the stomach. Giving rise to the greater
and lesser omentum. The greater omentum is attached to the inferolateral border of the
stomach and houses the gastro-epiploic arteries.
• It is of variable size but is less well developed in children. This is important as the omentum
M

confers protection against visceral perforation (e.g. Appendicitis).


• Inferiorly between the omentum and transverse colon is one potential entry point into the
lesser sac.
• Several malignant processes may involve the omentum of which ovarian cancer is the most
notable.

Next question
Question 158 of 235

A 48 year old man with newly diagnosed hypertension is found to have a phaeochromocytoma of the
left adrenal gland and is due to undergo a laparoscopic left adrenalectomy. Which of the following
structures is not directly related to the left adrenal gland?

h
Crus of the diaphragm

la
Lesser curvature of the stomach

Kidney Sa
Pancreas

Splenic artery
C
The left adrenal gland is slightly larger than the right. It is crescent in shape and its concavity is
adapted to the medial border of the upper part of the left kidney. The upper area is covered by
peritoneum of the omental bursa which separates it from the cardia of the stomach. The lower area
is in contact with the pancreas and splenic artery and is not covered by peritoneum. On the anterior
R

surface is a hilum from which the suprarenal vein emerges. The lateral aspect rests on the kidney.
The medial is small and is on the left crus of the diaphragm.
Please rate this question:
M

Discuss and give feedback

Next question

Adrenal gland anatomy

Anatomy
Location Superomedially to the upper pole of each kidney

Relationships of the right Diaphragm-Posteriorly, Kidney-Inferiorly, Vena Cava-Medially, Hepato-renal


adrenal pouch and bare area of the liver-Anteriorly

Relationships of the left Crus of the diaphragm-Postero- medially, Pancreas and splenic vessels-
adrenal Inferiorly, Lesser sac and stomach-Anteriorly

h
Superior adrenal arteries- from inferior phrenic artery, Middle adrenal
Arterial supply arteries - from aorta, Inferior adrenal arteries -from renal arteries

la
Venous drainage of the Via one central vein directly into the IVC
right adrenal

Venous drainage of the


left adrenal
Sa
Via one central vein into the left renal vein

Next question
C
R
M
Question 160 of 235

An 18 year old boy is undergoing an appendicectomy for appendicitis. At which of the following
locations is the appendix most likely to be found?

Pre ileal

h
Pelvic

la
Retrocaecal

Post ileal

None of the above


Sa
Most appendixes lie in the retrocaecal position. If a retrocaecal appendix is difficult to remove then
mobilisation of the right colon significantly improves access.
Please rate this question:
C
Discuss and give feedback
Next question
R

Appendix

• Location: Base of caecum.



M

Up to 10cm long.
• Mainly lymphoid tissue (Hence mesenteric adenitis may mimic appendicitis).
• Caecal taenia coli converge at base of appendix and form a longitudinal muscle cover over
the appendix. This convergence should facilitate its identification at surgery if it is retrocaecal
and difficult to find (which it can be when people start doing appendicectomies!)
• Arterial supply: Appendicular artery (branch of the ileocolic).
• It is intra peritoneal.

McBurney's point
• 1/3 of the way along a line drawn from the Anterior Superior Iliac Spine to the Umbilicus

6 Positions:

• Retrocaecal 74%
• Pelvic 21%
• Postileal
• Subcaecal
• Paracaecal
• Preileal

h
la
Sa
C
R
M
Question 161 of 235

A 56 year old man is undergoing a pancreatectomy for carcinoma. During resection of the gland
which of the following structures will the surgeon not encounter posterior to the pancreas itself?

Left crus of the diaphragm

Superior mesenteric vein

h
Common bile duct

la
Portal vein
Sa
Gastroduodenal artery

The gastroduodenal artery divides into the gastro-epiploic and pancreaticoduodenal arteries at the
superior aspect of the pancreas.
Please rate this question:
C
Discuss and give feedback
Next question

Pancreas
R

The pancreas is a retroperitoneal organ and lies posterior to the stomach. It may be accessed
surgically by dividing the peritoneal reflection that connects the greater omentum to the transverse
colon. The pancreatic head sits in the curvature of the duodenum. Its tail lies close to the hilum of
M

the spleen, a site of potential injury during splenectomy.

Relations
Posterior to the pancreas
Pancreatic head Inferior vena cava
Common bile duct
Right and left renal veins
Superior mesenteric vein and artery
Pancreatic neck Superior mesenteric vein, portal vein

Pancreatic body- Left renal vein


Crus of diaphragm
Psoas muscle
Adrenal gland
Kidney
Aorta

h
Pancreatic tail Left kidney

la
Anterior to the pancreas
Pancreatic head 1st part of the duodenum
Pylorus
Gastroduodenal artery
Sa SMA and SMV(uncinate process)

Pancreatic body Stomach


Duodenojejunal flexure
C
Pancreatic tail Splenic hilum

Superior to the pancreas


R

Coeliac trunk and its branches common hepatic artery and splenic artery

Grooves of the head of the pancreas


2nd and 3rd part of the duodenum
M

Arterial supply

• Head: pancreaticoduodenal artery


• Rest: splenic artery

Venous drainage

• Head: superior mesenteric vein


• Body and tail: splenic vein
Ampulla of Vater

• Merge of pancreatic duct and common bile duct


• Is an important landmark, halfway along the second part of the duodenum, that marks the
anatomical transition from foregut to midgut (also the site of transition between regions
supplied by coeliac trunk and SMA).

h
la
Sa
C
Image sourced from Wikipedia
R
M
Question 162 of 235

From which structure is the central tendon of the diaphragm derived?

Septum transversum

h
Pleuroperitoneal folds

la
Diaphragmatic crura

Dorsal mesocardium Sa
Oropharyngeal membrane

The septum transversum is a thick ridge of mesodermal tissue in the developing embryo that
separates the thoracic and abdominal cavities and forms the central tendon of the diaphragm.
Please rate this question:
C
Discuss and give feedback
Next question
R

Embryology of the diaphragm and diaphragmatic hernia

Embryology
The diaphragm is formed between the 5th and 7th weeks of gestation through the progressive fusion
M

of the septum transversum, pleuroperitoneal folds and via lateral muscular ingrowth. The muscular
origins of the diaphragm are somites located in cervical segments 3 to 5, which accounts for the long
path taken by the phrenic nerve. The components contribute to the following diaphragmatic
segments:

• Septum transversum - Central tendon


• Pleuroperitoneal membranes - Parietal membranes surrounding viscera
• Cervical somites C5 to C7 - Muscular component of the diaphragm

Diaphragmatic hernia
Type of hernia Features

Morgagni Anteriorly located


Minimal compromise on lung development
Minimal signs on antenatal ultrasound
Usually present later
Usually good prognosis

Bochdalek hernia Posteriorly located

h
Larger defect
Often diagnosed antenatally
Associated with pulmonary hypoplasia

la
Poor prognosis

The posterior hernias of Bochdalek are the most common type and if not diagnosed antenatally will
typically present soon after birth with respiratory distress. The classical finding is that of a scaphoid
Sa
abdomen on clinical examination because of herniation of the abdominal contents into the chest.
Bochdalek hernias are associated with a number of chromosomal abnormalities such as Trisomy 21
and 18. Infants have considerable respiratory distress due to hypoplasia of the developing lung.
Historically this was considered to be due to direct compression of the lung by herniated viscera.
This view over simplifies the situation and the pulmonary hypoplasia occurs concomitantly with the
hernial development, rather than as a direct result of it. The pulmonary hypoplasia is associated with
pulmonary hypertension and abnormalities of pulmonary vasculature. The pulmonary hypertension
renders infants at risk of right to left shunting (resulting in progressive and worsening hypoxia).
C
Diagnostic work up of these infants includes chest x-rays/ abdominal ultrasound scans and cardiac
echo.
Surgery forms the mainstay of treatment and both thoracic and abdominal approaches may be
utilised. Following reduction of the hernial contents a careful search needs to be made for a hernial
sac as failure to recognise and correct this will result in a high recurrence rate. Smaller defects may
R

be primarily closed, larger defects may require a patch to close the defect. Malrotation of the viscera
is a recognised association and may require surgical correct at the same procedure (favoring an
abdominal approach).
The mortality rate is 50-75% and is related to the degree of lung compromise and age at
presentation (considerably better in infants >24 hours old).
M
Question 163 of 235

A 55 year old man is admitted with a brisk haematemesis. He is taken to the endoscopy department
and an upper GI endoscopy is performed by the gastroenterologist. He identifies an ulcer on the
posterior duodenal wall and spends an eternity trying to control the bleeding with all the latest
haemostatic techniques. He eventually asks the surgeons for help. A laparotomy and anterior

h
duodenotomy are performed, as the surgeon opens the duodenum a vessel is spurting blood into the
duodenal lumen. From which of the following does this vessel arise?

la
Left gastric artery

Common hepatic artery

Right hepatic artery


Sa
Superior mesenteric artery
C
Splenic artery
R

The vessel will be the gastroduodenal artery, this arises from the common hepatic artery.
Please rate this question:
M

Discuss and give feedback

Next question

Gastroduodenal artery

Supplies
Pylorus, proximal part of the duodenum, and indirectly to the pancreatic head (via the anterior and
posterior superior pancreaticoduodenal arteries)
Path
The gastroduodenal artery most commonly arises from the common hepatic artery of the coeliac
trunk. It terminates by bifurcating into the right gastroepiploic artery and the superior
pancreaticoduodenal artery

h
la
Sa
C
R
M
Question 164 of 235

Which of the following is not a content of the rectus sheath?

Pyramidalis

Superior epigastric artery

h
Inferior epigastric vein

la
Internal iliac artery

Rectus abdominis
Sa
The rectus sheath also contains:
superior epigastric vein
inferior epigastric artery
Please rate this question:
C
Discuss and give feedback
Next question
R

Abdominal wall

The 2 main muscles of the abdominal wall are the rectus abdominis (anterior) and the quadratus
lumborum (posterior).
The remaining abdominal wall consists of 3 muscular layers. Each muscle passes from the lateral
M

aspect of the quadratus lumborum posteriorly to the lateral margin of the rectus sheath anteriorly.
Each layer is muscular posterolaterally and aponeurotic anteriorly.
Image sourced from Wikipedia

Muscles of abdominal wall


External • Lies most superficially
oblique • Originates from 5th to 12th ribs
• Inserts into the anterior half of the outer aspect of the iliac crest, linea
alba and pubic tubercle
• More medially and superiorly to the arcuate line, the aponeurotic layer
overlaps the rectus abdominis muscle
• The lower border forms the inguinal ligament
• The triangular expansion of the medial end of the inguinal ligament is
the lacunar ligament.

h
Internal • Arises from the thoracolumbar fascia, the anterior 2/3 of the iliac crest

la
oblique and the lateral 2/3 of the inguinal ligament
• The muscle sweeps upwards to insert into the cartilages of the lower 3
ribs
• The lower fibres form an aponeurosis that runs from the tenth costal
cartilage to the body of the pubis

Sa
At its lowermost aspect it joins the fibres of the aponeurosis of
transversus abdominis to form the conjoint tendon.

Transversus • Innermost muscle


abdominis • Arises from the inner aspect of the costal cartilages of the lower 6 ribs ,
from the anterior 2/3 of the iliac crest and lateral 1/3 of the inguinal
C
ligament
• Its fibres run horizontally around the abdominal wall ending in an
aponeurosis. The upper part runs posterior to the rectus abdominis.
Lower down the fibres run anteriorly only.
• The rectus abdominis lies medially; running from the pubic crest and
R

symphysis to insert into the xiphoid process and 5th, 6th and 7th costal
cartilages. The muscles lies in a aponeurosis as described above.
• Nerve supply: anterior primary rami of T7-12
M

Surgical notes
During abdominal surgery it is usually necessary to divide either the muscles or their aponeuroses.
During a midline laparotomy it is desirable to divide the aponeurosis. This will leave the rectus
sheath intact above the arcuate line and the muscles intact below it. Straying off the midline will
often lead to damage to the rectus muscles, particularly below the arcuate line where they may often
be in close proximity to each other.
Question 165 of 235

Which of the following vessels does not drain directly into the inferior vena cava?

Superior mesenteric vein

h
Right common iliac

la
Right hepatic vein

Left hepatic vein


Sa
Right testicular vein

The superior mesenteric vein drains into the portal vein. The right and left hepatic veins drain into it
C
directly, this can account for major bleeding in more extensive liver shearing type injuries.
Please rate this question:

Discuss and give feedback


R

Next question

Inferior vena cava


M

Origin

• L5

Path

• Left and right common iliac veins merge to form the IVC.
• Passes right of midline
• Paired segmental lumbar veins drain into the IVC throughout its length
• The right gonadal vein empties directly into the cava and the left gonadal vein generally
empties into the left renal vein.
• The next major veins are the renal veins and the hepatic veins
• Pierces the central tendon of diaphragm at T8
• Right atrium

h
la
Sa
Image sourced from Wikipedia

Relations
C
Anteriorly Small bowel, first and third part of duodenum, head of pancreas, liver and bile duct,
right common iliac artery, right gonadal artery
R

Posteriorly Right renal artery, right psoas, right sympathetic chain, coeliac ganglion

Levels
M

Level Vein

T8 Hepatic vein, inferior phrenic vein, pierces diaphragm

L1 Suprarenal veins, renal vein


L2 Gonadal vein

L1-5 Lumbar veins

L5 Common iliac vein, formation of IVC

Next question

h
la
Sa
C
R
M
Question 166 of 235

A 17 year old male has a suspected testicular torsion and the scrotum is to be explored surgically.
The surgeon incises the skin and then the dartos muscle. What is the next tissue layer that will be
encountered during the dissection?

Visceral layer of the tunica vaginalis

h
Cremasteric fascia

la
Parietal layer of the tunica vaginalis

External spermatic fascia


Sa
Internal spermatic fascia

The layers that will be encountered are (in order):


1. Skin
2. Dartos fascia and muscle
C
3. External spermatic fascia
4. Cremasteric muscle and fascia
5. Internal spermatic fascia
6. Parietal layer of the tunica vaginalis
R

The layers of the spermatic cord and scrotum are a popular topic in the MRCS exam.
A mnemonic which may help:
Some Damned Examiner Called It The Testes (skin dartos external fascia cremaster internal fascia
tunica Testes)
Please rate this question:
M

Discuss and give feedback


Next question

Scrotal and testicular anatomy

Spermatic cord
Formed by the vas deferens and is covered by the following structures:
Layer Origin

Internal spermatic fascia Transversalis fascia

Cremasteric fascia From the fascial coverings of internal oblique

External spermatic fascia External oblique aponeurosis

h
Contents of the cord

la
Vas deferens Transmits sperm and accessory gland secretions

Testicular artery Branch of abdominal aorta supplies testis and


epididymis

Artery of vas deferens


Sa Arises from inferior vesical artery

Cremasteric artery Arises from inferior epigastric artery


C
Pampiniform plexus Venous plexus, drains into right or left testicular vein

Sympathetic nerve fibres Lie on arteries, the parasympathetic fibres lie on the
R

vas

Genital branch of the genitofemoral Supplies cremaster


M

nerve

Lymphatic vessels Drain to lumbar and para-aortic nodes

Scrotum

• Composed of skin and closely attached dartos fascia.


• Arterial supply from the anterior and posterior scrotal arteries
• Lymphatic drainage to the inguinal lymph nodes
• Parietal layer of the tunica vaginalis is the innermost layer

Testes

• The testes are surrounded by the tunica vaginalis (closed peritoneal sac). The parietal layer
of the tunica vaginalis adjacent to the internal spermatic fascia.
• The testicular arteries arise from the aorta immediately inferiorly to the renal arteries.
• The pampiniform plexus drains into the testicular veins, the left drains into the left renal vein
and the right into the inferior vena cava.
• Lymphatic drainage is to the para-aortic nodes.

h
Next question

la
Sa
C
R
M
Question 167 of 235

A 19 year old man undergoes an open inguinal hernia repair. The cord is mobilised and the deep
inguinal ring identified. Which of the following structures forms its lateral wall?

External oblique aponeurosis

Transversalis fascia

h
Conjoint tendon

la
Inferior epigastric artery

Inferior epigastric vein


Sa
The transversalis fascia forms the superolateral edge of the deep inguinal ring. The epigastric
vessels form its inferomedial wall.
Please rate this question:
C
Discuss and give feedback
Next question

Inguinal canal
R

Location

• Above the inguinal ligament


• The inguinal canal is 4cm long
M

• The superficial ring is located anterior to the pubic tubercle


• The deep ring is located approximately 1.5-2cm above the half way point between the
anterior superior iliac spine and the pubic tubercle

Boundaries of the inguinal canal


Floor • External oblique aponeurosis
• Inguinal ligament
• Lacunar ligament
Roof • Internal oblique
• Transversus abdominis

Anterior wall External oblique aponeurosis

Posterior wall • Transversalis fascia


• Conjoint tendon

Laterally • Internal ring

h
• Transversalis fascia
• Fibres of internal oblique

la
Medially • External ring
• Conjoint tendon

Contents
Males
Sa
Spermatic cord and ilioinguinal As it passes through the canal the spermatic cord
nerve has 3 coverings:

• External spermatic fascia from external


oblique aponeurosis
C
• Cremasteric fascia
• Internal spermatic fascia
R

Females Round ligament of uterus and


ilioinguinal nerve

Related anatomy of the inguinal region


M

The boundaries of Hesselbachs triangle are commonly tested and illustrated below:
h
la
Sa
Image sourced from Wikipedia

The image below demonstrates the close relationship of the vessels to the lower limb with the
inguinal canal. A fact to be borne in mind when repairing hernial defects in this region.
C
R
M

Image sourced from Wikipedia


Question 168 of 235

h
During an inguinal hernia repair the surgeon identifies a small nerve whilst mobilising the cord
structures at the level of the superficial inguinal ring. Which nerve is this most likely to be?

la
Subcostal

Iliohypogastric

Ilioinguinal
Sa
Obturator
C
Pudendal

Ilioinguinal nerve entrapment may be a cause of neuropathic pain following inguinal hernia surgery.
R

The ilioinguinal nerve passes through the superfical inguinal ring and is routinely encountered when
exploring the inguinal canal during hernia surgery. The iliohypogastric nerve pierces the aponeurosis
of the external oblique muscle superior to the superficial inguinal ring.
Please rate this question:
M

Discuss and give feedback


Next question

Ilioinguinal nerve

Arises from the first lumbar ventral ramus with the iliohypogastric nerve. It passes inferolaterally
through the substance of psoas major and over the anterior surface of quadratus lumborum. It
pierces the internal oblique muscle and passes deep to the aponeurosis of the external oblique
muscle. It enters the inguinal canal and then passes through the superficial inguinal ring to reach the
skin.
Branches

• To supply those muscles of the abdominal wall through which it passes.


• Skin and fascia over the pubic symphysis, superomedial part of the femoral triangle, surface
of the scrotum, root and dorsum of penis or labum majus in females.

h
la
Sa
C
R
M
Question 169 of 235

What is the lymphatic drainage of the membranous urethra?

Deep inguinal nodes

h
Superficial inguinal nodes

la
Internal iliac nodes

External iliac nodes


Sa
Para-aortic nodes
C
The prostatic and membranous urethra drain to the internal iliac nodes.
Please rate this question:
R

Discuss and give feedback

Next question
M

Urethral anatomy

Female urethra
The female urethra is shorter and more acutely angulated than the male urethra. It is an extra-
peritoneal structure and embedded in the endopelvic fascia. The neck of the bladder is subjected to
transmitted intra-abdominal pressure and therefore deficiency in this area may result in stress
urinary incontinence. Between the layers of the urogenital diaphragm the female urethra is
surrounded by the external urethral sphincter, this is innervated by the pudendal nerve. It ultimately
lies anterior to the vaginal orifice.

Male urethra
In males the urethra is much longer and is divided into four parts.

Pre-prostatic Extremely short and lies between the bladder and prostate gland.It has a stellate lumen
urethra and is between 1 and 1.5cm long.Innervated by sympathetic noradrenergic fibres, as
this region is composed of striated muscles bundles they may contract and prevent
retrograde ejaculation.

Prostatic This segment is wider than the membranous urethra and contains several openings for
urethra the transmission of semen (at the midpoint of the urethral crest).

h
Membranous Narrowest part of the urethra and surrounded by external sphincter. It traverses the

la
urethra perineal membrane 2.5cm postero-inferior to the symphysis pubis.

Penile urethra Travels through the corpus spongiosum on the underside of the penis. It is the longest
urethral segment.It is dilated at its origin as the infrabulbar fossa and again in the gland
Sa
penis as the navicular fossa. The bulbo-urethral glands open into the spongiose section
of the urethra 2.5cm below the perineal membrane.

The urothelium is transitional in nature near to the bladder and becomes squamous more distally.
C
R
M
Question 170 of 235

A 34 year old lady presents with symptoms of faecal incontinence. Ten years previously she gave
birth to a child by normal vaginal delivery. Injury to which of the following nerves is most likely to
account for this process?

h
Genitofemoral

Ilioinguinal

la
Pudendal Sa
Hypogastric autonomic nerve

Obturator

S2,3,4 keeps the poo up off the floor - POOdendal nerve


C
Damage to the pudendal nerve is classically associated with faecal incontinence and it is for this
reason that sacral neuromodulation is a popular treatment for the condition. Injury to the hypogastric
autonomic nerves is an aetiological factor in the development of constipation.
R

Please rate this question:


M

Discuss and give feedback

Next question

Pudendal nerve

The pudendal nerve arises from nerve roots S2, S3 and S4 and exits the pelvis through the greater
sciatic foramen. It re-enters the perineum through the lesser sciatic foramen. It travels inferior to give
innervation to the anal sphincters and external urethral sphincter. It also provides cutaneous
innervation to the region of perineum surrounding the anus and posterior vulva.
Traction and compression of the pudendal nerve by the foetus in late pregnancy may result in late
onset pudendal neuropathy which may be part of the process involved in the development of faecal
incontinence.

h
la
Sa
C
R
M
Question 171 of 235

During a difficult thyroidectomy haemorrhage is noted from the thyroidea ima vessel. From which
structure does this vessel usually arise?

External carotid artery

Internal carotid artery

h
Brachiocephalic artery

la
Axillary artery

Superior thyroid artery

Rhyme isthmus location:


Sa
Rings 2,3,4 make the isthmus floor

This accessory vessel which usually lies at the inferior aspect of the gland is derived either from the
brachiocephalic artery or the arch of the aorta.
C
Please rate this question:

Discuss and give feedback


R

Next question

Thyroid gland

• Right and left lobes connected by isthmus


M

• Surrounded by sheath from pretracheal layer of deep fascia


• Apex: Lamina of thyroid cartilage
• Base: 4th-5th tracheal ring
• Pyramidal lobe: from isthmus
• May be attached to foramen caecum at the base of the tongue

Relations
Anteromedially • Sternothyroid
• Superior belly of omohyoid
• Sternohyoid
• Anterior aspect of sternocleidomastoid

Posterolaterally Carotid sheath

Medially • Larynx
• Trachea
• Pharynx
• Oesophagus
• Cricothyroid muscle
• External laryngeal nerve (near superior thyroid artery)

h
• Recurrent laryngeal nerve (near inferior thyroid artery)

la
Posterior Parathyroid glands
• Anastomosis of superior and inferior thyroid arteries

Isthmus • Anteriorly: Sternothyroids, sternohyoids, anterior jugular veins



Sa
Posteriorly: 2nd, 3rd, 4th tracheal rings (attached via Ligament of
Berry)

Blood Supply
Arterial • Superior thyroid artery (1st branch of external carotid)
• Inferior thyroid artery (from thyrocervical trunk)
C
• Thyroidea ima (in 10% of population -from brachiocephalic artery or aorta)

Venous • Superior and middle thyroid veins - into the IJV


R

• Inferior thyroid vein - into the brachiocephalic veins


M
Question 172 of 235

Which of the following is not a branch of the abdominal aorta?

Inferior mesenteric artery

h
Inferior phrenic artery

la
Superior mesenteric artery

Superior phrenic artery

Renal artery
Sa
Mnemonic for the Descending abdominal aorta branches from diaphragm to iliacs:

'Prostitutes Cause Sagging Swollen Red Testicles [in men] Living In Sin':
C
Phrenic [inferior]
Celiac
Superior mesenteric
Suprarenal [middle]
R

Renal
Testicular ['in men' only]
Lumbars
Inferior mesenteric
M

Sacral

The superior phrenic artery branches from the aorta in the thorax.
Please rate this question:

Discuss and give feedback

Next question
Abdominal aortic branches

Branches Level Paired Type

Inferior phrenic T12 (Upper border) Yes Parietal

Coeliac T12 No Visceral

h
Superior mesenteric L1 No Visceral

la
Middle suprarenal L1 Yes Visceral

Renal L1-L2 Yes Visceral

Gonadal
Sa L2 Yes Visceral

Lumbar L1-L4 Yes Parietal


C
Inferior mesenteric L3 No Visceral

Median sacral L4 No Parietal


R

Common iliac L4 Yes Terminal


M
Question 173 of 235

A 23 year old man is admitted with a suspected ureteric colic. A KUB style x-ray is obtained. In
which of the following locations is the stone most likely to be visualised?

The tips of the transverse processes between L2 and L5

h
The tips of transverse processes between T10-L1

la
At the crest of the ilium

Over the S3 foramina


Sa
Over the sacrococcygeal joint

The ureter lies anterior to L2 to L5 and stones may be visualised at these points, they may also be
identified over the sacro-iliac joints.
Please rate this question:
C
Discuss and give feedback
Next question
R

Ureter

• 25-35 cm long
• Muscular tube lined by transitional epithelium

M

Surrounded by thick muscular coat. Becomes 3 muscular layers as it crosses the bony pelvis
• Retroperitoneal structure overlying transverse processes L2-L5
• Lies anterior to bifurcation of iliac vessels
• Blood supply is segmental; renal artery, aortic branches, gonadal branches, common iliac
and internal iliac
• Lies beneath the uterine artery

Next question
Question 174 of 235

In a patient with an ectopic kidney where is the adrenal gland most likely to be located?

In the pelvis

h
On the contralateral side

la
In its usual position

Superior to the spleen

It will be absent
Sa
Because the kidney is present, rather than absent, the adrenal will usually develop and in the normal
location.
C
Please rate this question:
R

Discuss and give feedback

Next question

Adrenal gland embryology


M

First detected at 6 weeks' gestation, the adrenal cortex is derived from the mesoderm of the
posterior abdominal wall. Steroid secretion from the fetal cortex begins shortly thereafter. Adult-type
zona glomerulosa and fasciculata are detected in fetal life but make up only a small proportion of the
gland, and the zona reticularis is not present at all. The fetal cortex predominates throughout fetal
life. The adrenal medulla is of ectodermal origin, arising from neural crest cells that migrate to the
medial aspect of the developing cortex.

The fetal adrenal gland is relatively large. At 4 months' gestation, it is 4 times the size of the kidney;
however, at birth, it is a third of the size of the kidney. This occurs because of the rapid regression of
the fetal cortex at birth. It disappears almost completely by age 1 year; by age 4-5 years, the
permanent adult-type adrenal cortex has fully developed.

Anatomic anomalies of the adrenal gland may occur. Because the development of the adrenals is
closely associated with that of the kidneys, agenesis of an adrenal gland is usually associated with
ipsilateral agenesis of the kidney, and fused adrenal glands (whereby the 2 glands join across the
midline posterior to the aorta) are also associated with a fused kidney.

Adrenal hypoplasia occurs in the following 2 forms: (1) hypoplasia or absence of the fetal cortex with
a poorly formed medulla and (2) disorganized fetal cortex and medulla with no permanent cortex
present. Adrenal heterotopia describes a normal adrenal gland in an abnormal location, such as
within the renal or hepatic capsules. Accessory adrenal tissue (adrenal rests), which is usually
comprised only of cortex but seen combined with medulla in some cases, is most commonly located

h
in the broad ligament or spermatic cord but can be found anywhere within the abdomen. Even
intracranial adrenal rests have been reported
Next question

la
Sa
C
R
M
Question 177 of 235

A 55 year old man is due to undergo a radical prostatectomy for carcinoma of the prostate gland.
Which of the following vessels directly supplies the prostate?

External iliac artery

Common iliac artery

h
Internal iliac artery

la
Inferior vesical artery

None of the above


Sa
The arterial supply to the prostate gland is from the inferior vesical artery, it is a branch of the
prostatovesical artery. The prostatovesical artery usually arises from the internal pudendal and
inferior gluteal arterial branches of the internal iliac artery.
Please rate this question:
C
Discuss and give feedback
Next question

Prostate gland
R

The prostate gland is approximately the shape and size of a walnut and is located inferior to the
bladder. It is separated from the rectum by Denonvilliers fascia and its blood supply is derived from
the internal iliac vessels (via inferior vesical artery). The internal sphincter lies at the apex of the
M

gland and may be damaged during prostatic surgery, affected individuals may complain of
retrograde ejaculation.

Summary of prostate gland


Arterial supply Inferior vesical artery (from internal iliac)

Venous drainage Prostatic venous plexus (to paravertebral veins)


Lymphatic Internal iliac nodes
drainage

Innervation Inferior hypogastric plexus

Dimensions • Transverse diameter (4cm)


• AP diameter (2cm)
• Height (3cm)

h
Lobes • Posterior lobe: posterior to urethra
• Median lobe: posterior to urethra, in between ejaculatory ducts

la
• Lateral lobes x 2
• Isthmus

Zones • Peripheral zone: subcapsular portion of posterior prostate. Most


Sa


prostate cancers are here
Central zone
Transition zone
• Stroma

Relations
C
Pubic symphysis
Anterior Prostatic venous plexus
R

Posterior Denonvilliers fascia


Rectum
Ejaculatory ducts
M

Lateral Venous plexus (lies on prostate)


Levator ani (immediately below the puboprostatic ligaments)
h
la
Image sourced from Wikipedia

Next question
Sa
C
R
M
Question 181 of 235

From which embryological structure is the ureter derived?

Uranchus

Cloaca

Vitello-intestinal duct

h
Mesonephric duct

la
None of the above
Sa
The ureter develops from and outpouching that arises from the mesonephric duct. The mesonephric
duct is associated with the metanephric duct that develops within the metenephrogenic blastema.
This forms the site of the ureteric bud which branches off the mesonephric duct.
Please rate this question:

Discuss and give feedback


C
Next question

Ureter

• 25-35 cm long
R

• Muscular tube lined by transitional epithelium


• Surrounded by thick muscular coat. Becomes 3 muscular layers as it crosses the bony pelvis
• Retroperitoneal structure overlying transverse processes L2-L5
• Lies anterior to bifurcation of iliac vessels
• Blood supply is segmental; renal artery, aortic branches, gonadal branches, common iliac
M

and internal iliac


• Lies beneath the uterine artery

Next question
Question 185 of 235

A 62 year old man is undergoing a left hemicolectomy for carcinoma of the descending colon. The
registrar commences mobilisation of the left colon by pulling downwards and medially. Blood soon
appears in the left paracolic gutter. The most likely source of bleeding is the:

Marginal artery

Left testicular artery

h
Spleen

la
Left renal vein

None of the above


Sa
The spleen is commonly torn by traction injuries in colonic surgery. The other structures are
associated with bleeding during colonic surgery but would not manifest themselves as blood in the
paracolic gutter prior to incision of the paracolonic peritoneal edge.
Please rate this question:
C
Discuss and give feedback
Next question
R

Left colon

Position
M

• As the left colon passes inferiorly its posterior aspect becomes extraperitoneal, and the
ureter and gonadal vessels are close posterior relations that may become involved in
disease processes
• At a level of L3-4 (variable) the left colon becomes the sigmoid colon and wholly
intraperitoneal once again
• The sigmoid colon is a highly mobile structure and may even lie on the right side of the
abdomen
• It passes towards the midline, the taenia blend and this marks the transition between sigmoid
colon and upper rectum
Blood supply

• Inferior mesenteric artery


• However, the marginal artery (from the right colon) contributes, this contribution becomes
clinically significant when the IMA is divided surgically (e.g. During AAA repair)

Next question

h
la
Sa
C
R
M
Question 186 of 235

A 53 year old man with a chronically infected right kidney is due to undergo a nephrectomy. Which of
the following structures would be encountered first during a posterior approach to the hilum of the
right kidney?

Right renal artery

Ureter

h
Right renal vein

la
Inferior vena cava
Sa
Right testicular vein

The ureter is the most posterior structure at the hilum of the right kidney and would therefore be
encountered first during a posterior approach.
Please rate this question:
C
Discuss and give feedback
Next question

Renal arteries
R

• The right renal artery is longer than the left renal artery
• The renal vein/artery/pelvis enter the kidney at the hilum
M

Relations
Right Anterior- IVC, right renal vein, the head of the pancreas, and the descending part of the
duodenum

Left Anterior- left renal vein, the tail of the pancreas

Branches
• The renal arteries are direct branches off the aorta (upper border of L2- right side and L1 -
left side)
• In 30% there may be accessory arteries (mainly left side). Instead of entering the kidney at
the hilum, they usually pierce the upper or lower part of the organ.
• Before reaching the hilum of the kidney, each artery divides into four or five segmental
branches (renal vein anterior and ureter posterior); which then divide within the sinus into
lobar arteries supplying each pyramid and cortex.
• Each vessel gives off some small inferior suprarenal branches to the suprarenal gland, the
ureter, and the surrounding cellular tissue and muscles.

Next question

h
la
Sa
C
R
M
Question 188 of 235

Which of the following regions of the male urethra is entirely surrounded by Bucks fascia?

Preprostatic part

h
Prostatic part

la
Membranous part

Spongiose part Sa
None of the above

Bucks fascia is a layer of deep fascia that covers the penis it is continuous with the external
spermatic fascia and the penile suspensory ligament. The membranous part of the urethra may
partially pass through Bucks fascia as it passes into the penis. However, the spongiose part of the
C
urethra is contained wholly within Bucks fascia.

Image of penile cross section


Bucks fascia corresponds to the layer of deep fascia
R
M

Image sourced from Wikipedia

Please rate this question:


Discuss and give feedback

Next question

Urethral anatomy

Female urethra
The female urethra is shorter and more acutely angulated than the male urethra. It is an extra-
peritoneal structure and embedded in the endopelvic fascia. The neck of the bladder is subjected to
transmitted intra-abdominal pressure and therefore deficiency in this area may result in stress

h
urinary incontinence. Between the layers of the urogenital diaphragm the female urethra is
surrounded by the external urethral sphincter, this is innervated by the pudendal nerve. It ultimately
lies anterior to the vaginal orifice.

la
Male urethra
In males the urethra is much longer and is divided into four parts.

Pre-prostatic Extremely short and lies between the bladder and prostate gland.It has a stellate lumen
urethra
Sa
and is between 1 and 1.5cm long.Innervated by sympathetic noradrenergic fibres, as
this region is composed of striated muscles bundles they may contract and prevent
retrograde ejaculation.

Prostatic This segment is wider than the membranous urethra and contains several openings for
urethra the transmission of semen (at the midpoint of the urethral crest).
C
Membranous Narrowest part of the urethra and surrounded by external sphincter. It traverses the
urethra perineal membrane 2.5cm postero-inferior to the symphysis pubis.
R

Penile urethra Travels through the corpus spongiosum on the underside of the penis. It is the longest
urethral segment.It is dilated at its origin as the infrabulbar fossa and again in the gland
M

penis as the navicular fossa. The bulbo-urethral glands open into the spongiose section
of the urethra 2.5cm below the perineal membrane.

The urothelium is transitional in nature near to the bladder and becomes squamous more distally.
Question 189 of 235

A 48 year old lady is undergoing a left sided adrenalectomy for an adrenal adenoma. The superior
adrenal artery is injured and starts to bleed, from which of the following does this vessel arise?

Left renal artery

h
Inferior phrenic artery

la
Aorta

Splenic

None of the above


Sa
The superior adrenal artery is a branch of the inferior phrenic artery.
Please rate this question:
C
R

Discuss and give feedback

Next question

Adrenal gland anatomy


M

Anatomy

Location Superomedially to the upper pole of each kidney

Relationships of the right Diaphragm-Posteriorly, Kidney-Inferiorly, Vena Cava-Medially, Hepato-renal


adrenal pouch and bare area of the liver-Anteriorly
Relationships of the left Crus of the diaphragm-Postero- medially, Pancreas and splenic vessels-
adrenal Inferiorly, Lesser sac and stomach-Anteriorly

Superior adrenal arteries- from inferior phrenic artery, Middle adrenal


Arterial supply arteries - from aorta, Inferior adrenal arteries -from renal arteries

Venous drainage of the Via one central vein directly into the IVC
right adrenal

h
Venous drainage of the Via one central vein into the left renal vein
left adrenal

la
Next question
Sa
C
R
M
Question 190 of 235

An 80 year old lady with a caecal carcinoma is undergoing a right hemicolectomy performed through
a transverse incision. The procedure is difficult and the incision is extended medially by dividing the
rectus sheath. Brisk arterial haemorrhage ensues. From which of the following does the damaged
vessel originate?

Internal iliac artery

h
External iliac artery

la
Superior vesical artery

Inferior vesical artery

None of the above


Sa
The vessel damaged is the epigastric artery. This originates from the external iliac artery (see
below).
C
Please rate this question:
R

Discuss and give feedback

Next question

Epigastric artery
M

The inferior epigastric artery arises from the external iliac artery immediately above the inguinal
ligament. It then passes along the medial margin of the deep inguinal ring. From here it continues
superiorly to lie behind the rectus abdominis muscle.

This is illustrated below:


h
la
Sa
Image sourced from Wikipedia

Next question
C
R
M
Question 191 of 235

A 73 year old man has a large abdominal aortic aneurysm. During a laparotomy for planned surgical
repair the surgeons find the aneurysm is far more proximally located and lies near the origin of the
SMA. During the dissection a vessel lying transversely across the aorta is injured. What is this
vessel most likely to be?

Left renal vein

h
Right renal vein

la
Inferior mesenteric artery

Ileocolic artery
Sa
Middle colic artery

The left renal vein runs across the surface of the aorta and may require deliberate ligation during
C
juxtarenal aneurysm repair.
Please rate this question:
R

Discuss and give feedback

Next question
M

Abdominal aorta

Abdominal aortic topography

Origin T12

Termination L4
Posterior relations L1-L4 Vertebral bodies

Anterior relations Lesser omentum


Liver
Left renal vein
Inferior mesenteric vein
Third part of duodenum
Pancreas
Parietal peritoneum
Peritoneal cavity

h
Right lateral relations Right crus of the diaphragm

la
Cisterna chyli
IVC (becomes posterior distally)

Left lateral relations


Sa
4th part of duodenum
Duodenal-jejunal flexure
Left sympathetic trunk

The abdominal aorta


C
R
M
h
la
Sa
Image sourced from Wikipedia

Next question
C
R
M
Question 192 of 235

A 18 year old man presents with an indirect inguinal hernia and undergoes surgery. The deep
inguinal ring is exposed and held with a retractor at its medial aspect. Which structure is most likely
to lie under the retractor?

Ureter

Inferior epigastric artery

h
Internal iliac vein

la
Femoral artery
Sa
Lateral border of rectus abdominis

Boundaries of the deep inguinal ring:

• Superolaterally - transversalis fascia


• Inferomedially - inferior epigastric artery
C
The deep inguinal ring is closely related to the inferior epigastric artery. The inferior epigastric artery
forms part of the structure referred to as Hesselbach's triangle.
R
M
h
la
Sa
Image sourced from Wikipedia

Please rate this question:

Discuss and give feedback


C
Next question

Inguinal canal

Location
R

• Above the inguinal ligament


• The inguinal canal is 4cm long
• The superficial ring is located anterior to the pubic tubercle
M

• The deep ring is located approximately 1.5-2cm above the half way point between the
anterior superior iliac spine and the pubic tubercle

Boundaries of the inguinal canal


Floor • External oblique aponeurosis
• Inguinal ligament
• Lacunar ligament
Roof • Internal oblique
• Transversus abdominis

Anterior wall External oblique aponeurosis

Posterior wall • Transversalis fascia


• Conjoint tendon

Laterally • Internal ring

h
• Transversalis fascia
• Fibres of internal oblique

la
Medially • External ring
• Conjoint tendon

Contents
Males
Sa
Spermatic cord and ilioinguinal As it passes through the canal the spermatic cord
nerve has 3 coverings:

• External spermatic fascia from external


oblique aponeurosis
C
• Cremasteric fascia
• Internal spermatic fascia
R

Females Round ligament of uterus and


ilioinguinal nerve

Related anatomy of the inguinal region


M

The boundaries of Hesselbachs triangle are commonly tested and illustrated below:
h
la
Sa
Image sourced from Wikipedia

The image below demonstrates the close relationship of the vessels to the lower limb with the
inguinal canal. A fact to be borne in mind when repairing hernial defects in this region.
C
R
M

Image sourced from Wikipedia

Next question
Question 194 of 235

In a patient with a carcinoma of the distal sigmoid colon, what is the most likely source of its blood
supply?

Ileocolic artery

External iliac artery

h
Internal iliac artery

la
Superior mesenteric artery

Inferior mesenteric artery


Sa
During a high anterior resection of such tumours, the inferior mesenteric artery is ligated. Note that
the branches (mainly middle rectal branch) of the internal iliac artery are important in maintaining
vascularity of the rectal stump and hence the integrity of the anastomoses.
Please rate this question:
C
Discuss and give feedback
Next question

Rectum
R

The rectum is approximately 12 cm long. It is a capacitance organ. It has both intra and
extraperitoneal components. The transition between the sigmoid colon is marked by the
disappearance of the tenia coli.The extra peritoneal rectum is surrounded by mesorectal fat that also
contains lymph nodes. This mesorectal fatty layer is removed surgically during rectal cancer surgery
(Total Mesorectal Excision). The fascial layers that surround the rectum are important clinical
M

landmarks, anteriorly lies the fascia of Denonvilliers. Posteriorly lies Waldeyers fascia.

Extra peritoneal rectum

• Posterior upper third


• Posterior and lateral middle third
• Whole lower third

Relations
Anteriorly (Males) Rectovesical pouch
Bladder
Prostate
Seminal vesicles

Anteriorly (Females) Recto-uterine pouch (Douglas)


Cervix
Vaginal wall

h
Posteriorly Sacrum
Coccyx
Middle sacral artery

la
Laterally Levator ani
Sa Coccygeus

Arterial supply
Superior rectal artery

Venous drainage
Superior rectal vein
C
Lymphatic drainage

• Mesorectal lymph nodes (superior to dentate line)


• Inguinal nodes (inferior to dentate line)
R

Next question
M
Question 195 of 235

A patient is due to undergo a right hemicolectomy for a carcinoma of the caecum. Which of the
following vessels will require high ligation to provide optimal oncological control?

Middle colic artery

Inferior mesenteric artery

h
Superior mesenteric artery

la
Ileo-colic artery

None of the above


Sa
The ileo - colic artery supplies the caecum and would require high ligation during a right
hemicolectomy. The middle colic artery should generally be preserved when resecting a caecal
lesion.
This question is essentially asking you to name the vessel supplying the caecum. The SMA does not
directly supply the caecum, it is the ileocolic artery which does this.
C
Please rate this question:

Discuss and give feedback


R

Next question

Caecum


M

Location Proximal right colon below the ileocaecal valve


• Intraperitoneal

Posterior relations • Psoas


• Iliacus
• Femoral nerve
• Genitofemoral nerve
• Gonadal vessels
Anterior relations Greater omentum

Arterial supply Ileocolic artery

Lymphatic drainage Mesenteric nodes accompany the venous drainage

• The caecum is the most distensible part of the colon and in complete large bowel obstruction
with a competent ileocaecal valve the most likely site of eventual perforation.

h
Next question

la
Sa
C
R
M
Question 196 of 235

A 72 year old man is undergoing a repair of an abdominal aortic aneurysm. The aorta is cross
clamped both proximally and distally. The proximal clamp is applied immediately inferior to the renal
arteries. Both common iliac arteries are clamped distally. A longitudinal aortotomy is performed.
After evacuating the contents of the aneurysm sac a significant amount of ongoing bleeding is
encountered. This is most likely to originate from:

h
The coeliac axis

la
Testicular artery

Splenic artery Sa
Superior mesenteric artery

Lumbar arteries
C
The lumbar arteries are posteriorly sited and are a common cause of back bleeding during aortic
surgery. The other vessels cited all exit the aorta in the regions that have been cross clamped.
Please rate this question:
R

Discuss and give feedback


M

Next question

Abdominal aortic branches

Branches Level Paired Type

Inferior phrenic T12 (Upper border) Yes Parietal


Branches Level Paired Type

Coeliac T12 No Visceral

Superior mesenteric L1 No Visceral

Middle suprarenal L1 Yes Visceral

h
Renal L1-L2 Yes Visceral

la
Gonadal L2 Yes Visceral

Lumbar L1-L4 Yes Parietal


Sa
Inferior mesenteric L3 No Visceral

Median sacral L4 No Parietal


C
Common iliac L4 Yes Terminal
R

Next question
M
Question 197 of 235

A 63 year old man undergoes a radical cystectomy for carcinoma of the bladder. During the
procedure there is considerable venous bleeding. What is the primary site of venous drainage of the
urinary bladder?

h
Vesicoprostatic venous plexus

Internal iliac vein

la
External iliac vein Sa
Gonadal vein

Common iliac vein


C
The urinary bladder has a rich venous plexus surrounding it, this drains subsequently into the
internal iliac vein. The vesicoprostatic plexus may be a site of considerable venous bleeding during
cystectomy.
Please rate this question:
R

Discuss and give feedback


M

Next question

Bladder

The empty bladder is contained within the pelvic cavity. It is usually a three sided pyramid. The apex
of the bladder points forwards towards the symphysis pubis and the base lies immediately anterior to
the rectum or vagina. Continuous with the apex is the median umbilical ligament, during
development this was the site of the urachus.
The inferior aspect of the bladder is retroperitoneal and the superior aspect covered by peritoneum.
As the bladder distends it will tend to separate the peritoneum from the fascia of transversalis. For
this reason a bladder that is distended due to acute urinary retention may be approached with a
suprapubic catheter that avoids entry into the peritoneal cavity.
The trigone is the least mobile part of the bladder and forms the site of the ureteric orifices and
internal urethral orifice. In the empty bladder the ureteric orifices are approximately 2-3cm apart, this
distance may increase to 5cm in the distended bladder.

Arterial supply
The superior and inferior vesical arteries provide the main blood supply to the bladder. These are
branches of the internal iliac artery.

Venous drainage
In males the bladder is drained by the vesicoprostatic venous plexus. In females the bladder is
drained by the vesicouterine venous plexus. In both sexes this venous plexus will ultimately drain to

h
the internal iliac veins.

Lymphatic drainage
Lymphatic drainage is predominantly to the external iliac nodes, internal iliac and obturator nodes

la
also form sites of bladder lymphatic drainage.

Innervation
Parasympathetic nerve fibres innervate the bladder from the pelvic splanchnic nerves. Sympathetic
Sa
nerve fibres are derived from L1 and L2 via the hypogastric nerve plexuses. The parasympathetic
nerve fibres will typically cause detrusor muscle contraction and result in voiding. The muscle of the
trigone is innervated by the sympathetic nervous system. The external urethral sphincter is under
conscious control. During bladder filling the rate of firing of nerve impulses to the detrusor muscle is
low and receptive relaxation occurs. At higher volumes and increased intra vesical pressures the
rate of neuronal firing will increase and eventually voiding will occur.
Next question
C
R
M
Question 198 of 235

A 60 year old female is undergoing a Whipples procedure for adenocarcinoma of the pancreas. As
the surgeons begin to mobilise the pancreatic head they identify a large vessel passing inferiorly
over the anterior aspect of the uncinate process. What is it likely to be?

Superior mesenteric artery

h
Coeliac axis

la
Inferior mesenteric artery

Aorta Sa
Left gastric artery

The superior mesenteric artery arises from the aorta and passes anterior to the lower part of the
pancreas. Invasion of this structure is a relative contra indication to resectional surgery.
Please rate this question:
C
Discuss and give feedback
Next question
R

Pancreas

The pancreas is a retroperitoneal organ and lies posterior to the stomach. It may be accessed
surgically by dividing the peritoneal reflection that connects the greater omentum to the transverse
M

colon. The pancreatic head sits in the curvature of the duodenum. Its tail lies close to the hilum of
the spleen, a site of potential injury during splenectomy.

Relations
Posterior to the pancreas
Pancreatic head Inferior vena cava
Common bile duct
Right and left renal veins
Superior mesenteric vein and artery
Pancreatic neck Superior mesenteric vein, portal vein

Pancreatic body- Left renal vein


Crus of diaphragm
Psoas muscle
Adrenal gland
Kidney
Aorta

h
Pancreatic tail Left kidney

la
Anterior to the pancreas
Pancreatic head 1st part of the duodenum
Pylorus
Gastroduodenal artery
Sa SMA and SMV(uncinate process)

Pancreatic body Stomach


Duodenojejunal flexure
C
Pancreatic tail Splenic hilum

Superior to the pancreas


R

Coeliac trunk and its branches common hepatic artery and splenic artery

Grooves of the head of the pancreas


2nd and 3rd part of the duodenum
M

Arterial supply

• Head: pancreaticoduodenal artery


• Rest: splenic artery

Venous drainage

• Head: superior mesenteric vein


• Body and tail: splenic vein
Ampulla of Vater

• Merge of pancreatic duct and common bile duct


• Is an important landmark, halfway along the second part of the duodenum, that marks the
anatomical transition from foregut to midgut (also the site of transition between regions
supplied by coeliac trunk and SMA).

h
la
Sa
C
R
M
Question 199 of 235

An 18 year old man is undergoing an orchidectomy via a scrotal approach. The surgeons mobilise
the spermatic cord. From which of the following is the outermost layer of this structure derived?

Internal oblique aponeurosis

External oblique aponeurosis

h
Transversalis fascia

la
Rectus sheath

Campers fascia
Sa
The outermost covering of the spermatic cord is derived from the external oblique aponeurosis.This
layer is added as the cord passes through the superficial inguinal ring.
Please rate this question:
C
Discuss and give feedback
Next question

Scrotal and testicular anatomy


R

Spermatic cord
Formed by the vas deferens and is covered by the following structures:
Layer Origin
M

Internal spermatic fascia Transversalis fascia

Cremasteric fascia From the fascial coverings of internal oblique

External spermatic fascia External oblique aponeurosis

Contents of the cord


Vas deferens Transmits sperm and accessory gland secretions

Testicular artery Branch of abdominal aorta supplies testis and


epididymis

Artery of vas deferens Arises from inferior vesical artery

Cremasteric artery Arises from inferior epigastric artery

h
Pampiniform plexus Venous plexus, drains into right or left testicular vein

la
Sympathetic nerve fibres Lie on arteries, the parasympathetic fibres lie on the
Sa vas

Genital branch of the genitofemoral Supplies cremaster


nerve

Lymphatic vessels Drain to lumbar and para-aortic nodes


C
Scrotum
R

• Composed of skin and closely attached dartos fascia.


• Arterial supply from the anterior and posterior scrotal arteries
• Lymphatic drainage to the inguinal lymph nodes
• Parietal layer of the tunica vaginalis is the innermost layer
M

Testes

• The testes are surrounded by the tunica vaginalis (closed peritoneal sac). The parietal layer
of the tunica vaginalis adjacent to the internal spermatic fascia.
• The testicular arteries arise from the aorta immediately inferiorly to the renal arteries.
• The pampiniform plexus drains into the testicular veins, the left drains into the left renal vein
and the right into the inferior vena cava.
• Lymphatic drainage is to the para-aortic nodes.
Question 200 of 235

A 53 year old male presents with a carcinoma of the transverse colon. Which of the following
structures should be ligated close to their origin to maximise clearance of the tumour?

Superior mesenteric artery

h
Inferior mesenteric artery

la
Middle colic artery

Ileo-colic artery
Sa
Superior rectal artery

The middle colic artery supplies the transverse colon and requires high ligation during cancer
C
resections. It is a branch of the superior mesenteric artery.
Please rate this question:

Discuss and give feedback


R

Next question

Transverse colon
M

• The right colon undergoes a sharp turn at the level of the hepatic flexure to become the
transverse colon.
• At this point it also becomes intraperitoneal.
• It is connected to the inferior border of the pancreas by the transverse mesocolon.
• The greater omentum is attached to the superior aspect of the transverse colon from which it
can easily be separated. The mesentery contains the middle colic artery and vein. The
greater omentum remains attached to the transverse colon up to the splenic flexure. At this
point the colon undergoes another sharp turn.

Relations
Liver and gall-bladder, the greater curvature of the stomach, and the lower end of the
Superior
spleen

Inferior Small intestine

Anterior Greater omentum

From right to left with the descending portion of the duodenum, the head of the
Posterior
pancreas, convolutions of the jejunum and ileum, spleen

h
la
Sa
C
R
M
Question 201 of 235

Which of the following structures does not lie posterior to the right kidney?

Psoas major

12th rib

h
Quadratus lumborum

la
Iliolumbar ligament

10th rib
Sa
The 8th and10th ribs lie more superiorly. The 12th rib is a closer relation posteriorly.
Quadratus lumborum runs between the posterior part of the iliac crest, iliolumbar ligament and the
transverse processes of the lower lumbar vertebrae to the medial part of the lower border of the last
rib and transverse process of the upper lumbar vertebrae. In these last two locations it is posterior to
C
the kidney.
Please rate this question:
R

Discuss and give feedback

Next question
M

Renal anatomy

Each kidney is about 11cm long, 5cm wide and 3cm thick. They are located in a deep gutter
alongside the projecting vertebral bodies, on the anterior surface of psoas major. In most cases the
left kidney lies approximately 1.5cm higher than the right. The upper pole of both kidneys
approximates with the 11th rib (beware pneumothorax during nephrectomy). On the left hand side
the hilum is located at the L1 vertebral level and the right kidney at level L1-2. The lower border of
the kidneys is usually alongside L3.

The table below shows the anatomical relations of the kidneys:


Relations

Relations Right Kidney Left Kidney

Posterior Quadratus lumborum, diaphragm, psoas Quadratus lumborum, diaphragm, psoas


major, transversus abdominis major, transversus abdominis

Anterior Hepatic flexure of colon Stomach, Pancreatic tail

h
Superior Liver, adrenal gland Spleen, adrenal gland

la
Fascial covering
Each kidney and suprarenal gland is enclosed within a common layer of investing fascia, derived
from the transversalis fascia. It is divided into anterior and posterior layers (Gerotas fascia).

Renal structure
Sa
Kidneys are surrounded by an outer cortex and an inner medulla which usually contains between 6
and 10 pyramidal structures. The papilla marks the innermost apex of these. They terminate at the
renal pelvis, into the ureter.
Lying in a hollow within the kidney is the renal sinus. This contains:
1. Branches of the renal artery
2. Tributaries of the renal vein
C
3. Major and minor calyces's
4. Fat

Structures at the renal hilum


R

The renal vein lies most anteriorly, then renal artery (it is an end artery) and the ureter lies most
posterior.
Next question
M
Question 202 of 235

A 56 year old man is undergoing a radical nephrectomy via a posterior approach. Which of the
following structures is most likely to be encountered during the operative approach?

8th rib

h
10th rib

la
6th rib

12th rib
Sa
9th rib

The 11th and 12th ribs lie posterior to the kidneys and may be encountered during a posterior
C
approach. A pneumothorax is a recognised complication of this type of surgery.
Please rate this question:
R

Discuss and give feedback

Next question
M

Renal anatomy

Each kidney is about 11cm long, 5cm wide and 3cm thick. They are located in a deep gutter
alongside the projecting vertebral bodies, on the anterior surface of psoas major. In most cases the
left kidney lies approximately 1.5cm higher than the right. The upper pole of both kidneys
approximates with the 11th rib (beware pneumothorax during nephrectomy). On the left hand side
the hilum is located at the L1 vertebral level and the right kidney at level L1-2. The lower border of
the kidneys is usually alongside L3.

The table below shows the anatomical relations of the kidneys:


Relations

Relations Right Kidney Left Kidney

Posterior Quadratus lumborum, diaphragm, psoas Quadratus lumborum, diaphragm, psoas


major, transversus abdominis major, transversus abdominis

Anterior Hepatic flexure of colon Stomach, Pancreatic tail

h
Superior Liver, adrenal gland Spleen, adrenal gland

la
Fascial covering
Each kidney and suprarenal gland is enclosed within a common layer of investing fascia, derived
from the transversalis fascia. It is divided into anterior and posterior layers (Gerotas fascia).

Renal structure
Sa
Kidneys are surrounded by an outer cortex and an inner medulla which usually contains between 6
and 10 pyramidal structures. The papilla marks the innermost apex of these. They terminate at the
renal pelvis, into the ureter.
Lying in a hollow within the kidney is the renal sinus. This contains:
1. Branches of the renal artery
2. Tributaries of the renal vein
C
3. Major and minor calyces's
4. Fat

Structures at the renal hilum


R

The renal vein lies most anteriorly, then renal artery (it is an end artery) and the ureter lies most
posterior.
Next question
M
Question 203 of 235

How many unpaired branches leave the abdominal aorta to supply the abdominal viscera?

One

h
Two

la
Three

Four
Sa
Five
C
There are three unpaired branches to the abdominal viscera. These include the coeliac axis, the
SMA and IMA. Branches to the adrenals, renal arteries and gonadal vessels are paired. The fourth
unpaired branch of the abdominal aorta, the median sacral artery, does not directly supply the
abdominal viscera.
Please rate this question:
R
M

Discuss and give feedback

Next question

Abdominal aortic branches

Branches Level Paired Type

Inferior phrenic T12 (Upper border) Yes Parietal


Branches Level Paired Type

Coeliac T12 No Visceral

Superior mesenteric L1 No Visceral

Middle suprarenal L1 Yes Visceral

h
Renal L1-L2 Yes Visceral

la
Gonadal L2 Yes Visceral

Lumbar L1-L4 Yes Parietal


Sa
Inferior mesenteric L3 No Visceral

Median sacral L4 No Parietal


C
Common iliac L4 Yes Terminal
R

Next question
M
Question 204 of 235

A 45 year old man presents with bilateral inguinal hernias. The surgical team decide to repair these
laparoscopically through an extraperitoneal approach. Through an infraumbilical incision the
surgeons displace the inferior aspect of the rectus abdominis muscle anteriorly and place a
prosthetic mesh into the area to repair the hernias. Which structure will lie posterior to the mesh?

h
Peritoneum

la
Internal oblique aponeurosis
Sa
External oblique aponeurosis

Posterior aspect of the rectus sheath

Bucks fascia
C
During a TEP repair of inguinal hernia the only structure to lie posterior to the mesh is peritoneum.
The question is really only asking which structure lies posterior to the rectus abdominis muscle.
R

Since this region is below the arcuate line, the transversalis fascia and peritoneum lie posterior to it.
Bucks fascia lies in the penis.
Please rate this question:
M

Discuss and give feedback

Next question

Rectus abdominis muscle

The rectus sheath is formed by the aponeuroses of the lateral abdominal wall muscles. The rectus
sheath has a composition that varies according to anatomical level.
1. Above the costal margin the anterior sheath is composed of external oblique aponeurosis, the
costal cartilages are posterior to it.
2. From the costal margin to the arcuate line, the anterior rectus sheath is composed of external
oblique aponeurosis and the anterior part of the internal oblique aponeurosis. The posterior part of
the internal oblique aponeurosis and transversus abdominis form the posterior rectus sheath.
3. Below the arcuate line the aponeuroses of all the abdominal muscles lie in anterior aspect of the
rectus sheath. Posteriorly lies the transversalis fascia and peritoneum.

The arcuate line is the point at which the inferior epigastric vessels enter the rectus sheath.
Next question

h
la
Sa
C
R
M
Question 205 of 235

During embryological development, which of the following represent the correct origin of the
pancreas?

Ventral and dorsal endodermal outgrowths of the duodenum

Ventral and dorsal outgrowths of mesenchymal tissue from the posterior abdominal wall

h
Ventral and dorsal outgrowths of the vitellointestinal duct

la
Ventral and dorsal biliary tract diverticulae

Buds from the inferior aspect of the caudate lobe


Sa
The pancreas develops from a ventral and dorsal endodermal outgrowth of the duodenum. The
ventral arises close to, or in common with the hepatic diverticulum, and the larger, dorsal outgrowth
arises slightly cranial to the ventral extending into the mesoduodenum and mesogastrium. When the
buds eventually fuse the duct of the ventral rudiment becomes the main pancreatic duct.
Please rate this question:
C
Discuss and give feedback
Next question
R

Pancreas

The pancreas is a retroperitoneal organ and lies posterior to the stomach. It may be accessed
surgically by dividing the peritoneal reflection that connects the greater omentum to the transverse
M

colon. The pancreatic head sits in the curvature of the duodenum. Its tail lies close to the hilum of
the spleen, a site of potential injury during splenectomy.

Relations
Posterior to the pancreas
Pancreatic head Inferior vena cava
Common bile duct
Right and left renal veins
Superior mesenteric vein and artery
Pancreatic neck Superior mesenteric vein, portal vein

Pancreatic body- Left renal vein


Crus of diaphragm
Psoas muscle
Adrenal gland
Kidney
Aorta

h
Pancreatic tail Left kidney

la
Anterior to the pancreas
Pancreatic head 1st part of the duodenum
Pylorus
Gastroduodenal artery
Sa SMA and SMV(uncinate process)

Pancreatic body Stomach


Duodenojejunal flexure
C
Pancreatic tail Splenic hilum

Superior to the pancreas


R

Coeliac trunk and its branches common hepatic artery and splenic artery

Grooves of the head of the pancreas


2nd and 3rd part of the duodenum
M

Arterial supply

• Head: pancreaticoduodenal artery


• Rest: splenic artery

Venous drainage

• Head: superior mesenteric vein


• Body and tail: splenic vein
Ampulla of Vater

• Merge of pancreatic duct and common bile duct


• Is an important landmark, halfway along the second part of the duodenum, that marks the
anatomical transition from foregut to midgut (also the site of transition between regions
supplied by coeliac trunk and SMA).

h
la
Sa
C
Image sourced from Wikipedia

Next question
R
M
Question 206 of 235

A 63 year old man is due to undergo a splenectomy. Which splenic structure lies most posteriorly?

Gastrosplenic ligament

Splenic vein

h
Splenic artery

la
Splenic notch

Lienorenal ligament
Sa
The lienorenal ligament lies most posteriorly. The antero-lateral connection is via the phrenicocolic
ligament. Anteriorly the gastro splenic ligament. These structures condense around the vessels at
the splenic hilum.
Please rate this question:
C
Discuss and give feedback
Next question
R

Splenic anatomy

The spleen is the largest lymphoid organ in the body. It is an intraperitoneal organ, the peritoneal
attachments condense at the hilum where the vessels enter the spleen. Its blood supply is from the
M

splenic artery (derived from the coeliac axis) and the splenic vein (which is joined by the IMV and
unites with the SMV).

• Embryology: derived from mesenchymal tissue


• Shape: clenched fist
• Position: below 9th-12th ribs
• Weight: 75-150g

Relations
• Superiorly- diaphragm
• Anteriorly- gastric impression
• Posteriorly- kidney
• Inferiorly- colon
• Hilum: tail of pancreas and splenic vessels
• Forms apex of lesser sac (containing short gastric vessels)

Next question

h
la
Sa
C
R
M
Question 206 of 235

A 63 year old man is due to undergo a splenectomy. Which splenic structure lies most posteriorly?

Gastrosplenic ligament

Splenic vein

h
Splenic artery

la
Splenic notch

Lienorenal ligament
Sa
The lienorenal ligament lies most posteriorly. The antero-lateral connection is via the phrenicocolic
ligament. Anteriorly the gastro splenic ligament. These structures condense around the vessels at
the splenic hilum.
Please rate this question:
C
Discuss and give feedback
Next question

Splenic anatomy
R

The spleen is the largest lymphoid organ in the body. It is an intraperitoneal organ, the peritoneal
attachments condense at the hilum where the vessels enter the spleen. Its blood supply is from the
splenic artery (derived from the coeliac axis) and the splenic vein (which is joined by the IMV and
unites with the SMV).
M

• Embryology: derived from mesenchymal tissue


• Shape: clenched fist
• Position: below 9th-12th ribs
• Weight: 75-150g

Relations
• Superiorly- diaphragm
• Anteriorly- gastric impression
• Posteriorly- kidney
• Inferiorly- colon
• Hilum: tail of pancreas and splenic vessels
• Forms apex of lesser sac (containing short gastric vessels)

Next question

h
la
Sa
C
R
M
Question 207 of 235

Which of the following statements about the spleen is false?

The hilum contains the splenic vessels.

The spleen is derived from endodermal tissue.

The white pulp has immune function.

h
The colon lies inferiorly.

la
Weighs 150g.

1,3,5,7,9,11 (odd numbers up to 11)


Sa
The spleen is: 1 inch thick, 3 inches wide, 5 inches long, weighs 7oz (150-200g), lies between the
9th and 11th ribs

Most of the gut is derived endodermally except for the spleen which is from mesenchymal tissue.
Please rate this question:
C
Discuss and give feedback
Next question

Spleen
R

The spleen is located in the left upper quadrant of the abdomen and its size can vary depending
upon the amount of blood it contains. The typical adult spleen is 12.5cm long and 7.5cm wide. The
usual weight of the adult spleen is 150g.
M

The exact position of the spleen can vary with respiratory activity, posture and the state of
surrounding viscera. It usually lies obliquely with its long axis aligned to the 9th, 10th and 11th ribs. It
is separated from these ribs by both diaphragm and pleural cavity. The normal spleen is not
palpable.

The shape of the spleen is influenced by the state of the colon and stomach. Gastric distension will
cause the spleen to resemble the shape of an orange segment. Colonic distension will cause it to
become more tetrahedral.

The spleen is almost entirely covered by peritoneum, which adheres firmly to its capsule. Recesses
of the greater sac separate it from the stomach and kidney. It develops from the upper dorsal
mesogastrium, remaining connected to the posterior abdominal wall and stomach by two folds of
peritoneum; the lienorenal ligament and gastrosplenic ligament. The lienorenal ligament is derived
from peritoneum where the wall of the general peritoneum meets the omental bursa between the left
kidney and spleen; the splenic vessels lie in its layers. The gastrosplenic ligament also has two
layers, formed by the meeting of the walls of the greater sac and omental bursa between spleen and
stomach, the short gastric and left gastroepiploic branches of the splenic artery pass in its layers.
Laterally, the spleen is in contact with the phrenicocolic ligament.

Relations
Superiorly Diaphragm

Anteriorly Gastric impression

Posteriorly Kidney

h
Inferiorly Colon

la
Tail of pancreas and splenic vessels (splenic artery divides here, branches pass to the
Hilum
white pulp transporting plasma)

Contents
White Immune function. Contains central trabecular artery. The germinal centres are supplied
pulp

Red pulp
Sa
by arterioles called penicilliary radicles.

Filters abnormal red blood cells.

Function
C
• Filtration of abnormal blood cells and foreign bodies such as bacteria.
• Immunity: IgM. Production of properdin, and tuftsin which help target fungi and bacteria for
phagocytosis.
• Haematopoiesis: up to 5th month gestation or in haematological disorders.
• Pooling: storage of 40% platelets.
R

• Iron reutilisation
• Storage monocytes

Disorders of the spleen


M

Massive splenomegaly

• Myelofibrosis
• Chronic myeloid leukaemia
• Visceral leishmaniasis (kala-azar)
• Malaria
• Gaucher's syndrome

Other causes (as above plus)


• Portal hypertension e.g. secondary to cirrhosis
• Lymphoproliferative disease e.g. CLL, Hodgkin's
• Haemolytic anaemia
• Infection: hepatitis, glandular fever
• Infective endocarditis
• Sickle-cell*, thalassaemia
• Rheumatoid arthritis (Felty's syndrome)

*the majority of adult patients with sickle-cell will have an atrophied spleen due to repeated infarction
Next question

h
la
Sa
C
R
M
Question 208 of 235

Which of the following structures is not at the level of the transpyloric plane?

Hilum left kidney

Superior mesenteric artery

h
Fundus of the gallbladder

la
Cardioesophageal junction

Root of transverse mesocolon


Sa
Cardiooesophageal junction level = T11

A knowledge of this anatomic level is commonly tested.


The oesophagus extends from C6 (the lower border of the cricoid cartilage) to T11 at the
cardioesophageal junction. Note that in the neonate the oesophagus extends from C4 or C5 to T9.
Please rate this question:
C
Discuss and give feedback
Next question
R

Levels

Transpyloric plane
Level of the body of L1
M

• Pylorus stomach
• Left kidney hilum (L1- left one!)
• Fundus of the gallbladder
• Neck of pancreas
• Duodenojejunal flexure
• Superior mesenteric artery
• Portal vein
• Left and right colic flexure
• Root of the transverse mesocolon
• 2nd part of the duodenum
• Upper part of conus medullaris
• Spleen

Can be identified by asking the supine patient to sit up without using their arms. The plane is located
where the lateral border of the rectus muscle crosses the costal margin.

Anatomical planes
Subcostal plane Lowest margin of 10th costal cartilage

Intercristal plane Level of body L4 (highest point of iliac crest)

h
Intertubercular plane Level of body L5

la
Common level landmarks
Inferior mesenteric artery L3
Sa
Bifurcation of aorta into common iliac arteries L4

Formation of IVC L5 (union of common iliac veins)


C
Diaphragm apertures • Vena cava T8
• Oesophagus T10
• Aortic hiatus T12
R

Next question
M
Question 209 of 235

A 45 year old man is undergoing a left hemicolectomy. As the surgeons mobilise the left colon they
identify a tubular structure lying at the inferior aspect of psoas major. What is it most likely to be?

Left ureter

h
Left common iliac vein

la
Left common iliac artery

Left external iliac artery


Sa
Left external iliac vein

The left ureter lies posterior to the left colon. The sigmoid colon and upper rectum may be more
closely related to the iliac vessels. These are not typically found above L4.
Please rate this question:
C
Discuss and give feedback
Next question
R

Ureter

• 25-35 cm long
• Muscular tube lined by transitional epithelium
M

• Surrounded by thick muscular coat. Becomes 3 muscular layers as it crosses the bony pelvis
• Retroperitoneal structure overlying transverse processes L2-L5
• Lies anterior to bifurcation of iliac vessels
• Blood supply is segmental; renal artery, aortic branches, gonadal branches, common iliac
and internal iliac
• Lies beneath the uterine artery

Next question
Question 210 of 235

A 42 year old woman is due to undergo a left nephroureterectomy for a transitional cell carcinoma
involving the ureter. Which of the following structures is not related to the left ureter?

Round ligament of the uterus

Internal iliac artery

h
Ovarian artery

la
Peritoneum

Sigmoid mesocolon
Sa
The ureter is not related to the round ligament of the uterus, it is related to the broad ligament and is
within 1.5cm of the supravaginal part of the cervix.
Please rate this question:
C
Discuss and give feedback
Next question

Ureter
R

• 25-35 cm long
• Muscular tube lined by transitional epithelium
• Surrounded by thick muscular coat. Becomes 3 muscular layers as it crosses the bony pelvis
• Retroperitoneal structure overlying transverse processes L2-L5
M

• Lies anterior to bifurcation of iliac vessels


• Blood supply is segmental; renal artery, aortic branches, gonadal branches, common iliac
and internal iliac
• Lies beneath the uterine artery
Question 212 of 235

A 28 year old man is undergoing an appendicectomy. The external oblique aponeurosis is incised
and the underlying muscle split in the line of its fibres. At the medial edge of the wound is a tough

h
fibrous structure. Entry to this structure will most likely encounter which of the following?

Internal oblique

la
Rectus abdominis

Transversus abdominis
Sa
Linea alba

Peritoneum
C
This structure will be the rectus sheath and when entered the rectus abdominis muscle will be
encountered.
R

Please rate this question:

Discuss and give feedback


Next question
M

Abdominal incisions

Midline incision • Commonest approach to the abdomen


• Structures divided: linea alba, transversalis fascia, extraperitoneal fat,
peritoneum (avoid falciform ligament above the umbilicus)
• Bladder can be accessed via an extraperitoneal approach through the
space of Retzius
Paramedian • Parallel to the midline (about 3-4cm)
incision • Structures divided/retracted: anterior rectus sheath, rectus (retracted),
posterior rectus sheath, transversalis fascia, extraperitoneal fat,
peritoneum
• Incision is closed in layers

Battle • Similar location to paramedian but rectus displaced medially (and thus
denervated)
• Now seldom used

h
Kocher's Incision under right subcostal margin e.g. Cholecystectomy (open)

la
Lanz Incision in right iliac fossa e.g. Appendicectomy

Gridiron Oblique incision centered over McBurneys point- usually appendicectomy


(less cosmetically acceptable than Lanz
Sa
Gable Rooftop incision

Pfannenstiel's Transverse supra pubic, primarily used to access pelvic organs

McEvedy's Groin incision e.g. Emergency repair strangulated femoral hernia

Rutherford Extraperitoneal approach to left or right lower quadrants. Gives excellent


Morrison access to iliac vessels and is the approach of choice for first time renal
C
transplantation.
R
M
Question 213 of 235

A 35 year old man presents to the surgical clinic with a suspected direct inguinal hernia. These will pass
through Hesselbach's triangle. Which of the following forms the medial edge of this structure?

External oblique aponeurosis

h
Inferior epigastric artery

la
Rectus abdominis muscle
Sa
Inferior epigastric vein

Obturator nerve
C
Direct inguinal hernias pass through Hesselbachs triangle (although this is of minimal clinical
significance!). Its medial boundary is the rectus muscle.
Please rate this question:
R

Discuss and give feedback


M

Next question

Hesselbach's triangle

Direct hernias pass through Hesselbachs triangle.

Superolaterally Epigastric vessels


Medially Lateral edge of rectus muscle

Inferiorly Inguinal ligament

The boundaries of Hesselbachs triangle are commonly tested and illustrated below

h
la
Sa
C
R
M
Question 214 of 235

Where are accessory spleens not found?

Gonads

h
Tail of pancreas

la
Greater omentum

Splenorenal ligament
Sa
Ureter

Accessory spleens

- 10% population
C
- 1 cm size
- locations: hilum of the spleen, tail of the pancreas, along the splenic vessels, in the gastrosplenic
ligament, the splenorenal ligament, the walls of the stomach or intestines, the greater omentum, the
mesentery, the gonads
Please rate this question:
R

Discuss and give feedback


Next question
M

Spleen

The spleen is located in the left upper quadrant of the abdomen and its size can vary depending
upon the amount of blood it contains. The typical adult spleen is 12.5cm long and 7.5cm wide. The
usual weight of the adult spleen is 150g.
The exact position of the spleen can vary with respiratory activity, posture and the state of
surrounding viscera. It usually lies obliquely with its long axis aligned to the 9th, 10th and 11th ribs. It
is separated from these ribs by both diaphragm and pleural cavity. The normal spleen is not
palpable.

The shape of the spleen is influenced by the state of the colon and stomach. Gastric distension will
cause the spleen to resemble the shape of an orange segment. Colonic distension will cause it to
become more tetrahedral.

The spleen is almost entirely covered by peritoneum, which adheres firmly to its capsule. Recesses
of the greater sac separate it from the stomach and kidney. It develops from the upper dorsal
mesogastrium, remaining connected to the posterior abdominal wall and stomach by two folds of
peritoneum; the lienorenal ligament and gastrosplenic ligament. The lienorenal ligament is derived
from peritoneum where the wall of the general peritoneum meets the omental bursa between the left
kidney and spleen; the splenic vessels lie in its layers. The gastrosplenic ligament also has two
layers, formed by the meeting of the walls of the greater sac and omental bursa between spleen and
stomach, the short gastric and left gastroepiploic branches of the splenic artery pass in its layers.
Laterally, the spleen is in contact with the phrenicocolic ligament.

h
Relations
Superiorly Diaphragm

la
Anteriorly Gastric impression

Posteriorly Kidney

Inferiorly Colon

Hilum
Sa
Tail of pancreas and splenic vessels (splenic artery divides here, branches pass to the
white pulp transporting plasma)

Contents
White Immune function. Contains central trabecular artery. The germinal centres are supplied
pulp by arterioles called penicilliary radicles.
C
Red pulp Filters abnormal red blood cells.

Function
R

• Filtration of abnormal blood cells and foreign bodies such as bacteria.


• Immunity: IgM. Production of properdin, and tuftsin which help target fungi and bacteria for
phagocytosis.
• Haematopoiesis: up to 5th month gestation or in haematological disorders.
M

• Pooling: storage of 40% platelets.


• Iron reutilisation
• Storage monocytes

Disorders of the spleen


Massive splenomegaly

• Myelofibrosis
• Chronic myeloid leukaemia
• Visceral leishmaniasis (kala-azar)
• Malaria
• Gaucher's syndrome

Other causes (as above plus)

• Portal hypertension e.g. secondary to cirrhosis


• Lymphoproliferative disease e.g. CLL, Hodgkin's
• Haemolytic anaemia
• Infection: hepatitis, glandular fever
• Infective endocarditis
• Sickle-cell*, thalassaemia

h
• Rheumatoid arthritis (Felty's syndrome)

la
*the majority of adult patients with sickle-cell will have an atrophied spleen due to repeated infarction
Next question

Sa
C
R
M
Question 216 of 235

A 56 year old man is undergoing an anterior resection for a carcinoma of the rectum. Which of the
structures below is least likely to be encountered during the mobilisation of the anterior rectum?

Denonvilliers' fascia

h
Middle sacral artery

Bladder

la
Rectovesical pouch

Seminal vesicles
Sa
With the exception of the middle sacral artery all of the other structures lie anterior to the rectum.
They may all be palpated during digital rectal examination.
Please rate this question:
C
Discuss and give feedback
Next question

Rectum
R

The rectum is approximately 12 cm long. It is a capacitance organ. It has both intra and
extraperitoneal components. The transition between the sigmoid colon is marked by the
disappearance of the tenia coli.The extra peritoneal rectum is surrounded by mesorectal fat that also
M

contains lymph nodes. This mesorectal fatty layer is removed surgically during rectal cancer surgery
(Total Mesorectal Excision). The fascial layers that surround the rectum are important clinical
landmarks, anteriorly lies the fascia of Denonvilliers. Posteriorly lies Waldeyers fascia.

Extra peritoneal rectum

• Posterior upper third


• Posterior and lateral middle third
• Whole lower third
Relations
Anteriorly (Males) Rectovesical pouch
Bladder
Prostate
Seminal vesicles

Anteriorly (Females) Recto-uterine pouch (Douglas)


Cervix
Vaginal wall

h
Posteriorly Sacrum

la
Coccyx
Middle sacral artery

Laterally Levator ani


Sa Coccygeus

Arterial supply
Superior rectal artery

Venous drainage
C
Superior rectal vein

Lymphatic drainage
R

• Mesorectal lymph nodes (superior to dentate line)


• Inguinal nodes (inferior to dentate line)

Next question
M
Question 219 of 235

The inferior aspect of the vagina drains to which of the following lymph node groups?

Superficial inguinal nodes

h
Internal iliac nodes

la
Para-aortic nodes

Obturator nodes

Meso-rectal nodes
Sa
Please rate this question:
C
Discuss and give feedback
R

Next question

Lymphatic drainage of the vagina


M

The lymph vessels from the superior aspect of the vagina join the internal and external iliac nodes,
those from the inferior aspect of the vagina drain to the superficial inguinal nodes.
Next question
h
Question 221 of 235

A 25 year old man is being catheterised, prior to a surgical procedure. As the catheter enters the

la
prostatic urethra which of the following changes will occur?

Resistance will increase significantly


Sa
Resistance will increase slightly

It will lie horizontally

Resistance will decrease


C
It will deviate laterally
R

The prostatic urethra is much wider than the membranous urethra and therefore resistance will
decrease. The prostatic urethra is inclined superiorly.
Please rate this question:
M

Discuss and give feedback


Next question

Prostate gland

The prostate gland is approximately the shape and size of a walnut and is located inferior to the
bladder. It is separated from the rectum by Denonvilliers fascia and its blood supply is derived from
the internal iliac vessels (via inferior vesical artery). The internal sphincter lies at the apex of the
gland and may be damaged during prostatic surgery, affected individuals may complain of
retrograde ejaculation.
Summary of prostate gland
Arterial supply Inferior vesical artery (from internal iliac)

Venous drainage Prostatic venous plexus (to paravertebral veins)

Lymphatic Internal iliac nodes


drainage

h
Innervation Inferior hypogastric plexus

la
Dimensions • Transverse diameter (4cm)
• AP diameter (2cm)

Sa Height (3cm)

Lobes • Posterior lobe: posterior to urethra


• Median lobe: posterior to urethra, in between ejaculatory ducts
• Lateral lobes x 2
• Isthmus


C
Zones Peripheral zone: subcapsular portion of posterior prostate. Most
prostate cancers are here
• Central zone
• Transition zone
• Stroma
R

Relations
M

Pubic symphysis
Anterior Prostatic venous plexus

Posterior Denonvilliers fascia


Rectum
Ejaculatory ducts
Pubic symphysis
Anterior Prostatic venous plexus

Lateral Venous plexus (lies on prostate)


Levator ani (immediately below the puboprostatic ligaments)

h
la
Sa
Image sourced from Wikipedia
C
Next question
R
M
Question 223 of 235

A 78 year old man develops a carcinoma of the scrotum. To which of the following lymph node
groups may the tumour initially metastasise?

Para aortic

Obturator

h
Inguinal

la
Meso rectal

None of the above


Sa
The scrotum is drained by the inguinal nodes.
Please rate this question:

Discuss and give feedback


C
Next question

Scrotal and testicular anatomy


R

Spermatic cord
Formed by the vas deferens and is covered by the following structures:
Layer Origin
M

Internal spermatic fascia Transversalis fascia

Cremasteric fascia From the fascial coverings of internal oblique

External spermatic fascia External oblique aponeurosis

Contents of the cord


Vas deferens Transmits sperm and accessory gland secretions

Testicular artery Branch of abdominal aorta supplies testis and


epididymis

Artery of vas deferens Arises from inferior vesical artery

Cremasteric artery Arises from inferior epigastric artery

h
Pampiniform plexus Venous plexus, drains into right or left testicular vein

la
Sympathetic nerve fibres Lie on arteries, the parasympathetic fibres lie on the
Sa vas

Genital branch of the genitofemoral Supplies cremaster


nerve

Lymphatic vessels Drain to lumbar and para-aortic nodes


C
Scrotum
R

• Composed of skin and closely attached dartos fascia.


• Arterial supply from the anterior and posterior scrotal arteries
• Lymphatic drainage to the inguinal lymph nodes
• Parietal layer of the tunica vaginalis is the innermost layer
M

Testes

• The testes are surrounded by the tunica vaginalis (closed peritoneal sac). The parietal layer
of the tunica vaginalis adjacent to the internal spermatic fascia.
• The testicular arteries arise from the aorta immediately inferiorly to the renal arteries.
• The pampiniform plexus drains into the testicular veins, the left drains into the left renal vein
and the right into the inferior vena cava.
• Lymphatic drainage is to the para-aortic nodes.

Next question
Question 226 of 235

An 22 year old soldier is shot in the abdomen and amongst his various injuries is a major disruption
to the abdominal aorta. There is torrential haemorrhage and the surgeons decide to control the aorta
by placement of a vascular clamp immediately inferior to the diaphragm. Which of the following
vessels may be injured in this maneouvre?

Inferior phrenic arteries

h
Superior phrenic arteries

la
Splenic artery

Renal arteries
Sa
Superior mesenteric artery

As the first branches of the abdominal aorta the inferior phrenic arteries are at greatest risk. The
superior phrenic arteries lie in the thorax. The potential space at the level of the diaphragmatic hiatus
C
is a potentially useful site for aortic occlusion. However, leaving the clamp applied for more than
about 10 -15 minutes usually leads to poor outcomes.
Please rate this question:
R

Discuss and give feedback


M

Next question

Abdominal aorta

Abdominal aortic topography

Origin T12

Termination L4
Posterior relations L1-L4 Vertebral bodies

Anterior relations Lesser omentum


Liver
Left renal vein
Inferior mesenteric vein
Third part of duodenum
Pancreas
Parietal peritoneum
Peritoneal cavity

h
Right lateral relations Right crus of the diaphragm

la
Cisterna chyli
IVC (becomes posterior distally)

Left lateral relations


Sa
4th part of duodenum
Duodenal-jejunal flexure
Left sympathetic trunk

The abdominal aorta


C
R
M
h
la
Sa
Image sourced from Wikipedia

Next question
C
R
M
Question 227 of 235

Which of the following statements relating to the gallbladder is untrue?

The fundus is usually intra peritoneal

Arterial supply is from the cystic artery

h
The cystic artery is usually located in Calots triangle

la
Calots triangle may rarely contain an aberrant hepatic artery
Sa
Cholecystokinin causes relaxation of the gallbladder

CCK causes gallbladder contraction.


Please rate this question:
C
Discuss and give feedback
Next question

Gallbladder
R

• Fibromuscular sac with capacity of 50ml


• Columnar epithelium
M

Relations of the gallbladder


Anterior Liver

Posterior • Covered by peritoneum


• Transverse colon
• 1st part of the duodenum

Laterally Right lobe of liver


Medially Quadrate lobe of liver

Arterial supply
Cystic artery (branch of Right hepatic artery)

Venous drainage
Directly to the liver

Nerve supply
Sympathetic- mid thoracic spinal cord, Parasympathetic- anterior vagal trunk

Common bile duct

h
Origin Confluence of cystic and common hepatic ducts

la
Relations at • Medially - Hepatic artery
origin •
Sa Posteriorly- Portal vein

Relations distally • Duodenum - anteriorly


• Pancreas - medially and laterally
• Right renal vein - posteriorly

Arterial supply Branches of hepatic artery and retroduodenal branches of gastroduodenal


artery
C
Hepatobiliary triangle
R

Medially Common hepatic duct

Inferiorly Cystic duct


M

Superiorly Inferior edge of liver

Contents Cystic artery

Relations of the gallbladder


h
la
© Image provided by the University of Sheffield

Next question

Sa
C
R
M
Question 228 of 235

Which of the following nerves is the primary source of innervation to the anterior scrotal skin?

Iliohypogastric nerve

Pudendal nerve

h
Ilioinguinal nerve

la
Femoral branch of the genitofemoral nerve

Obturator nerve
Sa
The pudendal nerve may innervate the posterior skin of the scrotum. The anterior innervation of the
scrotum is primarily provided by the ilioinguinal nerve. The genital branch of the genitofemoral nerve
provides a smaller contribution.
Please rate this question:
C
R

Discuss and give feedback

Next question

Scrotal sensation
M

The scrotum is innervated by the ilioinguinal nerve and the pudendal nerve. The ilioinguinal nerve
arises from L1 and pierces the internal oblique muscle. It eventually passes through the superficial
inguinal ring to innervate the anterior skin of the scrotum.

The pudendal nerve is the principal nerve of the perineum. It arises in the pelvis from 3 nerve roots.
It passes through both greater and lesser sciatic foramina to enter the perineal region. The perineal
branches pass anteromedially and divide into posterior scrotal branches. The posterior scrotal
branches pass superficially to supply the skin and fascia of the perineum. It cross communicates
with the inferior rectal nerve.
Next question
Question 229 of 235

The transversalis fascia contributes to which of the following?

Pectineal ligament

Deep inguinal ring

h
Cremaster muscle and fascia

la
Inguinal ligament

External spermatic fascia


Sa
The internal spermatic fascia (derived from transversalis fascia) invests:
Ductus deferens
Testicular vessels

The principal outpouching of the transversalis fascia is the internal spermatic fascia. The mouth of
the outpouching is the deep inguinal ring.
Please rate this question:
C
Discuss and give feedback
Next question
R

Abdominal wall

The 2 main muscles of the abdominal wall are the rectus abdominis (anterior) and the quadratus
lumborum (posterior).
M

The remaining abdominal wall consists of 3 muscular layers. Each muscle passes from the lateral
aspect of the quadratus lumborum posteriorly to the lateral margin of the rectus sheath anteriorly.
Each layer is muscular posterolaterally and aponeurotic anteriorly.
Image sourced from Wikipedia

Muscles of abdominal wall


External • Lies most superficially

h
oblique • Originates from 5th to 12th ribs
• Inserts into the anterior half of the outer aspect of the iliac crest, linea
alba and pubic tubercle
• More medially and superiorly to the arcuate line, the aponeurotic layer

la
overlaps the rectus abdominis muscle
• The lower border forms the inguinal ligament
• The triangular expansion of the medial end of the inguinal ligament is
the lacunar ligament.

Internal •
Sa
Arises from the thoracolumbar fascia, the anterior 2/3 of the iliac crest
oblique and the lateral 2/3 of the inguinal ligament
• The muscle sweeps upwards to insert into the cartilages of the lower 3
ribs
• The lower fibres form an aponeurosis that runs from the tenth costal
cartilage to the body of the pubis
C
• At its lowermost aspect it joins the fibres of the aponeurosis of
transversus abdominis to form the conjoint tendon.
R

Transversus • Innermost muscle


abdominis • Arises from the inner aspect of the costal cartilages of the lower 6 ribs ,
from the anterior 2/3 of the iliac crest and lateral 1/3 of the inguinal
ligament
M

• Its fibres run horizontally around the abdominal wall ending in an


aponeurosis. The upper part runs posterior to the rectus abdominis.
Lower down the fibres run anteriorly only.
• The rectus abdominis lies medially; running from the pubic crest and
symphysis to insert into the xiphoid process and 5th, 6th and 7th costal
cartilages. The muscles lies in a aponeurosis as described above.
• Nerve supply: anterior primary rami of T7-12

Surgical notes
During abdominal surgery it is usually necessary to divide either the muscles or their aponeuroses.
During a midline laparotomy it is desirable to divide the aponeurosis. This will leave the rectus
sheath intact above the arcuate line and the muscles intact below it. Straying off the midline will
often lead to damage to the rectus muscles, particularly below the arcuate line where they may often
be in close proximity to each other.

h
la
Sa
C
R
M
Question 231 of 235

A 73 year old lady is admitted with right iliac fossa pain. A plain abdominal x-ray is taken and the
caecal diameter measured. Which of the following caecal diameters is pathological?

4cm

h
5cm

6cm

la
7cm

10cm
Sa
8 cm is still within normal limits. However, caecal diameters of 9 and 10 are pathological and should
prompt further investigation.
Please rate this question:
C
Discuss and give feedback
Next question

Right colon
R

Ileocaecal valve
M

• Entry point of the terminal ileum to the caecum


• An important colonoscopic landmark
• The ileocaecal valve is not always competent and this may allow partial decompression of an
obstructed colon

Appendix

• At the base of the caecum the taenia coalesce to mark the base of the appendix
• This is a reliable way of locating the appendix surgically and is a constant landmark
• The appendix has a small mesentery (the mesoappendix) and in this runs the appendiceal
artery, a branch of the ileocolic artery.

The posterior aspect of the right colon is extra peritoneal and the anterior aspect intraperitoneal.

Relations

• Posterior

h
Iliacus, Iliolumbar ligament, Quadratus lumborum, Transverse abdominis, Diaphragm at the tip of the
last rib; Lateral cutaneous, ilioinguinal, and iliohypogastric nerves; the iliac branches of the iliolumbar
vessels, the fourth lumbar artery, gonadal vessels, ureter and the right kidney.

la
• Superior

Right kidney which is embedded in the perinephric fat


Sa
• Medial

Mesentery which contains the ileocolic artery that supplies the right colon and terminal ileum. A
further branch , the right colic artery, also contributes to supply the hepatic flexure and proximal
transverse colon. Medially these pass through the mesentery to join the SMA. This occurs near to
C
the head of the pancreas and care has to be taken when ligating the ileocolic artery near to its origin
in cancer cases for fear of impinging on the SMA.

- Anterior
Coils of small intestine, the right edge of the greater omentum, and the anterior abdominal wall.
R

Nerve supply
M

• Parasympathetic fibres of the vagus nerve (CN X)

Arterial supply

• Ileocolic artery and right colic artery, both branches of the SMA. While the ileocolic artery is
almost always present, the right colic can be absent in 5-15% of individuals.
Question 233 of 235

Which of the following options in relation to the liver is true?

Ligamentum venosum is an anterior relation of the liver

The portal triad comprises the hepatic artery, hepatic vein and tributary of the bile duct

h
The liver is completely covered by peritoneum

la
There are no nerves within the porta hepatis
Sa
The caudate lobe is superior to the porta hepatis

'VC goes with VC'

The ligamentun Venosum and Caudate is on same side as Vena Cava [posterior].

Ligamentum venosum is posterior to the liver. The portal triad contains the portal vein rather than the
C
hepatic vein. There is the 'bare area of the liver' created by a void due to the coronary ligament
layers being widely separated. There are sympathetic and parasympathetic nerves in the porta
hepatis.
Please rate this question:
R

Discuss and give feedback


Next question
M

Liver

Structure of the liver


Right lobe • Supplied by right hepatic artery
• Contains Couinaud segments V to VIII (-/+Sg I)

Left lobe • Supplied by the left hepatic artery


• Contains Couinaud segments II to IV (+/- Sg1)
Quadrate lobe • Part of the right lobe anatomically, functionally is part of the left
• Couinaud segment IV
• Porta hepatis lies behind
• On the right lies the gallbladder fossa
• On the left lies the fossa for the umbilical vein

Caudate lobe • Supplied by both right and left hepatic arteries


• Couinaud segment I
• Lies behind the plane of the porta hepatis
• Anterior and lateral to the inferior vena cava

h
• Bile from the caudate lobe drains into both right and left hepatic ducts

la
Detailed knowledge of Couinaud segments is not required for MRCS

• Between the liver lobules are portal canals which contain the portal triad: Hepatic Artery,
Portal Vein, tributary of Bile Duct.
Sa
Relations of the liver
Anterior Postero inferiorly
C
Diaphragm Oesophagus

Xiphoid process Stomach


R

Duodenum
M

Hepatic flexure of colon

Right kidney

Gallbladder
Inferior vena cava

Porta hepatis
Location Postero inferior surface, it joins nearly at right angles with the left sagittal fossa, and
separates the caudate lobe behind from the quadrate lobe in front

Transmits • Common hepatic duct


• Hepatic artery

h
• Portal vein
• Sympathetic and parasympathetic nerve fibres
• Lymphatic drainage of the liver (and nodes)

la
Ligaments
Falciform ligament • 2 layer fold peritoneum from the umbilicus to anterior liver surface
Sa•

Contains ligamentum teres (remnant umbilical vein)
On superior liver surface it splits into the coronary and left
triangular ligaments

Ligamentum teres Joins the left branch of the portal vein in the porta hepatis

Ligamentum Remnant of ductus venosus


C
venosum

Arterial supply
R

• Hepatic artery
M

Venous

• Hepatic veins
• Portal vein

Nervous supply

• Sympathetic and parasympathetic trunks of coeliac plexus


Question 234 of 235

The following statements regarding the rectus abdominis muscle are true except:

It runs from the symphysis pubis to the xiphoid process

Its nerve supply is from the ventral rami of the lower 6 thoracic nerves

It has collateral supply from both superior and inferior epigastric vessels

h
It lies in a muscular aponeurosis throughout its length

la
It has a number of tendinous intersections that penetrate through the anterior layer of the
muscle Sa
Rectus abdominis

• Arises from the pubis.


• Inserts into 5th, 6th, 7th costal cartilages.
• The muscle lies in the rectal sheath, which also contains the superior and inferior epigastric
artery and vein.
• Action: flexion of thoracic and lumbar spine.
C
• Nerve supply: anterior primary rami of T7-12.

The aponeurosis is deficient below the arcuate line.


Please rate this question:
R

Discuss and give feedback


Next question
M

Abdominal wall

The 2 main muscles of the abdominal wall are the rectus abdominis (anterior) and the quadratus
lumborum (posterior).
The remaining abdominal wall consists of 3 muscular layers. Each muscle passes from the lateral
aspect of the quadratus lumborum posteriorly to the lateral margin of the rectus sheath anteriorly.
Each layer is muscular posterolaterally and aponeurotic anteriorly.
Image sourced from Wikipedia

Muscles of abdominal wall


External • Lies most superficially

h
oblique • Originates from 5th to 12th ribs
• Inserts into the anterior half of the outer aspect of the iliac crest, linea
alba and pubic tubercle
• More medially and superiorly to the arcuate line, the aponeurotic layer

la
overlaps the rectus abdominis muscle
• The lower border forms the inguinal ligament
• The triangular expansion of the medial end of the inguinal ligament is
the lacunar ligament.

Internal •
Sa
Arises from the thoracolumbar fascia, the anterior 2/3 of the iliac crest
oblique and the lateral 2/3 of the inguinal ligament
• The muscle sweeps upwards to insert into the cartilages of the lower 3
ribs
• The lower fibres form an aponeurosis that runs from the tenth costal
cartilage to the body of the pubis
C
• At its lowermost aspect it joins the fibres of the aponeurosis of
transversus abdominis to form the conjoint tendon.
R

Transversus • Innermost muscle


abdominis • Arises from the inner aspect of the costal cartilages of the lower 6 ribs ,
from the anterior 2/3 of the iliac crest and lateral 1/3 of the inguinal
ligament
M

• Its fibres run horizontally around the abdominal wall ending in an


aponeurosis. The upper part runs posterior to the rectus abdominis.
Lower down the fibres run anteriorly only.
• The rectus abdominis lies medially; running from the pubic crest and
symphysis to insert into the xiphoid process and 5th, 6th and 7th costal
cartilages. The muscles lies in a aponeurosis as described above.
• Nerve supply: anterior primary rami of T7-12

Surgical notes
During abdominal surgery it is usually necessary to divide either the muscles or their aponeuroses.
During a midline laparotomy it is desirable to divide the aponeurosis. This will leave the rectus
sheath intact above the arcuate line and the muscles intact below it. Straying off the midline will
often lead to damage to the rectus muscles, particularly below the arcuate line where they may often
be in close proximity to each other.

h
la
Sa
C
R
M
Question 235 of 235

During liver mobilisation for a cadaveric liver transplant the hepatic ligaments will require
mobilisation. Which of the following statements relating to these structures is untrue?

Lesser omentum arises from the porta hepatis and passes the lesser curvature of the
stomach

h
The falciform ligament divides into the left triangular ligament and coronary ligament

la
The liver has an area devoid of peritoneum

The coronary ligament is attached to the liver


Sa
The right triangular ligament is an early branch of the left triangular ligament

The right triangular ligament is a continuation of the coronary ligament.


Please rate this question:
C
Discuss and give feedback

Liver
R

Structure of the liver


Right lobe • Supplied by right hepatic artery

M

Contains Couinaud segments V to VIII (-/+Sg I)

Left lobe • Supplied by the left hepatic artery


• Contains Couinaud segments II to IV (+/- Sg1)

Quadrate lobe • Part of the right lobe anatomically, functionally is part of the left
• Couinaud segment IV
• Porta hepatis lies behind
• On the right lies the gallbladder fossa
• On the left lies the fossa for the umbilical vein

Caudate lobe • Supplied by both right and left hepatic arteries


• Couinaud segment I
• Lies behind the plane of the porta hepatis
• Anterior and lateral to the inferior vena cava
• Bile from the caudate lobe drains into both right and left hepatic ducts

Detailed knowledge of Couinaud segments is not required for MRCS

h
• Between the liver lobules are portal canals which contain the portal triad: Hepatic Artery,

la
Portal Vein, tributary of Bile Duct.

Relations of the liver


Anterior
Sa Postero inferiorly

Diaphragm Oesophagus

Xiphoid process Stomach


C
Duodenum
R

Hepatic flexure of colon

Right kidney
M

Gallbladder

Inferior vena cava

Porta hepatis
Location Postero inferior surface, it joins nearly at right angles with the left sagittal fossa, and
separates the caudate lobe behind from the quadrate lobe in front

Transmits • Common hepatic duct


• Hepatic artery
• Portal vein
• Sympathetic and parasympathetic nerve fibres
• Lymphatic drainage of the liver (and nodes)

h
Ligaments
Falciform ligament • 2 layer fold peritoneum from the umbilicus to anterior liver surface

la
Contains ligamentum teres (remnant umbilical vein)
• On superior liver surface it splits into the coronary and left
triangular ligaments

Ligamentum teres

Ligamentum
Sa
Joins the left branch of the portal vein in the porta hepatis

Remnant of ductus venosus


venosum

Arterial supply
C
• Hepatic artery

Venous
R

• Hepatic veins
• Portal vein
M

Nervous supply

• Sympathetic and parasympathetic trunks of coeliac plexus

You might also like